English

You might also like

Download as pdf or txt
Download as pdf or txt
You are on page 1of 144

stportal.mahendras.

org

English
Language
SALIENT FEATURES:

� Chapterwise Discussion
� Topicwise Rules and Examples
� Exercise based on previous papers.
� Easy and comprehensible language

Mahendra Publication Pvt. Ltd.


New Delhi

ENGLISH LANGUAGE 3
stportal.mahendras.org

Mahendra Publication Pvt. Ltd.

© Copyright Reserved
##No part of this issue can be printed in whole or in part without the written permission of the
publishers.
##All the disputes are subject to Delhi jurisdiction only.

Registered Office
Mahendra Publication Pvt. Ltd., 103, Pragatideep Building, Plot No. 08, Laxminagar,
District Centre, New Delhi - 110092, TIN-09350038898, w.e.f. 12-06-2014

Branch Office
Mahendra Publication Pvt. Ltd. E-42,43,44, Sector-7, Noida (U.P.)

For queries regarding promotion, distribution & advertisement, contact:-


E-mail-sales@mahendras.org
Ph.: 09208037962

Owned, printed & published by N.K. Jain


103, Pragatideep Building, Plot No. 08, Laxminagar, District Centre, New Delhi - 110092

Please send your suggestions and grievances to:-


Mahendra Publication Pvt. Ltd., CP-9, Vijayant Khand, Gomti Nagar Lucknow - 226010
E-mail:info@mahendras.org

4 ENGLISH LANGUAGE
stportal.mahendras.org

PREFACE
This gives us immense pleasure to present you the latest edition of this book. We thank you from the depth
of our hearts, for the love and affection given by you just from the beginning. Dear friends, change is the law
of nature. We must learn to tolerate the blows of time with patience and learn not only to endure, but also
to expect, welcome, and enjoy both the joys as well as the sorrows of life. We must do what we can to "get
success." We have to operate with the information and skills that are necessary for winning.
Today’s era is governed by technology. The technology has increased the pace of changing the world we see
day by day, and so the pattern of examination and criteria of selection has also changed. As we are aware
that interview is a part of the various examinations and the written/objective exams are going to be tougher
than earlier.
Every choice you make — including the thoughts you have — has consequences. When you start choosing
the right behaviour and thoughts — which will take a lot of discipline — you'll get the right outcomes.
Be aware of the factors that influence the way you see the world, so that you can deal with them and react
against them. You are your own most important resource for making your life work. Success is a moving
target that must be tracked and continuously pursued.
In this context we have completely updated this book keeping the forthcoming examination pattern in mind.
This edition caters to need of English Language that is asked in Preliminary and Mains stages of most of
the Banking and Insurance examinations for various posts. This book is thoroughly based on the latest pat-
tern in which time limit is given for each section. Every chapter in this book describes the concept with the
help of various examples and at the end gives miscellaneous examples to clear the concepts. Examples have
been solved with standard as well as short methods. At the end of each chapter exercises has been given to
master the topic by solving variety of exam oriented question. So we have now given a complete focus on
concept building and mastering each topic through this book.
We are truly dedicated to provide you the best among the rest. This book is an initiative from our side to
make you perfect in this subject.
We will be highly gratified, if this book helps students in getting selection.
Any suggestions related to the this book shall always be welcomed and we shall endeavor to incorporate
them in our upcoming issues.

Research Team

ENGLISH LANGUAGE 5
stportal.mahendras.org

CONTENTS
S.No. TOPICS Pg. No.

1. Introduction 7-10

2. Noun 11-15

3. Pronoun 16-20

4. Verb 21-28

5. Conjunction 29-33

6. Time and Tense 34-35

7. Rearrangement of Sentences 36-49

8. Preposition 50-56

9. Adjective 57-61

10. Cloze Test 62-71

11. Articles 72-76

12. Adverb 77-82

13. Error Detection 83-90

14. Vocabulary 91-98

15. Reading Comprehension 99-107

16. Fill in the Blanks 108-122

17. Phrase Replacement 123-131

18. Mis-Spelt and Inappropriate Words 132-137

19. Spotting Similar and Dissimilar words 138-145

6 ENGLISH LANGUAGE
stportal.mahendras.org

CHAPTER

1 introduction
Scan the QR code to get video of this chapter.

They worry that they won't say things correctly or that


they will look stupid so they don't talk at all. Don't do this.
Why to learn English? The fastest way to learn anything is to do it again and
again until you get it right. Like anything, learning English
requires practice so don't let fear stop you from getting
what you want.
Reading Skills
Reading is the most important skill in English language which is
tested in various competitive exams to ensure that the candidate
can understand documents(in English) and write in English.
Reading is the ability of deriving the meaning from written
1. You can get more knowledge. or printed text.
If you know English well, an incredible amount of The understanding ability of the reader depends on-
resources becomes accessible: books, courses, videos
1. Reader’s prior knowledge.
and of course, a great part of the Internet.
2. Knowledge that reading makes sense.
2. You may get a better job if you know English.
3. This knowledge helps to recognize the words and their
Many positions require you to be fluent in English writing
meanings.
or speaking. In such cases, your knowledge of English is
an advantage. It can help you get a better job and advancement 4. Clarify confusing parts of the text.
further in your career. 5. Skilled readers may predict what will happen next in a
Approach to Learning English story/ text by using clues presented in the text.
People must have a clear purpose of learning EnglishThey Comparison Of Slow And Efficient Readers
must know why they want to learn English. The awareness
EFFICIENT READER SLOW READER
for learning gives us the right motivation and focus. English
can be learned for several purposes. It would be a good idea Reads ideas Reads words
to identify one's reason to learn English in order to encourage Reads multiword phrases Reads one word at a time
oneself to continue learning English .
Visualizes ideas Reads to ‘the end of
English is one of the most widely spoken languages around the book’
the world as well as the most taught. If you learn English
Sets a purpose No purpose
well, you will be able to get ahead professionally.
Adjusts reading Reads everything
How To Learn English
speed slowly and deliberately.
##Read as many English articles (i.e. stories, newspapers
and texts) as you can on a regular basis. Keeps reading Rereads the sentences
##Try writing texts on various topics in English to improve while understanding so as to be sure of
your understanding of grammar. understanding
##Watch movies and TV programmes and listen to radio Has large vocabulary Has a limited
programmes in English.
in that subject area. vocabulary in that area
##Try enhancing your vocabulary by learning new words.
Practices speed Rarely attempts
##Whenever you find new words, find their meanings in
reading daily. speed reading.
the dictionary and use them in writing.
Marks text for memory Reads everything
##Try to speak English without fear. The biggest problem
irregularly
most people face in learning English is their own fear.

ENGLISH LANGUAGE 7
stportal.mahendras.org
at that dress! Isn't it beautiful? I want that dress, but I don't
Speed Reading
have enough money."
##Can help us to read and understand written information
Veena called: "What are you talking about? This is an ugly
much more quickly.
dress! It is just horrible! I don't even want to see this dress."
##Is to read large volumes of information quickly.
"Ok, ok…" Mina whispered sadly.
##Saves time.
Suddenly Veena called: "Oh my god! Look at this dress! It
Facts that reduce Reading Speed is beautiful! I want this dress. Oh, but look at the price. It is
Word-by-word reading. too expensive for me."
##Expanded length of time spent on reading each Now Mina called: "What are you talking about? This is an
block. ugly dress! It is really horrible! I don't even want to see it."
##Faulty eye movement regression. "Ok, ok…" Veena whispered sadly.
##Missing concentration. Now Mina is sad, and Veena is sad too. They are walking
home. They have no new clothes, but they know that next
##Lack of practice in reading.
time they should respect each other’s opinions.
##Fear of losing comprehension.
Vocabulary
##Reading slowly.
##Whisper - speaking softly
##Poor evaluation of important and unimportant
asepects. Garima's Acting

##The effort to remember everything rather than to Garima works as a secretary.


remember selectively. She answers phone calls and
types letters. She makes coffee,
Tips for increasing Reading Speed and goes to the post office.
##Work on vocabulary improvement. She doesn't like her work. She
##Determine your purpose before reading. is bored! "I would like to do
something more exciting," she
##Try to read faster than normal speed. thinks, "I want an exciting job!"
##Move your hand or index finger as you read along . Garima actually wants to be an actress. She goes to auditions,
##Use eyes efficiently. but she doesn't pass any. She is confused. She doesn't know
what to do to solve this problem, so she decides to ask her
##Increase your vocabulary by using a dictionary as
friend Ritu.
and when required.
Ritu is a good actress. She is not very famous, but she is more
##Do not vocalize the words.
successful than Garima.
##Do not re-read words.
Garima asks Ritu to look at her acting and tell her what the
##To improve reading speed , practice for about 15 problem is.
to 30 minutes each day.
Ritu is happy to help. She watches Garima carefully. She
##Train yourself to read more quickly by setting takes notes. In the end, Garima asks her: "Well… be honest!
time to read. What is wrong with me?"
In the next chapter some pieces of texts are given to read Ritu smiles and says: "Garima, you are a wonderful actress.
and improve reading skills which would be of great help in You act very well. There is only one problem."
cracking the exam.
"What? What is the problem?" Garima asks.
Texts For Reading "Well," Ritu answers, "You act very well, but you speak too
What are you talking quietly. I can't hear a word!"
about ? Catching A Cold
Mina and Veena are Many people catch a cold
walking to the mall. in the springtime and fall
They want to buy new ill. It makes us wonder... if
clothes. Mina has some scientists can send a man to
money and Veena has the moon, why can't they find
some money too. a cure for the common cold.
Suddenly, Mina called: The answer is easy. There
"Veena! Veena! Look are literally hundreds of cold
8 ENGLISH LANGUAGE
stportal.mahendras.org
viruses out there. You never know which one you will get, Their parents often told them, "You are not meant to go to
so there isn't a cure for each one. school. What use is school for carrying garbage which is
When a virus attacks your body, your body works hard to get your only job?"
rid of it. Blood rushes to your nose and brings congestion with "Don't go near those people; they are high born, we are low
it. You feel terrible because you can't breathe well, but your born."
body is actually "eating" the virus. Your temperature rises and Ekalavya didn't understand.
you get a fever, but the heat of your body is killing the virus.
You also have a running nose to stop the virus from getting "Why, but why?" he asked his mother. She replied "God set
to your cells. You may feel miserable, but actually your these limits."
wonderful body is doing everything it can to kill the cold. "God! Why would God want nice things for them and dirty
Different people have different remedies for cold. In the things for us? Hasn't God made all of us?"
United States and some other countries, for example, people Ekalavya asked.
might eat chicken soup to feel better. Some people take hot
She sighed, "Darling I don't know, but there are bounds we
baths and drink warm liquids. Other people take medicines
cannot cross, this is God's rule."
to stop the fever, congestion, and runny nose.
Ekalavya became quiet. From that day the only important
There is one interesting thing to note- some scientists say
thing for him was to understand the meaning of "limit".
taking medicines when you have a cold is actually bad for
you. The virus stays in you longer because your body doesn't You know, kids (and grown ups) are sometimes cruel. One
have a way to fight it and kill it. Bodies can do an amazing job day Ekalavya and his friends trapped a little ant and were
on their own. There is a joke, however, on taking medicine watching it try to escape. The ant tried and tried till it found
when you have a cold. It goes like this: a little opening at the edge of the trap and escaped.
It takes about 1 week to get over a cold if you don't take Other children went to trap it again but Ekalavya stopped
medicine, but only 7 days to get over a cold if you take medicine. them shouting, "The brave ant has broken the bounds. Limits
are meant to be broken. I am free, free".
Vocabulary
One day, Ekalavya saw beautiful chariots come into the forest
cure something that makes you well after being sick near his village. He saw boys of his age get out of the chariot
literally really, actually, exactly one by one. What lovely clothes they wore!
virus that causes sickness Last, an old man with snowy white hair and spotless white
clothes came out looking stern and calm. The boys seemed a
to get rid of to stop, to throw away
little scared of the old man. Ekalavya heard his father's voice,
congestion stopped up (when your nose is congested "Come away boy, there is work to do. Those are the Kuru
you can't breathe) princes, with their teacher Drona. The boy he just patted is
miserable very terrible feeling his favourite, Arjuna. They have come to practice 'archery'.
fever heat in your head and body Don't go near them."
remedy cure, something that makes you well "Father, all I want is a pat from the grand old man." so saying,
liquid wet and runny like water, milk Eklavya ran towards the old man Drona.
to get over to finish By then the boys had started shooting with bows and arrows.
What amazing things Arjuna did! He could shoot at a target
Eklavya
with his eyes shut. He could shoot with his left hand as well
Ekalavya was a little boy, as the right. And the teacher Drona? His arrows made fire,
born in a poor family, chased things in circles, brought rain and lightning. It was
many many years ago. magical.
His family lived a little
Arjuna's eyes never left his teacher. It was like he wanted to
away from Hastinapura,
absorb every bit of Drona. And you could see that Drona too
the capital of the Kuru
was very proud of Arjuna.
kings. They used to
clean other people's dirt The practice ended. Ekalavya went up to Drona and with
for a profession. folded hands said, "Great sir, please teach me".
And for this reason they "I don't teach the low-born," was the cold reply as Drona
were shunned by society. turned away.
Ekalavya and other kids "Master, your arrows don't seem to mind any limits, they
of his group knew they bring rain and fire, they bring night and day. How then can
too had to follow their you be bound by stupid thoughts of high and low birth?"
parents' professions.
ENGLISH LANGUAGE 9
stportal.mahendras.org
With this Ekalavya walked away from the glaring Drona. Ekalavya, you have taught me this." but with one dusky
The next day, Ekalavya carved a statue of Drona on a tree handsome boy, the old man's behaviour was different, he
trunk with a knife. He made himself a bow and arrows. Each smiled and patted this boy on the head.
day he would bow before the statue, practise shooting and Vocabulary
imagine a pat on his back from Drona.
Away far
Some months passed. The grand princes and their master
came to the forest again. As Arjuna reached to take aim at a Shunned avoided
particularly difficult target, an arrow reached Arjuna's target Garbage waste
before he could even aim. Shocked the boys and their teacher Scared afraid of
looked around. They saw Ekalavya, who went up to touch
Patted tapped with hands
Drona's feet.
Absorb learn
"Who is your teacher?" Drona asked. Ekalavya quietly led
him to the statue. Drona looked at it for some time and said, Courage bravery
"If I am your Guru, give me my fee, my guru dakshina". The Man Who Saved The Moon
"Ask, sir", bowed Ekalavya. "I want your right thumb," A kind-hearted man, looking
replied the stone hearted Drona. Without the right thumb to down into a well, saw the
support it, how can any archer ever hold a bow?As Arjuna reflection of the moon in the
and the other princes watched in shock, Ekalavya wordlessly water.
cut off his right thumb and laid it at Drona's feet. "Oh! My God! The moon
Many years passed. A great war was on in Kurukshetra has fallen into the well," he
between the Kauravas and the Pandavas, cousins who were muttered mournfully and
related. Arjuna was on the side of the Pandavas. His teacher hurried to fetch a hook tied to
Drona, and many of his cousins, who had been Drona's the end of a long rope.
students with him, were on the other side. Quickly, he let the hook fall
After the day's battle, a sad Drona was sitting in his camp. deep into the well, holding fast
Suddenly, like a respectful prayer, arrows fell near his feet to the rope.
one after another. He looked up and who do you think he saw? The hook hit the water and,
Why, Ekalavya ! The young Ekalavya said, "Master I have reaching the bottom of the well, caught fast to a stone.
learned to shoot without my right thumb. I have learned to Thinking that he had got the moon, the man pulled up the
shoot with my left hand, and with my feet. I have taught rope with such force that it broke and he fell sharply down
others and raised an army. Today I'm known as a great archer." flat on his back, nearly unconscious.
Drona was speechless. The first thing he saw when he recovered was the moon,
"Master, I set your greatness free from the limits of your which now shone serenely high up in the sky.
own mind. I made it my teacher in the form of your statue. The man groaned in pain but said contentedly, "I broke my
Your great love for Arjuna crossed the bounds of fairness back, but, thank God, the moon is saved."
when you asked for my thumb. You thought that it would
finish me as an archer. But great masters always end up
Vocabulary
teaching something. By asking for my thumb, you made me Reflection image
learn to shoot with both hands and feet. And for this lesson,
I offer my services to you in this war." Muttered say something unclearly
Drona's eyes filled with tears. He answered, "Yes, it is true, Serenely calmly
bounds and limits are in the mind. Real courage is fighting
against wrong limits and respecting the correct ones. Groaned cried

10 ENGLISH LANGUAGE
stportal.mahendras.org

CHAPTER

2 nOUN
Scan the QR code to get video of this chapter.

A noun is a word that names a person, a place or a thing.


The Collective Noun
Examples:
Collective Noun is a special word that shows a collection of
common nouns.
Check Out The Chart Below:

army audience board

company corporation council

department faculty family

firm group jury

majority minority navy

public school senate

society team troupe


Sarah, lady, cat, New York, Canada, room, school, football, Use correct verbs and pronouns with collective nouns.
reading.
Each noun from the list above is a single thing. That thing,
Examples: however, is made up of more than one person. You cannot
##People like to go to the beach. have a committee, team, or family of one; you need at least
two people to compose the unit.
##Sheena passed the test.
Because people behave as both herd animals and solitary
##My parents are travelling to Jaipur next month.
creatures, collective nouns can be either singular or plural,
The word "noun" comes from the Latin word nomen which depending on context. In writing, this double status often
means "name," and nouns are indeed how we name people, causes agreement errors. How do you tell if a collective noun
places and things. is singular or plural? What verbs and pronouns do you use
with the collective noun?
Types Of Nouns
Here is the key: Imagine a flock of pigeons pecking at
Proper Noun birdseed on the ground. Suddenly, a cat races out of the
A proper noun is the special word (or name) that we use for a bushes. What do the pigeons do? They fly off as a unit in an
person, place or an organization, like Ravi, Anita, Lucknow, attempt to escape the predator, wheeling through the sky in
India or Sony. the same direction.
Common Noun People often behave in the same manner, doing one thing in
A common noun begins with a lowercase letter unless it is at unison with the other members of their group. When these
the beginning of a sentence. As it refers to a common name. people are part of a collective noun, that noun becomes
singular and requires singular verbs and pronouns. As you
For example:- read the following examples, notice that all members of the
People:- man, girl, boy, mother, father, child, person, teacher, collective noun are doing the same thing at the same time:
student, etc. Every afternoon the baseball team follows its coach out to
Animals:- cat, dog, fish, ant, snake, etc. the hot field for practice.

Things:-book, table, chair, phone, etc. Team = singular; follows = a singular verb; its = a singular
pronoun. All members of the team arrive at the same place
Places:- school, city, building, shop, etc. at the same time.

ENGLISH LANGUAGE 11
stportal.mahendras.org
Today, Dr. Gupta's class takes its first 100-item exam.
Material Noun
Class = singular; takes = a singular verb;its = a singular
Material Noun is the name of a material or a substance or an
pronoun. All members of the class are testing at the same
ingredient of an alloy.
time.
Examples :
The jury agrees that the state prosecutors did not provide
enough evidence, so its verdict is not guilty. 1. This dress is made of silk

Jury = singular; agrees = a singular verb;its = a singular 2. This ring is made of gold .
pronoun. All members of the jury are thinking the same 3. This is a shop for diamonds.
way.
4. Calcium is good for health.
Now imagine three house cats in the living room. Are
the cats doing the same thing at the same time? Not this They are called Material Nouns.
group! One cat might be sleeping on top of the warm They may be placed in groups as follows:
television. Another might be grooming on the sofa.
The third animal might be perched on the windowsill, (1) The metals: iron, gold, platinum, etc.
watching the world outside. There is one group of (2) Products spoken of in bulk: tea, sugar, rice, wheat, etc.
animals, but the members of that group are all doing
their own thing. (3) Geological bodies: mud, sand, granite, rock, stone, etc.

Team = plural; shower, change, head = plural verbs; their (4) Natural phenomena: rain, dew, cloud, frost, mist, etc.
= a plural pronoun. The teammates are dressing into their (5) Various manufactures: cloth (and the different kinds
individual outfits and leaving in different directions for their of cloth), potash, soap, rubber, paint, celluloid, etc.
individual homes.
Countable Nouns
After the long exam, the class started their research papers
on famous mathematicians. A countable noun is a noun that indicates something you
could actually count.
Class = plural; start = a plural verb; their = a plural
pronoun. The students are beginning their own research Cup is a countable noun.
papers—in different places, at different times, on different For example, you can count cows: one cow, two cows, three
mathematicians. cows...
The jury disagree about the guilt of the accused and have told However, you can't count water: one water, two water – no,
the judge that they are hopelessly deadlocked. it doesn't work...
Jury = plural; disagree, have told = plural verbs; they = a A countable noun has both a singular and a plural form, and
plural pronoun. Not everyone on the jury is thinking the it can be used with the indefinite articles (a/an).
same way.
Examples: Window, teacher, tree, lion, eye, cloud, pencil,
Whenever you cannot decide if a collective noun heart, movie.
is singular or plural, exercise your options as a
writer. You have two ways that you can compose the Uncountable Nouns
sentence without causing an agreement error: 1) insert
An uncountable noun is a noun that indicates something you
the word members after the collective noun [jury
cannot count.
members, committee members, board members], or
2) use an entirely different word [players instead of Water is an uncountable noun.
team, students instead of class, soldiers instead of
Examples: Furniture, advice, mail, news, equipment, luggage,
army]. Then you can use plural verbs and pronouns
work, coffee, information.
without worrying about making mistakes or sounding
unnatural. Gender:

Abstract Noun Masculine Gender: Nouns which are the names of males.

An abstract noun is a noun that you cannot sense, it is the Example: tiger, duke, husband, gentleman, boy.
name we give to an emotion or idea. They have no physical Feminine Gender: Nouns which are the names of females.
existence, you can't see, hear, touch, smell or taste them.
Example: tigress, lady, lass, wife, girl.
For example:-
Neuter Gender: Nouns which are neither male nor female.
Justice; an idea, bravery and happiness are all abstract
nouns. Example: box, book, desk, bench, book, barn.

12 ENGLISH LANGUAGE
stportal.mahendras.org
Common Gender: Nouns for which the gender of an object The following nouns have no singular:
is not known.
scissors oats mumps
Example: neighbour, parent, friend, bird, people, cousin.
trousers Measles cattle Tongs
Singular And Plural Noun
Nouns may be singular referring to one, or plural, referring Some nouns are always singular in form.
to more than one.
Gold Silver Wheat
Most nouns change their form by adding “s” when they are
written in plural. Corn Molasses Copper

Singular Noun Definition: When a noun means only one, it Sugar Cotton Cattle
is said to be singular.
Sheep Deer
Examples: boy, girl, book, church, box
Singular nouns use this and that.
Plural Noun Definition: When a noun means more than one,
it is said to be plural. Plural nouns use these and those.

Examples: boys, girls, books, churches· Possessive Nouns


Irregular Plurals Common and proper nouns can sometimes be further
classified as possessive nouns. A possessive noun shows
man, men foot, feet mouse, mice ownership, belonging, or that something is part of something
else. They are easy to find in a sentence because they always
woman, women tooth, teeth goose, geese
include an apostrophe (’).
child, children ox, oxen

ENGLISH LANGUAGE 13
stportal.mahendras.org

exercise
Q.6. All the furnitures of my house have been stolen.
EXERCISE 1
Q.7. A poet and writer are dead.
Correct the following sentences
Q.8. The committee was divided on this issues.
Q.1. Her hairs are curly.
Q.9. Mathematics are not a dificuit subject.
Q.2. Our elders gave us many advices.
Q.10. My sympathies are always with the poor.
Q.3. I have many work to do.
Q.4. Sita has sold all her furnitures EXERCISE 4
Q.5. She gave me informations about this bank. Correct the following sentences
Q.6. The sceneries of Himachal Pradesh are very charming. Q.1. His Mathematics are weak.
Q.7. The peoples are hard working. Q.2. I go for a two miles walk daily.
Q.8. The cattles are grazing in the field. Q.3. Politics are a very interesting subject.
Q.9. I want a paper. Q.4. There are four breads in the kitchen.
Q.10. I bring fruits and vegetables from the market. Q.5. One should not hate the poors.
Q.6. The magistrate passed order of his release.
EXERCISE 2
Q.7. She has committed not one but many mischiefs.
Correct the following sentences
Q.8. The first inning is going to over now.
Q.1. He got only passing mark.
Q.9. There is no place in this compartment.
Q.2. Ram was true to his words
Q.10. He bought some stationeries.
Q.3. Ten miles are a long distance.
Q.4. They want two dozens apple. EXERCISE 5
Q.5. I want two pairs of white shoes. Correct the following sentences
Q.6. Rahul has finished two-third of his work. Q.1. He is a sixty years old man.
Q.7. The chair’s legs are broken. Q.2. A five men committee had a three hours meeting
yesterday.
Q.8. Good night, I am so glad to meet you.
Q.3. An all parties meeting was called to consider the drought
Q.9. There is no space in the car. situation in the country.
Q.10. We should help the poors. Q.4. There is two ways traffic here, so no problem of traffic
EXERCISE 3 blockage.
Correct the following sentences Q.5. How can our country accept the two nations theory?
Q.1. I brought my furnitures by goods train. Q.6. He has gone to market to purchase vegetable.
Q.2. I saw two beautiful fishes in the pond. Q.7. He said, ‘Riches has wings’.
Q.3. Thomson’s poetries are very charming. Q.8. I asked him where his spectacles was.
Q.4. Is your scissors dull? Q.9. She said that she was suffering from measle.
Q.5. Light travel faster than sound. Q.10. She said, ‘Please give him an alms’.

14 ENGLISH LANGUAGE
stportal.mahendras.org

explanation
Q.7. A poet and writer is dead.
EXERCISE 1
Q.8. The committee were divided on this issue.
Q.1. Her hair is curly
Q.9. Mathematics is not a dificult subject.
Q.2. Our elders gave us many pieces of advice
Q.10. My sympathy is always with the poor.
Q.3. I have much work to do.
Q.4. Sita has sold all her furniture. EXERCISE 4
Q.5. She gave me information about this bank. Q.1. No error (Here mathematical abilities are being
discussed)
Q.6. The scenery of Himachal Pradesh is very charming.
Q.2. I go for a two-mile walk daily.
Q.7. The people are hard working.
Q.3. Politics is a very interesting subject.
Q.8. The cattle are grazing in the field.
Q.4. There are four loaves of bread in the kitchen.
Q.9. I want a piece of paper.
Q.5. One should not hate the poor.
Q.10. I bring fruit and vegetables from the market.
Q.6. The magistrate passed orders of his release.
EXERCISE 2
Q.7. She has committed not one but many acts of mischief.
Q.1. He got only pass marks.
Q.8. The first innings is going to be over now.
Q.2. Ram was true to his word.
Q.9. There is no room in this compartment.
Q.3. Ten miles is a long distance.
Q.10. He bought some stationery.
Q.4. They want two dozen apples.
EXERCISE 5
Q.5. I want two pair of white shoes.
Q.1. He is a sixty-year old man.
Q.6. Rahul has finished two-thirds of his work.
Q.2. A five men committee had a three-hour meeting
Q.7. The legs of the chair are broken. yesterday.
Q.8. Good evening, I am so glad to meet you. Q.3. An all party meeting was called to consider the drought
Q.9. There is no room in the car. situation in the country.
Q.10. We should help the poor. Q.4. There is two-way traffic here so no problem of traffic
blockage.
EXERCISE 3
Q.5. How can our country accept the two-nation theory.
Q.1. I brought my furniture by goods train.
Q.6. He has gone to market to purchase vegetables.
Q.2. I saw two beautiful fish in the pond.
Q.7. He said, ‘Riches have wings’.
Q.3. Thomson’s poetry is very charming.
Q.8. I asked him where his spectacles were.
Q.4. Are your scissors dull?
Q.9. She said that she was suffering from measles.
Q.5. Light travels faster than sound.
Q.10. She said, ‘Please give him alms’.
Q.6. All the furniture of my house have been stolen.

ENGLISH LANGUAGE 15
stportal.mahendras.org

CHAPTER

3 pRONOUN
Scan the QR code to get video of this chapter.

A pronoun is a word that is used instead of a noun. In the following sentences, each of the highlighted words is
For example, you could say "Rita is a nice girl." an objective personal pronoun: After reading the pamphlet,
Mala threw it into the garbage can.
Then you could replace the noun “ Rita" with the word "She"
and get the following sentence: The pronoun "it" is the direct object of the verb "threw."

"She is a nice girl." The agitated assistant stood up and faced the angry delegates
and said, "Our leader will address you in five minutes."
Here, "She" is a pronoun.
In this sentence, the pronoun "you" is the direct object of the
Examples: verb "address."
I, he, it, we, them, us, mine, itself. Give the list to me.
Example sentences: Here the objective personal pronoun "me" is the object of
He doesn't want to go with them. the preposition "to."
Would they help us? c) Possessive Personal Pronouns
His house is bigger than ours. A possessive pronoun indicates that the pronoun is acting as
a marker of possession and defines who owns a particular
Who is she?
object or person. The possessive personal pronouns are
The word "pronoun" comes from "pro" (in the meaning of "mine," "yours," "hers," "his," "its," "ours," and
"substitute") + "noun." "theirs." Note that possessive personal pronouns are very
Grammarians classify pronouns into several types, similar to possessive adjectives like "my," "her," and "their."
including the personal pronoun, the demonstrative In each of the following sentences, the highlighted word is
pronoun, the interrogative pronoun, the indefinite a possessive personal pronoun:
pronoun, the relative pronoun, the reflexive pronoun,
The smallest gift is mine.
and the emphatic pronoun.
Here the possessive pronoun "mine" functions as a subject
Personal Pronouns complement.
A personal pronoun refers to a specific person or thing and
Demonstrative Pronouns
changes its form to indicate person, number, gender, and
case. A demonstrative pronoun points to and identifies a noun or
a pronoun. "This" and "these" refer to things that are nearby
A) Subjective Personal Pronouns either in space or in time, while "that" and "those" refer to
A subjective personal pronoun indicates that the pronoun is things that are farther away in space or time.
acting as the subject of the sentence. The subjective personal The demonstrative pronouns are "this," "that," "these,"
pronouns are "I," "you," "she," "he," "it," "we," "you," and "those." "This" and "that" are used to refer to singular
"they." nouns and "these" and "those" are used to refer to plural
nouns. Note that the demonstrative pronouns are identical to
In the following sentences, each of the highlighted words is
demonstrative adjectives, though, obviously, you use them
a subjective personal pronoun and acts as the subject of the
differently. It is also important to note that "that" can also be
sentence:
used as a relative pronoun.
I have finished my work on time. In the following sentences, each of the highlighted words is
You are surely the strangest child I have ever met. a demonstrative pronoun:
b) Objective Personal Pronouns This must not continue.
An objective personal pronoun indicates that the pronoun Here "this" is used as the subject of the compound verb "must
is acting as an object. The objective personal pronouns are: not continue."
"me," "you," "her," "him," "it," "us," "you," and Three customers wanted these.
"them." Here "these" is the direct object of the verb "wanted."
16 ENGLISH LANGUAGE
stportal.mahendras.org
The crate which was left in the corridor has now been moved
Interrogative Pronouns
into the storage closet.
An interrogative pronoun is used to ask questions. The
In this example "which" acts as the subject of the compound
interrogative pronouns are "who," "whom," "which,"
verb "was left" and introduces the subordinate clause "was
"what" and the compounds formed with the suffix "ever"
left in the corridor." The subordinate clause acts as an
("whoever," "whomever," "whichever," and "whatever").
adjective modifying the noun "crate."
Note that either "which" or "what" can also be used as an
interrogative adjective, and that "who," "whom," or "which" Indefinite Pronouns
can also be used as a relative pronoun.
An indefinite pronoun is a pronoun referring to an identifiable
You will find "who," "whom," and occasionally "which" used but not specified person or thing. An indefinite pronoun
to refer to people, and "which" and "what" used to refer to conveys the idea of all, any, none, or some.
things and to animals.
T he most common indefinite pronouns are "all,"
"Who" acts as the subject of a verb, while "whom" acts as "another," "any," "anybody," "anyone," "anything,"
the object of a verb or a preposition. "each," "everybody," "everyone," "everything,"
The highlighted word in each of the following sentences is "few," "many," "nobody," "none," "one," "several,"
an interrogative pronoun: "some," "somebody," and "someone." Note that
some indefinite pronouns can also be used as indefinite
Who wrote the novel Rockbound? adjectives.
Here "who" is the subject of the sentence. The highlighted words in the following sentences are
Whom do you think we should invite? indefinite pronouns:
In this sentence, "whom" is the object of the verb "invite." Many were invited to the lunch but only twelve showed up.
Who will meet the delegates at the railway station? Here "many" acts as the subject of the compound verb "were
In this sentence, the interrogative pronoun "who" is the invited."
subject of the compound verb "will meet." The office had been searched and everything was thrown
To whom did you give the paper? onto the floor.

In this example the interrogative pronoun "whom" is the In this example, "everything" acts as a subject of the
object of the preposition "to." compound verb "was thrown."

What did she say? Reflexive Pronouns


Here the interrogative pronoun "what" is the direct object of You can use a reflexive pronoun to refer back to the subject
the verb "say." of the clause or sentence.

Relative Pronouns Reflexive pronouns are "myself," "yourself," "herself,"


"himself," "itself," "ourselves," "yourselves," and
You can use a relative pronoun which is used to link one "themselves."
phrase or clause to another phrase or clause. The relative
pronouns are "who," "whom," "that," and "which." The Note: each of these can also act as an emphatic pronoun
compounds "whoever," "whomever," and "whichever" are when placed after the subject.
also relative pronouns. The highlighted word in the following sentence is a reflexive
You can use the relative pronouns "who" and "whoever" to pronoun:
refer to the subject of a clause or sentence, and "whom" and Diabetics give themselves insulin shots several times a day.
"whomever" to refer to the objects of a verb or a preposition.
Emphatic Pronouns
In each of the following sentences, the highlighted word is
a relative pronoun. An emphatic pronoun is a pronoun used to emphasise its
antecedent. Emphatic pronouns are used after the subject of
The candidate who wins the greatest popular vote is not the sentence.
always elected.
The highlighted words in the following sentences are
In this sentence, the relative pronoun is the subject of the emphatic pronouns:
verb "wins" and introduces the subordinate clause "wins
the greatest popular vote." This subordinate clause acts as I myself believe that honesty is the best policy.
an adjective modifying "candidate." T he Prime Minister himself said that he would lower
Whoever broke the window will have to replace it. taxes.

H ere "whoever" functions as the subject of the verb They themselves promised to come to the party even though
"broke." they had a final exam at the same time.

ENGLISH LANGUAGE 17
stportal.mahendras.org
Rule 6.
Rules Of Pronouns
R eflexive pronouns - myself, himself, herself, itself,
Rule 1.
themselves, ourselves, yourself, yourselves- should be used
Subjective pronouns are used when the pronoun is the only when they refer back to another word in the sentence.
subject of the sentence. You can remember subject pronouns
Correct: I pushed myself on the project.
easily by filling in the blank subject space for a simple
sentence. Incorrect: My brother and myself did it.
Example: The word myself does not refer back to another word.
I, you, he, she, it, we, and they all fit into the blank and are, Correct: My brother and I did it.
therefore, subject pronouns. Incorrect: Please give it to Jay or myself.
Rule 2. Correct: Please give it to Jay or me.
Subject pronouns are also used if they rename the subject. Sometimes we use the word one as an adjective, as in "I'll
They follow to be verbs such as is, are, was, were, am, and have just one scoop of ice-cream," and we seldom have
will be. trouble with that usage. But we also use one as a pronoun,
Examples: and this is where "one" becomes surprisingly complex.
It is we who are responsible for the decision to downsize. S ometimes the pronoun one functions as a numerical
expression:
Rule 3.
These are lovely dresses. I think I'll buy one.
Objective pronouns are used everywhere else (direct object,
indirect object, object of the preposition). Object pronouns One must work hard.
are me, you, him, her, it, us, and them. One is purple, the other is green.
Examples: Jyoti talked to him. The three brothers get along quite well; in fact they adore
Are you talking to me? one another.
Rule 4. One of the senators will lead the group in the senate.
To decide whether to use the subject or object pronoun after The yellow car is fast, but I think the blue one will win.
the words than or as, logically complete the sentence. As a pronoun, one can also function in an impersonal,
Examples: objective manner, standing for the writer or for all people
who are like the writer or for the average person or for all
Tripty is as smart as she/her. people who belong to a class.
If we logically complete the sentence, we would say, "Tripty One would think the airlines would have to close down.
is as smart as she is." Therefore, she is the correct answer.
If one fails, then one must try harder next time.
Zoya is taller than I /me.
One's Reflexive and Possessive Forms
Logically completing the sentence, we have, "Zoya is taller
than I am." One must be conscientious about one's dental hygiene.

Dipti would rather talk to her than I/me. One must learn from one's mistakes.

We can logically complete this sentence in two ways: "Dipti The Plural of One
would rather talk to her than to me." OR "Dipti would rather To pluralize one:
talk to her than I would." As you can see, the meaning will I really like the bright ones.
change depending on the pronouns you choose.
Are these the ones you want?
Rule 5.
Do you want these ones?
P ossessive pronouns show ownership and never need
apostrophes. The phrases "one in [plural number]" and "more than one"
always take a singular verb:
Possessive pronouns: mine, yours, his, hers, its, ours, theirs
One in four dentists recommends this toothpaste.
NOTE
One out of every five instructors gets this question wrong.
The only time 'it' has an apostrophe is when it is a contraction
for it is or it has. There is more than one reason for this.

Examples: More than one lad has lost his heart to this lass.

It's a cold morning. The "one" in the phrase "more than one" apparently controls
the number of the verb.
The thermometer reached its highest reading.
18 ENGLISH LANGUAGE
stportal.mahendras.org

exercise
Q.7. That is my uncle, _____ car was stolen.
EXERCISE 1
Q.8. That woman, _____ you saw, was my auntie.
Fill in the blanks with suitable pronouns.
Q.9. Kangaroos, _____ use their pouch to carry their
Q.1. Does ____ (her, she) know that ____ (me, I) was absent?
babies, are found in Australia.
Q.2. Please tell _____ ( he, him) _____ (I, me) have ob
Q.10. The policeman, ______ caught the thief, is a very
tained a degree in Chemistry.
brave man.
Q.3. I remember that _____ (they, them) bought the fruits
from _____ (we, us)
EXERCISE 4
Correct the following sentences
Q.4. Please don’t tell ____ (she, her) about _____ (I, me).
Q.1. All which she said was incorrect.
Q.5. _____ (It, its) can swim because _____ (it, them) has
webbed feet. Q.2. Who was the girl which came to meet you?
Q.6. I met Jaya yesterday. (she, her) _____ invited (me, Q.3. What is his opinion about the work which you have
I) _____ to her house. completed?
Q.7. Shreya has a cat; (she, her) _____ likes to play with Q.4. I always purchase the item which is best in the mar
(him, it) _____. ket.
Q.8. When the dog chased Jayant, (he, him) _____ ran as Q.5. This is the same shirt which I saw in the shop.
fast as _____ (he, him) could. Q.6. Who is she who can abuse me like this?
Q.9. My uncle works in a factory.(he, it) _____ says _____ Q.7. Only those employees should be promoted whom
(its, it) is a noisy place. are sincere.
Q.10. The teacher said to the class, “When (you, your) Q.8. The leaders which do not love their country are not
_____ finish your work, please pass _____ (it, them) respected.
up to me.” Q.9. Those whose live in glass houses should not throw
stones at others.
EXERCISE 2
Q.10. What was the reply which she submitted ?
Fill each blank with a suitable Reflexive Pronoun.
Q.1. He rewarded _____ with an ice-cream.
EXERCISE 5
Q.2. They agreed amongst _____ that they would not tell Find the errors and justify your answer
anyone. Q.1. The board of directors (A)/ want the facilities (B)/
Q.3. I taught _____ to draw. of car and accommodation (C)/ for itself (D).
Q.4. You must explain _____ more clearly. Q.2. Every teacher (A)/ and every student (B)/ of this
school is determined to do their best (C)/ for the
Q.5. He has a bad temper so he must learn to control benefit of all (D)
_____.
Q.3. The officer (A)/ as well as the (B)/ clerks absented
Q.6. We thoroughly enjoyed _____ at the party on Sunday.
themselves (C)/ from the office (D).
Q.7. That monkey is scratching _____.
Q.4. There were (A)/ five active workers (B)/ and three
Q.8. One must defend _____ against bullies. lazy one (C)/ in the factory (D).
Q.9. My brother and I bought _____ some apples. Q.5. There is none (A)/ who can (B)/ support you (C)/ in
Q.10. She cooks for _____ every day. this crucial period (D).
EXERCISE 3 Q.6. Any of the (A)/ two photos which reflect (B)/ the
Fill in the blanks with ‘which, ‘who’, ‘whom’ and ‘whose’. natural beauty of Kashmir (C)/ is worth seeing(D).
Q.1. The boy, _____ father is a doctor, is my best friend. Q.7. The guests (A)/ whom we were talking (B)/ about
Q.2. This is not something _____ we like to do. have arrived (C)/ are my relatives (D).
Q.3. That man, _____ left leg was amputated, suffers from Q.8. He introduced (A)/ to the chairman as (B)/ the
diabetes. president of the (C)/ workers association (D).
Q.4. The thief, _____ they caught, was sent to the prison.
Q.9. As a student (A)/ of arts (B)/ you are much better
Q.5. Our friends, _____ we invited to the party, arrived (C)/ than him (D).
rather early. Q.10. He hates everybody (A)/ and everything who (B)/
Q.6. The girl, _____ broke the mirror, was scolded by her reminds him (C)/ of his blunder (D).
mother.
ENGLISH LANGUAGE 19
stportal.mahendras.org

EXPLANATION
Q.6. The girl, who broke the mirror, was scolded by her
EXERCISE 1
mother.
Q.1. Does she know that I was absent?
Q.7. That is my uncle, whose car was stolen.
Q.2. Please tell him I have obtained a degree in Chemistry.
Q.8. That woman, whom you saw, was my auntie.
Q.3. I remember that they bought the fruits from us.
Q.9. Kangaroos, which use their pouch to carry their
Q.4. Please don’t tell her about me. babies, are found in Australia.
Q.5. It can swim because it has webbed feet. Q.10. The policeman, who caught the thief, is a very brave
Q.6. I met Jaya yesterday. She invited me to her house. man.
Q.7. Shreya has a cat; she likes to play with it. EXERCISE 4
Q.8. When the dog chased Jayant, he ran as fast as he Q.1. All that she said was incorrect.
could. Q.2. Who was the girl that came to meet you?
Q.9. My uncle works in a factory. He says it is a noisy Q.3. What is his opinion about the work that you have
place. completed?
Q.10. The teacher said to the class, “When you finish your Q.4. I always purchase the item that is the best in the
work, please pass it up to me.” market.
EXERCISE 2 Q.5. This is the same shirt that I saw in the shop.
Q.1. He rewarded himself with an ice-cream. Q.6. Who is she that can abuse me like this?
Q.2. They agreed amongst themselves that they would not Q.7. Only those employee should be promoted who are
tell anyone. sincere.
Q.3. I taught myself to draw. Q.8. The leaders who do not love their country are not
Q.4. You must explain yourself more clearly. respected.
Q.5. He has a bad temper so he must learn to control Q.9. Those who live in glass houses should not throw
himself. stones at others.
Q.6. We thoroughly enjoyed ourselves at the party on Q.10. What was the reply that she submitted?
Sunday. EXERCISE 5
Q.7. That monkey is scratching itself. Q.1. Replace ‘itself by ‘themselves’.
Q.8. One must defend oneself against bullies. Q.2. Replace ‘their’ by ‘his’ .
Q.9. My brother and I bought ourselves some ap Q.3. Replace ‘themselves’ by ‘himself’.
ples.
Q.4. Replace ‘lazy one’ by ‘lazy ones’. Ones is used for
Q.10. She cooks for herself every day. plurals.
EXERCISE 3 Q.5. Replace ‘who’ by ‘that’. For example : All that glitters
Q.1. The boy, whose father is a doctor, is my best friend. is not gold. He is the same man that supported me.
Q.2. This is not something which we like to do. Q.6. Replace ‘any’ by ‘either’. Either is used for two only.
Q.3. That man, whose left leg was amputated, suffers from Q.7. Replace ‘whom’ by ‘who’.
diabetes. Q.8. Place 'himself' after 'introduced'.
Q.4. The thief, whom they caught, was sent to the prison. Q.9. Replace ‘him’ by ‘he’.
Q.5. Our friends, whom we invited to the party, arrived Q.10. Replace ‘who’ by ‘that’.
rather early.

20 ENGLISH LANGUAGE
stportal.mahendras.org

CHAPTER

4 VERBS
Scan the QR code to get video of this chapter.

A verb is a word or group of words that expresses an action Example sentences (the auxiliary verb is bold, and the main
or a state. verb is underlined):
The word "verb" comes for the Latin word verbum, which They are jogging. She was sitting.
We were waiting for hours. Is she sleeping?
means "work."
He didn't know the answer.
Examples:
We have gone a long way.
Go, jump, sleep, eat, think, be, change, become, drive, Has she received any of my letters?
complete. Do you smoke? Will she help?
Example sentences: Compound Verbs
We had a nice lunch. I think that he is right. A compound verb = auxiliary verb + main verb.
He drove for hours. Phrasal Verbs
A phrasal verb is a verb that is combined with an adverb or a
preposition. The combination creates a new meaning.
Examples:
Run = to move very quickly with your legs. ("She can run
fast!")
Into = in the direction of something. ("He looked into my
eyes.")
Auxiliary Verbs (also called "helping verbs") Run into = to meet someone by accident. (I ran into Juhi
Auxiliary verbs are verbs that are used together with the main yesterday.")
verb of the sentence to express the action or state. Make = to create or do something. (He made a lot of noise.)
Main verb + auxiliary verb = complete idea Up = to a higher point. ("Look up!")
The main auxiliary verbs are: be, am, is, are, was, were, do, Make up = invent (a story, an excuse). ("It has never h a p -
did, have, has, had. pened. He made the whole thing up!")
Infinitive (base) Present tense Past tense Present Past Participle
form Iwith 3rd person Participle
singular)
to park park, parks (-s) Parked (ed) Parking (ing) Parked (-ed)
to sing sing, sings (-s) sang singing (-ing) sung
to carry carry, carries (-ies ) Carried (-ed)
carrying (ing) carried (-ied)
to write write, writes (-s) wrote writing (drop e before - written
ing)
to take take, takes (-s) took taking taken
(drop e before - ing)
to be am, is (singular), are was (3rd person being (-ing) been
(plural) were
to set set, sets(-s) set setting (double T, add- set
ing)
to think think, thinks (-s) thought thinking (-ing) thought
to drink drink, drinks (-s) drank drinking (-ing) Drunk
to have have, has had having (drop e before-ing) Had

ENGLISH LANGUAGE 21
stportal.mahendras.org

Modal Example Uses


Can They can control their own budgets. Ability / Possibility
We can’t fix it. Inability / Impossibility
Can I smoke here? Asking for permission
Can you help me? Request
Could Could I borrow your dictionary? Asking for permission
Could you say it again more slowly? Request
We could try to fix it ourselves. Suggestion
I think we could have another Gulf War. Future possibility
He gave up his old job so he could work for us. ability in the past
May May I have another cup of coffee? Asking for permission

China may become a major economic power. Future possibility


Might We'd better phone tomorrow, they might be Present possibility
eating their dinner now.
They might give us a 10% discount. Future possibility
Must We must say good-bye now. Necessity / Obligation
They mustn’t disrupt the work more Prohibition
than necessary.
Ought to We ought to employ a professional writer. Saying what’s right or
correct
Shall Shall I help you with your luggage? Offer
Shall we finalise 2.30 then? Suggestion
Shall I do that or will you? Asking what to do
Should We should sort out this problem at once. Saying what’s right or correct
I think we should check everything again. Recommending action
Profits should increase next year. Uncertain prediction
Will I can’t see any taxis so I’ll walk. Instant decisions
I'll do that for you if you like. Offer
I’ll get back to you first thing on Monday. Promise
Profits will increase next year. Certain prediction
Would Would you mind if I bring a colleague with me? Asking for permission
Would you pass the salt please? Request
Would you mind waiting a moment? Request
"Would three o`clock suit you?" - "That’d be fine." Making arrangements
Would you like to play golf this Friday? Invitation
"Would you prefer tea or coffee?" Preferences
- "I’d like tea please."
22 ENGLISH LANGUAGE
stportal.mahendras.org
Rule 4.
Modal Verbs
When a singular subject is connected by 'or' or 'nor' to a plural
What does MODAL mean?
subject, put the plural subject last and use a plural verb.
modal = expressing mood.
Example:
mood = a way to express the attitude of the speaker to what
The serving bowl or the plates go on that shelf.
is being said.
Rule 5.
Examples:
When a singular and plural subject are connected by either/or
"I can paint" means the speaker believes he has the ability
or neither/nor, put the plural subject last and use a plural verb.
to paint.
Example:
"I might paint" means the speaker believes there is a possibility
for that to happen. Neither Tina nor the others are available.
"I will paint" means the speaker has the intention to paint. Rule 6.
English Modal Verbs show us the attitude of the speaker to what As a general rule, use a plural verb with two or more subjects
is being said or done. when they are connected by and.
Subject Verb Agreement Example:
Basic Rule. A car and a bike are my means of transportation.
The basic rule states that a singular subject takes a singular Rule 7.
verb, while a plural subject takes a plural verb. Sometimes the subject is separated from the verb by words
NOTE: The trick is in knowing whether the subject is singular such as along with, as well as, besides, and not. Ignore these
or plural. The next trick is recognizing a singular or plural verb. expressions when determining whether to use a singular or
plural verb.
Hint: Verbs do not form their plurals by adding an 's' as nouns
do. In order to determine which verb is singular and which Examples:
one is plural, think of which verb you would use with he or The politician, along with the newsmen, is expected shortly.
she and which verb you would use with they. Excitement, as well as nervousness, is the cause of her
Example: shaking.
talks, talk Rule 8.
Which one is the singular form? Which word would you use The pronouns each, everyone, every one, everybody, anyone,
with he? We say, "He talks." Therefore, talks is singular. We anybody, someone, and somebody are singular and require
say, "They talk." Therefore, talk is plural. singular verbs. Examples:
Rule 1. Each of the girls sings well.
Two singular subjects connected by or require a singular verb. Each student was present in the class.
Example: Rule 9.
My aunt or my uncle is arriving by train today. The expression the number is followed by a singular verb
Rule 2. while the expression a number is followed by a plural verb.
Two subjects connected by either, or; neither, nor require Examples:
the verb according to the second subject. The number of people we need to hire is thirteen. A number
Examples: of people have written about this subject.
Neither Ajay nor Aman is available. Rule 10.
Either Karishma or her friends are helping today with stage When either and neither are subjects, they always take
decorations. singular verbs.
Rule 3. Examples:
When I is one of the two subjects connected by either/or or Neither of them is available to speak right now.
neither/nor, put it second and follow it with the singular verb Either of us is capable of doing the job.
am/or a verb that is suitable with I. Rule 11.
Example: The words here and there have generally been labelled
Neither she nor I am going to the festival. as adverbs even though they indicate place. In sentences

ENGLISH LANGUAGE 23
stportal.mahendras.org
beginning with here or there, the subject follows the verb. Five years is the maximum imprisonment for that offence.
Examples: Rule 13.
There are four hurdles to jump. Sometimes the pronoun who, that, or which is the subject
There is a high hurdle to jump. of a verb in the middle of the sentence. The pronouns who,
that, and which become singular or plural according to the
Rule 12. noun directly before them. So, if that noun is singular, use a
Use singular verb with sums of money or periods of time singular verb. If it is plural, use a plural verb.
when these are taken as/compared to a single unit/amount Examples:
Examples: Salma is the scientist who writes the reports.
Ten dollars is a high price to pay. The word before who is scientist, which is singular.
Therefore, use the singular verb 'writes'.
Forms of Some Regular Verbs
Base Form Past Form Past s / es/ ies ‘ing’ form
Participle
Form
abash abashed abashed abashes abashing
abate abated abated abates abating
abide abode abode abides abiding
bang banged banged bangs banging
banish banished banished banishes banishing
bash bashed bashed bashes bashing
call called called calls calling
canvass canvassed canvassed canvasses canvassing
capture captured captured captures capturing
damage damaged damaged damages damaging
damp damped damped damps damping
dance danced danced dances dancing
earn earned earned earns earning
eat ate eaten eats eating
educate educated educated educates educating
face faced faced faces facing
fail failed failed fails failing
faint fainted fainted faints fainting
gag gagged gagged gags gagging
gain gained gained gains gaining
gainsay gainsaid gainsaid gainsays gainsaying
handle handled handled handles handling
hang hung hung hangs hanging
happen happened happened happens happening
illuminate illuminated illuminated illuminates illuminating
illumine illumined illumined illumines illumining
illustrate illustrated illustrated illustrates illustrating
join joined joined joins joining
jump jumped jumped jumps jumping
justify justified justified justifies justifying
keep kept kept keeps keeping
24 ENGLISH LANGUAGE
stportal.mahendras.org

kick kicked kicked kicks kicking


kid kidded kidded kids kidding
lade laded laden lades lading
land landed landed lands landing
last lasted lasted lasts lasting
marry married married marries marrying
mash mashed mashed mashes mashing
match matched matched matches matching
nail nailed nailed nails nailing
nap napped napped naps napping
need needed needed needs needing
obey obeyed obeyed obeys obeying
oblige obliged obliged obliges obliging
observe observed observed observes observing
part parted parted parts parting
partake partook partaken partakes partaking
participate participated participated participates participating
qualify qualified qualified qualifies qualifying
quarrel quarrelled quarrelled quarrels quarrelling
question questioned questioned questions questioning
race raced raced races racing
rain rained rained rains raining
rattle rattled rattled rattles rattling
sabotage sabotaged sabotaged sabotages sabotaging
sack sacked sacked sacks sacking
sacrifice sacrificed sacrificed sacrifices sacrificing
tee teed teed tees teeing
tell told told tells telling
tempt tempted tempted tempts tempting
undo undid undone undoes undoing
uproot uprooted uprooted uproots uprooting
upset upset upset upsets upsetting
value valued valued values valuing
vanish vanished vanished vanishes vanishing
vary varied varied varies varying
weep wept wept weeps weeping
weigh weighed weighed weighs weighing
welcome welcomed welcomed welcomes welcoming
yawn yawned yawned yawns yawning
yell yelled yelled yells yelling
yield yielded yielded yields yielding

ENGLISH LANGUAGE 25
stportal.mahendras.org

exercise
EXERCISE 1 EXERCISE 3
Complete the following sentences with the correct form of Each of the sentences below has a mistake. Find it and
the verb in brackets: correct it.
(example : I play or I am playing) (Example : Did you ever visit Canada? Have you ever visited
Q.1. Every day Rashi _______________ (take) the bus to Canada?)
go to her office. Q.1. I live in the house I have bought 7 years ago.
Q.2. At the moment you _______________ (do) an English Q.2. The man who lives next door wrote four books about
exercise. his travels.
Q.3. Rahul and Priya _______________ (learn) English Q.3. I have been listening to the recording several times
this year. today.
Q.4. The Bank __________________ (open) at 9.30 every Q.4. Did you ever taste a cereal called “quinoa”?
morning from Monday to Friday. Q.5. How many new words did you note down so far in
Q.5. Our cousins ___________________ (come) to see us this lesson?
next Sunday. Q.6. Shimona has been visiting Hong Kong five times
Q.6. Rashi______________ (read) the newspaper every already.
morning on the train.
Q.7. I’ve seen that programme on television last night
Q.7. Sanya usually __________________ (clean) the
house on Saturdays. Q.8. How many times have you been asking for an
Q.8. At the moment she _________________ (write) a explanation?
letter to a client. Q.9. My English improved a lot this year.
Q.9. Aditi _________ (speak) three languages : English, Q.10. In her previous job, Rita has spoken English a lot.
French and Spanish. EXERCISE 4
Q.10. Today is Sunday. Ankush and Aayush ______________ Complete the following sentences with the appropriate form
of the verb in brackets :
(relax) in their garden.
The future simple (ex : I will watch) future continuous (ex
EXERCISE 2 : I will be watching)
Each of the sentences below has one or two mistakes. Find Q.1. I promise I ______________ (call) you as soon as I
them and correct them. have any news.
(Example : I am watching television since I came home Q.2. This time tomorrow Ronit (fly) over the Atlantic on
from school. I have been watching television since I came
home.) his way to Boston.
Q.1. How long are you learning English ? Q.3. Those bags look heavy. I __________
Q.2. How many cigarettes have you been smoking this ______________ (carry) one of them for you.
afternoon ? Q.4. They are getting married on Saturday. All the guests
Q.3. I am working here since 1995. _________________ (wear) white.
Q.4. I’ve had my watch since a long time. Q.5. The following week they ___________
Q.5. How many years are you living in London ? _____________ (enjoy) the sun in the West Indies.
Q.6. Mohan is exhausted. He is playing tennis since two Q.6. The sky is a bit cloudy. I think, it __________________
hours. (rain) what do you think?
Q.7. Sonal is doing very well at school since the beginning Q.7. If you look at this map you ____________ (see) where
of the year. the islands are.
Q.8. I have worn this sweater since 9 o’clock this morning. Q.8. You should have no problem finding him. He
I know Lalita for many years. ___________________ (carry) a guitar.
Q.9. Pawan is attending English classes since two years. Q.9. They___________________ (bring) many changes
Q.10. My best friend has always been hating fish. so that the new amendment comes into force.
26 ENGLISH LANGUAGE
stportal.mahendras.org
Q.10. He______________(be) appointed principal secretary Q.5. Saqib was delighted (meet) ___________ a former
to the Cheif Minister, next week. colleague at the conference.
EXERCISE 5 EXERCISE 7
Complete the following sentences with one of the following
In the following sentence, five parts i.e. (a), (b), (c), (d) and
words :
(e) are given, there can be errors in any part of the sentence,
can | could | may | might | should | ought to | shall | you will have to choose the incorrect part of the sentence and
will | would | must mark your answer accordingly.
Q.1. The article stated that he ___________ read at the Q.1. During the preparation of examinations (a) / many a
age of three.
candidate (b) / were supposed to (c)/study tooth and
Q.2. You ____________ take a sweater in case it gets cold. nail (d) / No error (e)
Q.3. It's going to rain all afternoon. ___________ we go Q.2. There are a dozen (a) / of Historical novels kept in
to the cinema? the almirah (b)/ of my bedroom (c) / and you can take
Q.4. Subodh ________ be 21 at the end of the month. them whenever you want (d) / No error (e)
Q.5. Jyoti said that the traffic was heavy, so she Q.3. The number of amendments to our constitution (a) /
___________ be late for the meeting. have been very large (b) / during the past (c)/50 years
Q.6. You look a bit confused. ________ I help you? of independence (d) / No error (e)
Q.7. You stole a CD from a friend? You _____________ Q.4. No one in this world (a) / can be able to (b)/do work
be ashamed of yourself! continuously(c)/ for ten hours (d) / No error (e)
Q.8. I don't know the meaning of this word. _______ I Q.5. A young girl student (a)/disappeared and found (b)/
borrow your dictionary? dead near a canal (c)/outside the city (d) / No error (e)
Q.9. I_________like you to go visit your aunt. Q.6. My childhood friend (a)/had been to Mumbai (b)/
Q.10. Jaya_________participate in the competition. last year to visit his (c)/parents (d) / No error (e)

EXERCISE 6 Q.7. Either you (a)/or the professor is to be blamed (b) /


for the mismanagement (c)/of the annual function (d)
Fill in the blanks below with the correct form of the verb
/ No error (e)
in brackets.
Q.8. It were the naughty street boys (a) / that caused a lot of
Q.1. It’s obvious he’s only interested in (make)
______________ money. chaos(b)/ to their parents (c) / during long bus journey
to school picnic (d) / No error (e)
Q.2. Anita couldn’t find a taxi so I offered (drive)
________________ her to the station. Q.9. They will be likely (a) / to leave for the new (b)/
country last year to visit(c)/ his brother in law (d) /
Q.3. I managed (book) ________________ two seats on
No error (e)
the morning flight
Q.10. The soldiers along with the commander (a) / was
Q.4. I promise (send) _______________ you our new court-martialled (b) / for defying the orders (c) / No
brochure as soon as it’s available. error (d)

ENGLISH LANGUAGE 27
stportal.mahendras.org

EXPLANATION
EXERCISE 1 EXERCISE 4
Q.1. takes Q.1. will call
Q.2. are doing Q.2. will be flying
Q.3. are learning Q.3. will carry
Q.4. opens Q.4. will be wearing
Q.5. are coming Q.5. will be enjoying
Q.6. reads Q.6. it will rain
Q.7. cleans Q.7. will see
Q.8. is writing Q.8. will be carrying
Q.9. speaks Q.9. will bring
Q.10. are relaxing Q.10. will be
EXERCISE 2 EXERCISE 5
Q.1. How long have you been learning English ? Q.1. could
Q.2. How many cigarettes have you smoked this after- Q.2. should/ought to
noon ? Q.3. shall
Q.3. I have been working here since 1995. Q.4. will
Q.4. I’ve had my watch for a long time. Q.5. might
Q.5. For how many years have you been living in London? Q.6. can
Q.6. Mohan is exhausted. He has been playing tennis Q.7. ought to/should
for two hours.
Q.8. may
Q.7. Sonal has been doing very well at school since the
beginning of the year. Q.9. would
Q.8. I have been wearing this sweater since 9 o’clock Q.10. must
this morning. I have known Lalita for many years. EXERCISE 6
Q.9. Pawan has been attending English classes for two Q.1. making
years.
Q.2. to drive
Q.10. My best friend has always hated fish.
Q.3. to book
EXERCISE 3 Q.4. to send
Q.1. I live in the house I bought 7 years ago.
Q.5. to meet
Q.2. The man who lives next door has written four
books about his travels. EXERCISE 7
Q.3. I have listened to the recording several times today. Q.1.(c) Replace ‘were’ by ‘was’

Q.4. Have you ever tasted a cereal called “quinoa”? Q.2.(a) Replace ‘are’ by ‘is’

Q.5. How many new words have you noted down so Q.3.(b) Replace ‘have’ by ‘has’
far in this lesson? Q.4.(b) Replace ‘can be’ by ‘is
Q.6. Shimona has visited Hong Kong five times already. Q.5.(b) Add ‘was’ after ‘and’
Q.7. I saw that programme on television last night Q.6.(b) Replace ‘had been’ by ‘went’
Q.8. How many times have you asked for an explana- Q.7.(e) No error
tion? Q.8.(a) Replace ‘were’ by ‘was’
Q.9. My English has improved a lot this year. Q.9.(a) Replace ‘will be’ by ‘were’
Q.10. In her previous job, Rita spoke English a lot. Q.10.(b) Replace ‘was’ by ‘were’

28 ENGLISH LANGUAGE
stportal.mahendras.org

CHAPTER

5 conjunction
Scan the QR code to get video of this chapter.

A conjunction is a word that connects words or group of no sooner ... than No sooner had I reached
words. the corner than the bus came.
Kinds of Conjunctions :- not only ... but also She is not only clever but also
Co-ordinating Conjunctions hard-working.
Co-ordinating conjunctions are conjunctions which connect rather ... than I would rather go swimming
two equal parts of a sentence .The most common ones are than go to the library.
and, or, but, and so these are used in the following ways. scarcely ... when Scarcely had we left home
when it started to rain.
And is used to join or add words together in a sentence.
whether ... or I am going, whether she likes
They ate and drank. it or not.
Or is used to show choice or possibilities in a sentence. Subordinating conjunctions
He will be here on Monday or Tuesday. Subordinating conjunctions connect two groups of words by
But is used to show opposite or conflicting things in a making one into a subordinating clause. The subordinating
sentence. clause acts as one huge adverb, answering the questions
She is small but strong. “when" or “why" about the main clause, or imposing
conditions or opposition on it.
So is used to show result in a sentence.
Here are some examples of subord nating conjunctions
I was tired so I went to sleep. changing a clause into adverbial subordinating clauses in
Correlative conjunctions different ways:
Correlative conjunctions are used in pairs, in order to show I can go shopping after I finish studying for my exam.
the relationship between the ideas expressed in different parts (when)
of a sentence. Because the night was young, Gertrude decided to take a
For instance, in the following example, the expression either walk. (why)
... or is used to indicate that the ideas expressed in the two I’ll give you a dime if you give me a dollar. (condition)
clauses represent two alternative choices of action.
Although he never figured out why, Hanna winked on her
e.g. You should either study harder or take a different course. way out the door. (opposition)
The most commonly used correlative conjunctions are both Note: The subordinating conjunction does not always come
... and, either ... or and neither ... nor. In the table below, between the two clauses it connects. Often, it comes at the
each pair of correlative conjunctions is accompanied by an beginning of the first clause.
example of its use. Note in the construction if ... then, the
word then can usually be omitted. Subordinating Conjunctions
As 1. because: As he is my friend, I will
Correlative Conjunctions help him.
both ... and He is both intelligent and good- 2. when: We watched as the plane took off.
natured.
After 1. later in time: After the train had left, we
either ... or I will either go for a walk or read
went home.
a book.
neither ... nor He is neither rich nor famous. Although or though
hardly ... when He had hardly begun to work 1. in spite of the fact that: Although it was
when he was interrupted. after midnight, we did not feel tired.
if ... then If that is true then what Before 1. earlier than: I arrived before the
happened is not surprising. stores were open.

ENGLISH LANGUAGE 29
stportal.mahendras.org
Because 1. for the reason that: We had to Whether 1. if: I do not know whether she was
wait, because we arrived early. invited.

For 1. for, because: He is happy, for he While 1. at the time when: While it was
enjoys his work. snowing, we played cards.

If 1. on condition that: If she is here, we 2. on the other hand: He is rich, while


his friend is poor.
will see her.
3. although: While I am not an expert,
Lest 1. for fear that: work hard, lest I will do my best.
you should fail. In addition, the following phrases are often used at the
Note: beginning of subordinate clauses.
the use of the Subjunctive Mood in the clause with lest. As if 1. in a similar way: She talks as if
she knows everything.
Provided1. on condition that: All will be well,
provided you are careful. As long as
1. if: As long as we cooperate, we can
Since 1. from a past time: I have been here
finish the work easily.
since the sun rose.
2. while: He has lived there as long as
2. as, because: Since you are here,
I have known him.
you can help me.
As soon as
So or so that
1. consequently: It was raining, so we 1. immediately when: Write to me as
did not go out. soon as you can.
2. in order that: I am saving money so that As though
I can buy a bicycle. 1. in a similar way: It looks as though
Note: When used with the meaning in order that, so is there will be a storm.
usually followed by that in formal English.
Even if 1. in spite of a possibility: I am going
e.g. I am saving money so that I can buy a bicycle. out even if it rains.
Suppose 1. if: Suppose that happens, what will In case 1. because of a possibility: Take a
you do? sweater in case it gets cold.
Than 1. used in comparisons: He is taller Or else 1. otherwise: Please be careful, or else
than you are. you may have an accident.
Unless 1. except when, if not: Unless he helps So as to 1. in order to: I hurried so as to be on
us, we cannot succeed.
time.
Until or till 1. up to the time when: I will wait until
I hear from you. Certain words, such as after, before, since and until may
function either as prepositions or subordinate conjunctions.
Whereas 1. because: Whereas this is a public However it should be noted that in some cases different words
building, it is open to everyone. must be used as prepositions and subordinate conjunctions, in
2. on the other hand: He is short, order to express similar meanings.
whereas you are tall.

30 ENGLISH LANGUAGE
stportal.mahendras.org

exercise
Q.5. It was not only a beautiful day, _________ the first
EXERCISE 1
day of Spring.
Pay attention to the meanings of the sentences, and to the
Q.6. If you follow the instructions, __________ you
presence of inverted word order, fill in the blanks with the
should have no difficulty.
correct coordinate conjunctions choosing from the pairs given
in brackets. Q.7. He is neither proud _____ condescending.
For example: Q.8. What with one thing _____ another, it was very late
## I would like to come, ___ I do not have time. by the time we left the house.
(but, nor) Q.9. No sooner had I opened my eyes, _________ I
remembered where I was.
## I would like to come, but I do not have time.
Q.10. Scarcely had I heard the news, _______ my friend
## He has not written, ___ has he called me. (but, nor)
arrived.
## He has not written, nor has he called me.
Q.1. I opened the door _____ looked out. (and, yet) EXERCISE 3
Q.2. She was not in the back yard, _________ was she Fill in the blanks with the correct subordinate conjunctions or
upstairs. (or, nor) similar expressions choosing from the pairs given in brackets.
Q.3. The sun had set, _________ it was still light outside. For example:
(or, yet) I went for a walk _______ the sun was shining. (because,
Q.4. Do you know his address _________ telephone otherwise)
number? (but, or) I went for a walk because the sun was shining.
Q.5. He has not arrived yet, _________ have they. (and, Do you know _______ the stores are open today? (as if,
nor) whether)
Q.6. I read the book, _____ did not understand it. Do you know whether the stores are open today?
(but, or) Q.1. We recognized her at once, ________ we had not seen her
Q.7. We searched diligently, ________ found nothing. (or, for years. (although, in case)
yet)
Q.2. He kept reading ___________ he fell asleep. (for,
Q.8. I invited him ______ his friends. (and, but) until)
Q.9. I do not know whether he has seen the movie before Q.3. The moon will rise ____________ the sun sets. (as
________ not. (or, nor) soon as, than)
Q.10. I would rather wait here _________ risk missing the Q.4. It looks ____________ the train will be late. (while,
bus. (than, when) as though)
EXERCISE 2 Q.5. ____________ she got her degree, she became a
teacher. (After, Than)
Pay attention to the expressions used in the following
sentences, fill in the blanks with the words and, but also, nor, Q.6. We will not go skiing ____________ the weather is
or, than, then and when, as appropriate. good. (as if, unless)
For example: Q.7. ____________ he left, he made sure he had his keys
## We should either walk quickly __ take the bus. with him. (Before, For)
## We should either walk quickly or take the bus. Q.8. ____________ I told the truth, you would not believe
me. (Even if, So that)
## I had scarcely sat down ____ the telephone rang.
Q.9. They have known her ___________ she was a child.
## I had scarcely sat down when the telephone rang.
(until, since)
Q.1. I have both respect ______ admiration for them.
Q.10. I must leave now, ____________ I have a great deal
Q.2. Hardly had I finished reading over the problem, ___
of work to do. (as, than)
the answer leapt to my mind.
Q.3. It will rain either today ______ tomorrow. EXERCISE 4
Q.4. He could not decide whether to tell the truth Fill in the blanks with either the conjunction or the preposition
_________ keep silent. given in brackets.

ENGLISH LANGUAGE 31
stportal.mahendras.org
For example: Q.3. It was very misty. ___________, we could not
_______ my warning, they went ahead with their plan. get a clear view of the mountain. (Hence
(Although, Despite) ,However)

Despite my warning, they went ahead with their plan. Q.4. We had walked several miles. ________, we
did not feel tired. (Accordingly, Still)
________ the sun was shining, the water was cold. (Although,
Despite) Q.5. She is a talented actress. ____________, she is very
beautiful. (Moreover, Thus)
Although the sun was shining, the water was cold.
Q.6. We take the bus every day. __________, we are
The price of oranges is high, __________ frost damage. familiar with the bus route. (Nevertheless, Thus)
(because, because of)
Q.7. The child was sleepy. __________, we went home
The price of oranges is high, because of frost damage. early. (Otherwise, Therefore)
(because, because of)
Q.8. The food was delicious. _____________, the service
I went to see the play, _______ it had good reviews. (because,
because of) was excellent. (Likewise, Nevertheless)

I went to see the play, because it had good reviews. (because, Q.9. We looked everywhere._________, we could not
because of) find the keys. (However, Thus)
Q.1. We stayed up late, ____________ we were tired. Q.10. The book is long. _____, the vocabulary is difficult.
(although, despite) (Consequently, Furthermore)
Q.2. They went swimming, ____________ the
coldness of the water. (although, despite)
EXERCISE 6
In the following sentence, five parts i.e. (a), (b), (c), (d) and
Q.3. I enjoy the course, _______ the professor is a
(e) are given, there can be errors in any part of the sentence,
good teacher. (because, because of)
you will have to choose the incorrect part of the sentence and
Q.4. She looks _______ your sister. (as if, like) mark your answer accordingly.
Q.5. Please wait ___________ I make a phone call. Q.1. Although all the new employees(a)/ listen to the
(during, while) new administrator (b)/ but their actions (c) / prove
Q.6. Did you hear any noises ____________ the otherwise (d) / No error (e)
night? (during, while) Q.2. The newly appointed (a)/ C.M. treats the residents of
Q.7. It looked __________ we would not be able to (b)/ his state as (c) / they are slaves (d) / No error (e)
leave until the next day. (as if, like) Q.3. Many of the candidates (a)/ are doing their post
Q.8. We all felt tired ____________ the hot weather. graduation (b)/because they may (c)/get a bright
(because, because of) future (d) / No error (e)
Q.9. I read a book ___________ I was waiting. Q.4. Such was his (a)/ pronunciation as (b) / I could not
(during, while) (c) understand him (d) / No error (e)
Q.10. Her eyes shone. _______ stars. (as if, like) Q.5. While we were discussing (a)/environmental issues
(c)/the examiner asked (c) / that who (d) / we were
EXERCISE 5 No error (e)
Fill in the blanks with the correct connecting adverbs Q.6. The candidates of this (a)/batch are interested (b) / in
choosing from the pairs given in brackets. such books that are full of (c)/ various complicated
For example: questions.(d)/ No error (e)
Will you come with me? _________ I shall have to go alone. Q.7. Each member of the (a)/alliance agree to (b)/take such
(Also, Otherwise) action (c) / that it deems necessary (d) / No error (e)
Will you come with me? Otherwise I shall have to go alone. Q.8. They had hardly finished (a) / their meals (b)/that at
once (c) / they resumed their duty (d) / No error (e)
S he is kind. ____, she is rather forgetful. (However,
Consequently) Q.9. It is disappointing not only (1)/to the players and
spectators,(2)/ but to the organisers and advertisers
She is kind. However, she is rather forgetful. (3)/ who stand to lose financially. (4)/ No error (5).
Q.1. The work was new to me. ____________, it did Q.10. India’s banking sector (1)/ is stagnant (2)/ and faces
not seem difficult. (Consequently, Nevertheless) (3)/ neither increase or decrease in growth. (4)/ No
Q.2. Continue along Queen Street. ________ turn error (5)
left. (Then, Therefore)
32 ENGLISH LANGUAGE
stportal.mahendras.org

eXPLANATION
Q.3. because Q.4. like
EXERCISE 1
Q.5. while Q.6. during
Q.1. and Q.2. nor
Q.7. as if Q.8. because of
Q.3. yet Q.4. or
Q.9. while Q.10. like
Q.5. nor Q.6. but
Q.7. yet Q.8. and EXERCISE 5
Q.9. or Q.10. than Q.1. Nevertheless Q.2. Then
Q.3. Hence Q.4. Still
EXERCISE 2
Q.5. Moreover Q.6. Thus
Q.1. and Q.2. when
Q.7. Therefore Q.8. Likewise
Q.3. or Q.4. or
Q.9. However
Q.5. but also Q.6. no conjuncion required
Q.10. Furthermore (it is used as an adverb and means in
Q.7. Nor (condescending- means displaying a
addition or moreover)
patronizingly superior attitude).
Q.8. and Q.9. than EXERCISE 6
Q.10. when Q.1.(b) Replace ‘but’ by ‘yet’
Q.2.(d) No error
EXERCISE 3
Q.3.(b) Replace ‘because’ by ‘so that’
Q.1. although Q.2. until
Q.4.(b) Replace ‘as’ by ‘that’
Q.3. as soon as Q.4. as though
Q.5.(b) Delete ‘that’
Q.5. After Q.6. unless
Q.6.(c) Replace ‘that’ by ‘as’
Q.7. Before Q.8. Even if
Q.7.(d) Replace ‘that’ by ‘as’
Q.9. since Q.10. as
Q.8.(c) Replace ‘that’ by ‘when’
EXERCISE 4
Q.9.(c) No error
Q.1. although Q.2. despite
Q.10.(c) Replace 'or' by 'nor'

ENGLISH LANGUAGE 33
stportal.mahendras.org

CHAPTER

6 time and tense


Scan the QR code to get video of this chapter.

What does TENSE mean ?


A tense is a form of verb which shows the time at which an action happens.
It comes from the Latin word "tempus", which means "time".
Examples
"Sang" is a form of the verb "sing". This form shows the action happened in the
past. This is Past Tense.
"Sleeps" is a form of the verb "sleep". This form shows the
action that is happening in the present. This is Present Tense.
In fact, in English verbs and tenses are not much complicated. We will
go over them step by step so that you get a full understanding of them.

Tenses Simple Continuous Perfect Perfect continuous

Present Active I like I am liking I have liked I have been liking - - - -


Passive I am liked I am being I have been liked
liked
Past Active I liked I was liking I had liked I had been liking
Passive I was liked I was being I had been liked -----------
liked
Future Active I will like I will be liking I will have liked I will have been liking
Passive I will be ---------- I will have been -----------
liked liked

We will start with the Simple English Tenses.


Simple tense is a form of a verb that simply shows when the action takes place.
Simple Past is a form of the verb that shows the action took place in the past.
For example: Rima danced yesterday.
Simple Present Simple Past Simple Present Simple Past

Aim Aimed Bake Baked


Look Looked Talk Talked

Simple Present is a form of the verb that shows the action takes place in the present.
For example: Rima dances every day.
Simple Future is a form of the verb that shows the action will take place in the future.
For example:Rima will dance tomorrow.
Progressive (Continuous) Tenses
PROGRESSIVE means "continuing".
Progressive tense is a form of a verb that shows the action in progress or in other words, that it continues.

34 ENGLISH LANGUAGE
stportal.mahendras.org
Past Progressive is a form of the verb that shows the action For example: Rima will have danced tomorrow by 9 o'clock.
was in progress at some time earlier in the past. Perfect + Progressive
For example: Rima was dancing yesterday at 8 o'clock. Yes! There are 3 more
Present Progressive is a form of the verb that shows the ac- tenses. These are the
tion in progress in the present. Perfect Progressive
For example: Rima is dancing right now. (Continuous) tenses.

Future Progressive is a form of the verb that shows the action Perfect Progressive
will be in progress at some time in the future. tense is a form of the
verb that shows the
For example: Rima will be dancing tomorrow at 8 o'clock. action started, contin-
Perfect Tenses ued, and was complet-
ed until some point.
PERFECT means "com-
plete, finished". Past Perfect Progres-
sive is a form of the
Perfect tense is a form of
verb that shows the action started in the past and continued
a verb that shows the ac-
until some point in the past.
tion is complete. It does not
mean the action is "perfect" For example: Rima had been dancing for 2 hours before
(100%). It means the action she was tired.
is finished. Present Perfect Progressive/Continuous is a form of the verb
Past Perfect is a form of the that shows the action started in the past and continued until
verb that shows the action was completed before some time the present.
in the past. For example: Rima has been dancing for 3 hours without
For example: Rima had danced before she came. stopping!
Present Perfect is a form of the verb that shows the action Future Perfect Progressive is a form of the verb that shows
was completed in the present time. the action will continue until some point in the future.
For example: Rima has already danced. For example: By tomorrow morning, Rima will have been
dancing for 12 hours!
Future Perfect is a form of the verb that shows the action will
be completed before some time in the future.

exercise
Q.1. She __ the house when I __ to her after a long time. (b) Were doing/ were working
(a) Will be cleaning/ go (c) Have done/ have worked
(b) Had been cleaning/ went (d) Have been doing/ work
(c) Has been cleaning/ have gone (e) Is cleaning/ am going
(d) Is cleaning/ am going Q.4. Some time ago, an interesting discovery __ by ar-
(e) Has cleaned/ have been chaeologists on the Aegean Island of Kec
Q.2. While mother _ the VCD I hired, I __ my assigment. (a) Had to be made (b) Has been made
(a) Was watching/ finished (c) Was made (d) Is made
(b) Is watching/ I had finished (e) Used to be made
(c) Watches/ would finish Q.5. For the time being we _____ for the exam, but this
(d) Will watch/ am finishing time tomorrow we _____ the best movie of this year
(e) Was going to watch/ finish (a) Study/ will watch
Q.3. These days, people who _____ manual work, often (b) Are studying/ will be watching
receive far more money than clerks who _____ in (c) Have been studying/ are watching
offices. (d) Are studying/ will have watched
(a) Do/ work (e) Have studied/ will be watching
EXPLANATION
Q.1.(b) had been cleaning/ went Q.4.(c) Was made
Q.2.(a)Was watching/ finished Q.5.(b) are studying / will be watching
Q.3.(a) Do/ work
ENGLISH LANGUAGE 35
stportal.mahendras.org

CHAPTER
REARRANGEMENT
7 OF SENTENCES
Scan the QR code to get video of this chapter.

One of the most interesting but difficult units of objective test


Step-1
in English is sentence arrangement. In this test a candidate
is required to arrange parts of sentence or sentences into
a meaningful sentence or a paragraph. This requires a
command over the structure of sentences along with a sound
knowledge of grammar. This art can be gradually mastered
by reading good books, newspapers and constant practice.
You must read books very carefully giving full attention to
the structures of the sentences. Gradually you will begin to
feel that there are internal rules that bind a sentence because a
loose sentence expresses the main thought near the beginning
adds explanatory material as it progresses and compiles a
complete thought or an idea as it ends.
Examples:-
This question appears in a variety of different forms .
(A) Lakhs of students (B) on all India basis
(C) from all over India (D) which are held
(E) appear in the examination
(F) By different organizations.
This type of a question may be solved by Subject-Verb-
Object-Qualifiers Location.
NOTE: Step-1
A word or phrase that precedes an adjective or adverb, Let us analyse the following sentence in the graph below:
increases or decreases the quality signified by the word it
Sunil who is the son of Damodar reads hurriedly the book
modifies.
which has been written by William Shakespear.
Here are the most common qualifiers in English (though
some of these words have other functions as well: very,
quite, rather, somewhat, more, most, less, least, too, so, just,
enough, indeed, still, almost, fairly, really, pretty, even, a bit,
a little, a (whole) lot, a good deal, a great deal, kind of, sort
of.
Let us look at the above example:
1. Lakhs of students - (subject)
2. From all over India – (qualifier of the subject (Q)
3. Appear in the examination – (verb)
4. Which are held - (qualifier of the verb)
5. On an all India basis - (qualifier of which are held)
6. By different organizations – (the last part)

Thus the sequence is ACEDBF
Here, who is the son of Damodar, is the qualifier of Sunil,
What is a qualifier? the subject.
36 ENGLISH LANGUAGE
stportal.mahendras.org
commonly used transition words: also, again, as well as,
There Are Several Sentences Without Any
besides, furthermore, in addition, likewise, moreover,
Definite Beginning Or End:
similarly, consequently, hence, otherwise, subsequently,
(A) Miss Shital arrived at Kirti’s Home when Shilpa was therefore, thus, as a rule, generally, for instance, for
seven. example, for one thing, above all, aside from, barring,
(B) The deaf and blind Shilpa learned to communicate besides, in other words, in short, instead, likewise, on
verbally. one hand, on the other hand, rather, similarly, yet, but,
(C) But ,eventually Miss Shital’s effort was rewarded. however, still, nevertheless, first of all, to begin with, at
the same time, for now, for the time being, in time, later
(D) Before Shilpa Kirti was three years old, she lost her
sight and her hearing. on, meanwhile, next, then, soon, the meantime, later, while,
earlier, simultaneously, afterward, in conclusion, with this
(E) Miss Shital worked closely with Shilpa, her new student.
in mind, after all, all in all, to sum up.
(F) At times the teacher became frustrated.
Examples:-
Method (A) But in the industrial era destroying the enemy’s
The same method of arranging the paragraph first at the level productive capacity means bombing the factories
of idea and then at the level of sentences should be followed which are located in the cities.
1. First, find some clues. For instance ,in this paragraph, we
may find a clue in the reference to the time.The paragraph (B) So in the agrarian era, if you need to destroy the
is about Miss Shital and Shilpa Kirti. The first thing that enemy’s productive capacity, what you want to do
happened in relation to time was the fact that Shilpa lost is burn his fields, or if you’re really vicious, salt
her sight before she was three years old. Miss Shital arrived them.
when Shilpa was seven.The other incidents took place later. (C) Now in the information era, destroying the enemy’s
Thus it is logical that the first sentence should be ‘D’ - the productive capacity means destroying the information
sentence telling us that Shilpa lost her sight before she was
infrastructure.
three years old.After that we can arrange other sentences
according to the time sequence.So, the sequence is “D A E (D) How do you do battle with your enemy?
F C B”. (E) The idea is to destroy the enemy’s productive capacity,
Important Clues:- and depending upon the economic foundation, that
In any paragraph there would be: productive capacity is different in each case.
(A) A central theme around which the passage is framed. (F) With regard to defence, the purpose of the military
Identify the central theme, which is usually the first is to defend the nation and be prepared to do battle
sentence. with its enemy.
(B) A logical sequence is present which usually describes Answer:
the central theme.
Look at the transition word “but” in the first sentence. It
(C) Sentences that begin with ‘And’, ‘But’ etc usually follow signifies that the sentence is expressing an idea contrary to an
the central theme with an explanation to the same or give idea expressed in some previous sentence. Now we need to
additional information. find that previous sentence. If we further look at the beginning
Tips For Beginners of the first sentence, it says “but in the industrial era...” which
Focus on improving your reading skills. Also try to improve suggests that the contrariness is with respect to eras. Looking
your cognitive ability. For example — Go to a random further, we see that sentence B and C are also starting with
website article. Go immediately to the second paragraph statement about eras. But the transition word at the start of
and after reading it try to guess what the author could C is “now” which expresses present era and hence it cannot
have possibly said in the previous paragraph and the next chronologically come before any other past era. That is, if
paragraph. This will help you with a couple of other types information era is the present era, talk about any other era
of questions as well . will come before this. So sentence B is the correct sentence
Transition Words to come before the first sentence. Likewise, sentence C is the
correct sentence to come after the first sentence (sentence C
Transition words make the shift from one idea to another
is continuing the idea). Therefore, we have the link BAC.
very smoothly. They organize and connect the sentences
logically. Observing the transition words found in a sentence Furthermore, all the three options have the link EBAC.
can often give you a clue about the sentence that will come Therefore, we only need to arrange D and F. The sentence F
before/after that particular sentence. Given below are some states that “The purpose is...to battle with the enemy” and D

ENGLISH LANGUAGE 37
stportal.mahendras.org
questions “how do you battle with the enemy?” Therefore, In the sentence E, a noun identifies the saying as the second
D will come after F. sentence C covers the person who said this, then sentence
B gives the response of the student, then sentence A gives
Hence FDEBAC is the correct arrangement. the thinking of the student and then again the teacher in
NOTE: The flow of author’s ideas should be completely sentence D concludes by reconfirming in the sentence
logical. The author does not jump from one idea to another F.
suddenly. Example:-2
Personal Pronouns (A) However many people do not believe in it
Personal pronouns are he, she, it, him, her, they, you, your (B) Whereas those people who are corrupt
etc. Remember that personal pronouns always refer to a (C) But it doesn’t apply to them when it comes to applying
person, place or thing etc. Therefore, if a sentence contains in daily life
a personal pronoun without mentioning the person, place or
(D) Honesty is the best policy
object it is referring to, the person, place or object must have
come in the previous sentence. Often, this is a good lead to (E) For the reason that it takes more than it gives
identify a link. (F) Also believe in the same ideology
Demonstrative Pronouns Ans. [D, A, E, B, F, C]
The demonstrative pronouns are “this,” “that,” “these,” and Sentence D is the first because it introduces a concept,
“those.” “This” and “that” are used to refer to singular nouns sentence A gives its opposition while sentence E gives reason
or noun phrases and “these” and “those” are used to refer to why and sentence B gives its opposite believers with F as
plural nouns and noun phrases. Whenever a sentence contains their acceptance and C is the conclusion in the practical sense.
a demonstrative pronoun without mentioning the noun or Example:-3
the noun phrase, it means that the previous sentence must be
(A) It was about the concept of God.
mentioning that noun or noun phrase. Finding that noun or
noun phrase helps us connect two sentences. (B) Who never cared about the world he made with so much
love and affection.
Given below are some examples with illustrations-
(C) Recently, I saw a movie with my friends
Example:-1
(D) And his thinking about humans
(A) Is there such a thing called easy in life?
(E) And so he decided to put an end to this world he made.
(B) On hearing this one student questioned
Ans. [C, A, D, B, E]
(C) Said the teachers to the students.
Sentence C tells the starting point of the activity while
(D) On this the teacher concluded sentence A gives the idea of the story after A, sentence D tells
(E) “life is not a bed of roses” us the progression of sentence A, while sentence B gives an
inside story out of what god thinks and sentence E gives the
(F) No my dear nothing is easy final reaction of God.
Ans. [E, C, B, A, D, F]

38 ENGLISH LANGUAGE
stportal.mahendras.org

exercise
What will be the correct rearrangement of the parts of a Q.8. The competition is most acute in Bandipur and
sentence given below and in case you feel the given sequence Badlapur, (A)/ and Badlapur is surrounded by 100
is in its correct form then mark option (5) i.e no change old villages (B)/ which forms the core of the elephant-
required as your answer. cum-tiger reserve (C)/ while Bandipur is surrounded
EXERCISE-1 by 180 villages (D)

Q.1. The rates of death from heart disease and cancer (1) BACD (2) BDCA (3) DACB
(A)/ servings of whole grains appears (B)/ a diet that (4) ACDB (5) No change required
includes three daily (C)/ to help people live longer, Q.9. Rhesus macaque, and nilgai as vermin within
including by reducing (D) specified (A)/ from Uttarakhand, Himachal Pradesh
(1) BACD (2) BDCA (3) CBDA and Bihar to declare wild boar (B)/ since 2015, the
(4) DBCA (5) No change required Union Environment Ministry has acceded to requests
(C)/ territories of these states and outside forests and
Q.2. Devastating than last year’s (A)/ the Himalayan region protected areas.(D)
may (B)/ be under the risk of earthquakes more (C)/
Nepal that killed over 8,000 (D) (1) CBAD (2) BDCA (3) DACB

(1) BACD (2) BDCA (3) BACD (4) DBCA (5) No change required

(4) BCAD (5) No change required Q.10. As thousands prepared to (A)/ strikes closed the Eiffel
Tower (B)/ with the Euro 2018 games in full swing
Q.3. The metro cities which do (A)/ there are some schools (C)/ march against labour reforms (D)
in (B)/ not try to subject the (C)/ child to too much
pressure (D) (1) BACD (2) BDCA (3) DACB

(1) BDCA (2) BACD (3) DACB (4) BADC (5) No change required

(4) DBCA (5) No change required Q.11. During the hearing, the counsel said (A)/ in the
case and added that the bank’s (B)/ claims were
Q.4. By scrapping the five year domestic (A)/ liberalising completely against facts (C)/ his client was not a party
the right to fly abroad (B)/ real room for manoeuvre for or guarantor (D)
investors (C)/ flight operations requirement doesn’t
create (D) (1) BACD (2) ADBC (3) DACB

(1) BACD (2) BADC (3) DACB (4) DBCA (5) No change required

(4) DBCA (5) No change required Q.12. Concluded in the favour (A)/ we hope to do an
intensive (B)/ of both the countries (C)/ exercise so
Q.5. Announcement of the start of preparatory (A)/ that it can be (D)
reactors does mark a significant thaw in (B)/ work in
India for six Westinghouse nuclear (C)/ civil nuclear (1) BACD (2) BDCA (3) BDAC
matters after the deep freeze of many years. (D) (4) DBCA (5) No change required
(1) BACD (2) BDCA (3) DACB Q.13. The coming together of these two big private players
(4) DBCA (5) No change required (A)/ this will help them efficiently use the resources
Q.6. There are many challenges before (A)/ there is the both capital and policy holder base (B)/ the market
factor of migrant labour. (B)/ the arrest shows that share of the combined entity, for another (C)/ makes
(C)/ the police especially when (D) enormous business sense, for one it could improve
(D)
(1) BACD (2) BDCA (3) CADB
(1) BACD (2) BDCA (3) ADCB
(4) DBCA (5) No change required
(4) DBCA (5) No change required
Q.7. To relieve teachers and particularly children (A)/ from
facing undue stress and strain (B)/ the continuous Q.14. The current norms could be amended based on public
and comprehensive (C)/ evaluation system must be feedback (A)/ the board of SEBI met in Mumbai and
introduced (D) decided (B)/ for portfolio managers and REITs so that
(C)/ to release consultation papers on regulation (D)
(1) BACD (2) CDAB (3) DACB
(1) BACD (2) ACBD (3) DACB
(4) DBCA (5) No change required
(4) BDCA (5) No change required

ENGLISH LANGUAGE 39
stportal.mahendras.org
Q.15. Insurance market and the rest 19 private insurers (A)/ (E) for success failure of the committee.
constitute 65 per cent of the private (B)/ the top four Q.1. Which of the following should be the FIFTH sentence
private insurers (C)/ have a combined share of 35 per after the rearrangement?
cent (D)
(1) C (2) D (3) A
(1) BACD (2) BDCA (3) CBAD
(4) B (5) E
(4) DBCA (5) No change required
Q.2. Which of the following should be the SECOND
Q.16. Rs. 5,000 a candy, in the last few (A)/ the steep hike sentence after the rearrangement?
in cotton prices, by about (B)/ for which cotton is the
main raw material (C)/ weeks is a matter of concern (1) B (2) C (3) D
to textile industry (D) (4) A (5) E
(1) ACBD (2) BDCA (3) BADC Q.3. Which of the following should be the THIRD sentence
(4) DBCA (5) No change required after the rearrangement?
Q.17. Contrary to advertisements, most so called ice creams (1) D (2) E (3) C
(A)/ it a rich creamy texture, and a characteristic (4) B (5) A
flavour (B)/ milk and other dairy products. The milk
Q.4. Which of the following should be the FIRST sentence
fat in dairy gives (C)/ are actually frozen desserts,
after the rearrangement?
made with vegetable oil (D)
(1) C (2) E (3) B
(1) BACD (2) ADCB (3) DACB
(4) A (5) D
(4) DBCA (5) No change required
Q.5. Which of the following should be the FOURTH
Q.18. A recent study at the European (A)/ the risks of stroke
sentence after the rearrangement?
and heart diseases (B)/ Norfolk found that antioxidants
reduce (C)/ Prospective Investigation of Cancer(D) (1) A (2) D (3) C
(1) BACD (2) BDCA (3) DACB (4) E (5) B
(4) ADCB (5) No change required EXERCISE-3
Q.19. Ability on par with that of primates (A)/ birds have (A) Normally it is very small but when the one of the
more densely packed (B)/ animals, contributing to bodies is a planet, like earth, the force is considerable.
cognitive (C)/ neurons in their brains than other (D) (B) It has been investigated by many scientists including
(1) BACD (2) BDCA (3) DACB Galileo and Newton.
(4) DBCA (5) No change required (C) A force exists between everybody in the universe.
Q.20. The reasonable restrictions under Article 19(2) have (D) The greater the mass, the greater is the earth’s force
(A)/ such things like offensive grain of social or of attraction on it. We can call this force of attraction
political opinions (B)/ these restrictions were never gravity.
meant to include (C)/ been routinely invoked to choke (E) Everything on or near the surface of the Earth is
free speech and expression (D) attracted by the mass of earth.
(1) BACD (2) BDCA (3) ADCB (F) This gravitational force depends on the mass of the
(4) DBCA (5) No change required bodies involved.
Directions: Sentences given in each question, when Q.1. Which of the following should be the FIRST sentence
properly sequenced, form a coherent paragraph. Each after the rearrangement?
sentence is labelled with a letter. Choose the most (1) F (2) E (3) C
logical order of sentences from among the given
(4) D (5) B
choices to construct a coherent paragraph,
Q.2. Which of the following should be the SECOND
EXERCISE-2
sentence after the rearrangement?
(A) The most important person in the room
(1) A (2) C (3) D
(B) on him rests much of the resonsibility
(4) B (5) E
(C) In the middle of one side of the square sits the
Q.3. Which of the following should be the THIRD sentence
chairman of the committee.
after the rearrangement?
(D) As the meeting opens he runs briskly through a number
(1) F (2) A (3) B
of formalities.
(4) C (5) D
40 ENGLISH LANGUAGE
stportal.mahendras.org
Q.4. Which of the following should be the FOURTH
EXERCISE-5
sentence after the rearrangement?
(A) The study of speech disorders due to brain injury
(1) F (2) D (3) C
suggests that patients can think without having
(4) B (5) A adequate control over their language.
Q.5. Which of the following should be the SIXTH sentence (B) They can even use the concepts needed for chess
after the rearrangement? playing, though they are unable to express many of
(1) D (2) C (3) A the concepts in ordinary language.
(C) Some patients, for example fail to find the names of
(4) B (5) F
objects presented to them.
EXERCISE-4 (D) How they manage to do this we do not know.
(A) The philosopher Descartes thought that both men and (E) But they succeed in playing games of chess.
animals were machines.
Q.1. Which of the following should be the FIRST sentence
(B) Therefore, some scientists think that life is just a very after the rearrangement?
complicated mechanism. (1) B (2) C (3) A
(C) But a machine such as a motorcar or a steamship (4) F (5) E
moves itself, as soon as machines which moved
Q.2. Which of the following should be the SECOND
themselves had been made, people asked “Is man a
sentence after the rearrangement?
machine?”
(1) B (2) C (3) A
(D) And before the days of machinery that was a good
(4) D (5) E
definition.
Q.3. Which of the following should be the THIRD sentence
(E) Our ancestors thought that anything which moved after the rearrangement?
itself was alive.
(1) E (2) A (3) B
(F) He also thought that the human machine was partly
(4) D (5) C
controlled by the soul action on a certain part of the
brain, while animals had no souls. Q.4. Which of the following should be the FOURTH
sentence after the rearrangement?
Q.1. Which of the following should be the SECOND
sentence after the rearrangement? (1) A (2) B (3) C
(4) D (5) E
(1) F (2) C (3) A
Q.5. Which of the following should be the FIFTH sentence
(4) D (5) E after the rearrangement?
Q.2. Which of the following should be the FOURTH (1) A (2) E (3) B
sentence after the rearrangement? (4) C (5) D
(1) C (2) B (3) F
EXERCISE-6
(4) A (5) E Rearrange the following sentences (A),(B),(C),
Q.3. Which of the following should be the FIRST sentence (D) and (E) in the proper sequence to form a
after the rearrangement? meaningful paragraph, then answer the questions
given below.
(1) B (2) E (3) C
(A) The FAO has said two-thirds of the country’s vast
(4) F (5) D wheat crop.
Q.4. Which of the following should be the THIRD sentence (B) The Food and Agriculture Organization’s diagnosis
after the rearrangement? of China’s wheat.
(1) A (2) E (3) C (C) Is at risk due to the winter drought.
(4) F (5) D (D) Crisis defers from the one.
(E) Presented by the Chinese government.
Q.5. Which of the following should be the FIFTH sentence
after the rearrangement? Q.1. Which of the following should be the FOURTH
sentence after the rearrangement?
(1) B (2) A (3) C
(1) B (2) A (3) C
(4) E (5) F
(4) D (5) E
ENGLISH LANGUAGE 41
stportal.mahendras.org
Q.2. Which of the following should be the SECOND (1) C (2) A (3) G
sentence after the rearrangement? (4) D (5) H
(1) A (2) C (3) E Q.5. Which of the following should be the FIRST sentence
(4) B (5) D after the rearrangement?
Q.3. Which of the following should be the THIRD sentence (1) A (2) B (3) D
after the rearrangement? (4) F (5) G
(1) C (2) B (3) D
EXERCISE-8
(4) E (5) A
If the (C) part of the sentence is fixed at its place
Q.4. Which of the following should be the FIRST sentence then what will be the correct rearrangement of
after the rearrangement? the sentence given below and in case you feel the
(1) B (2) E (3) D given sentence is in its correct sequence then mark
option (5) i.e no change required as your answer.
(4) A (5) C
Q.1. he is planning(A)/ with his first smartphone (B)/
Q.5. Which of the following should be the FIFTH sentence
and catch the hinterland consumer while he is still
after the rearrangement?
fiddling(C)/ to race ahead of e-retailers.(D)
(1) B (2) E (3) C
(1) BACD (2) BDCA (3) DACB
(4) D (5) A
(4) ADCB (5) No change required
EXERCISE-7 Q.2. people who earn more money tend to become
Rearrange the following sentences (A),(B),(C), selfish,(A)/ while those who earn less take greater
(D) and (E) in the proper sequence to form a pleasure(B)/ in their relationships and ability(C)/ to
meaningful paragraph, then answer the questions connect with others, according to a study.(D)
given below. (1) BACD (2) BDCA (3) DACB
(A) The interest subsidy it pays to banks. (4) DCAB (5) No change required
(B) Farm loans in the budget. Q.3. several states have reduced VAT,(A)/ consum-
(C) Had suggested that farm loans be made available at ers interest," the minister told reporters (B)/
four percent. they should reduce for(C)/ we will appeal again
States, which have high VAT component,.(D)
(D) The government plans to. (1) BACD (2) ADCB (3) DACB
(E) The working group on agricultural production. (4) BDCA (5) No change required
(F) Provide incentives on. Q.4. which can result in sudden and prolonged
(G) Will have to increase. crash exposing investors, (A)/ There is a real
and heightened risk of investment bubble of the
(H) This would mean that the government. type seen in Ponzi schemes (B)/ especially retail
Q.1. Which of the following should be the LAST sentence consumers(C)/ losing their hard-earned money.(D)
after the rearrangement? (1) BDCA (2) BACD (3) DACB
(1) G (2) A (3) D (4) ADCB (5) No change required

(4) E (5) B Q.5. losing their hard-earned money (A)/ There is a real
and heightened risk of investment (B)/ and pro-
Q.2. Which of the following should be the THIRD sentence longed crash exposing investors, especially retail
after the rearrangement? consumers(C)/ bubble of the type seen in Ponzi
(1) C (2) A (3) D schemes which can result in sudden.(D)

(4) E (5) B (1) BACD (2) DACB (3) ABCD


(4) BDCA (5) No change required
Q.3. Which of the following should be the SECOND
sentence after the rearrangement? Q.6. Patel, who started his entrepreneurial journey(A)/
in the late 1960s as a manufacturer of detergent,(B)/
(1) G (2) H (3) D
and cycling around towns and villages to sell it,(C)/
(4) F (5) C is known for creating low-priced products(D)
Q.4. Which of the following should be the FIFTH sentence (1) BDCA (2) ADCB (3) DACB
after the rearrangement? (4) DBCA (5) No change required
42 ENGLISH LANGUAGE
stportal.mahendras.org
Q.7. 2010 for the first time in 400 years, after another pe- Q.1.  Which of the following would be the LAST sentence
riod of inactivity (A)/ Mount Sinabung roared back after rearrangement?
to life in (B)/ it erupted once more in 2013,(C)/ and (1) D (2) E (3) F ]
has remained highly active since.(D)
(4) A (5) C
(1) BDCA (2) DACB (3) BACD
Q.2.  Which of the following would be the FIRST sentence
(4) ADCB (5) No change required after rearrangement?
Q.8. themselves to by investing in Bitcoin (A)/ by the Re- (1) B (2) F (3) C
serve Bank of India about the potential financial,(B)/
and security related risks that they are exposing(C)/ (4) D (5) A
users and traders of VCs have already been cautioned Q.3.  Which of the following would be the FOURTH
three times,.(D)  sentence after rearrangement?
(1) BDCA (2) DBCA (3) ADCB (1) A (2) B (3) C
(4) DACB (5) No change required (4) F (5) E
Q.9. he further said that the petroleum sector (A)/would be Q.4. Which of the following would be the FIFTH sentence
brought under (B)/the ambit of the Goods and Services after rearrangement?
tax (GST) (C)/as and when the consensus is reached
(1) A (2) C (3) D
among the states.(D)
(4) B (5) E
(1) ADCB (2) BDCA (3) DACB
Q.5.  Which of the following would be the THIRD sentence
(4) DBCA (5) No change required after rearrangement?
Q.10. and data become cheaper, the e-retailers wait (A)/ as (1) D (2) E (3) F
internet penetrates Bharat and phones (B)/ for Indians
(4) A (5) C
in the hinterland(C)/ to get online and buy stuff from
them.(D) EXERCISE-10
(1) ADCB (2) BDCA (3) DACB Q.1-5. Rearrange the following sentences (A),(B),(C),
(D) ,(E)and (F) in the proper sequence to form a
(4) BACD (5) No change required meaningful paragraph, then answer the questions
Q.11. most people think of money as some kind of unmiti- given below them.
gated good.(A)/ But some recent research suggests that (A)  But also makes us virtuous and happy.
this may not actually be the case.(B)/ In many ways,
(B)  Money can show us all the roads to happiness but not
money does not(C)/ necessarily buy you happiness.(D) happiness itself.
(1) BDCA (2) DACB (3) ADCB (C)  It not only gives us the necessities and comforts of life
(4) DBCA (5) No change required (D)  But money is merely a means and man is always more
EXERCISE-9 important than money.

Q.1-5.  Rearrange the following sentences (A),(B),(C), (E)  Money is the foundation of our society.
(D) (E) and (F) in the proper sequence to form a (F)  And it enables us to attain our cherished ambitions.
meaningful paragraph, then answer the questions Q.1.  Which of the following should be the FIRST sentence
given below them. after rearrangement?
(A)  An increasingly larger percentage of Indian population (1) A (2) B (3) C
today lives in the Urban areas.
(4) D (5) E
(B)  Over the last fifty years, while the country's population
has grown by 2.5 times Q.2.  Which of the following should be the THIRD sentence
after rearrangement?
(C)  India's Urban population is now second largest in the
world after China. (1) C (2) D (3) B
(D)  In the Urban areas it has grown by five times. (4) A (5) F
(E)  And is higher than the total Urban population of all the Q.3.  Which of the following should be the FIFTH sentence
countries put together barring China,USA and Russia. after rearrangement?
(F)  Contrary to popular concepts of a predominantly rural (1) D (2) B (3) A
India. (4) E (5) F

ENGLISH LANGUAGE 43
stportal.mahendras.org
Q.4.  Which of the following should be the FOURTH Q.5.  Which of the following would be the THIRD sentence
sentence after rearrangement? after rearrangement?
(1) A (2) B (3) F (1) D (2) E (3) F
(4) E (5) D (4) A (5) C
Q.5.  Which of the following should be the LAST sentence EXERCISE-12
after rearrangement?
Q.1-5.  Rearrange the following sentences (A),(B),(C),
(1) A (2) B (3) C (D) ,(E)and (F) in the proper sequence to form a
(4) D (5) E meaningful paragraph, then answer the questions
given below them. 
EXERCISE-11
(A) to these tragedies is similar to that of suicides in other
Q.1-5.  Rearrange the following sentences (A),(B),(C), groups in our society. This is no better 
(D) (E) and (F) in the proper sequence to form a
(B) knowledge of effective interventions to prevent
meaningful paragraph, then answer the questions
suicide, there remains no coordinated effort 
given below them. 
(C) to address suicide as a public health issue in India.
(A) and gender discrimination to the pressures of academic
Thus it comes as no surprise that the suicide 
performance and the abject failure of the systems
intended to hold private  (D) illustrated than by the discourse on farmer suicides
being viewed almost entirely through a socio-political
(B) Each of these events has been extensively dissected as
lens.
discrete events, with commentators emphasising the
relationship of the  (E) rates in young people in India are among the highest
in the world. In this regard, our response 
(C) deaths of these young people to the particular social
problems they found themselves struggling with, from (F) Despite this mountain of evidence testifying to the
caste-based oppression  huge toll of suicide in our youth and the
(D) Institutions accountable for the quality of their Q.1.  Which of the following should be the FIRST sentence
education. In the din that has surrounded each of these after rearrangement?
tragedies, with the predictable  (1) A (2) B (3) C
(E) has escaped our collective attention: the loss of hope (4) D (5) F
in the young educated victims trapped in a system that Q.2.  Which of the following should be the THIRD sentence
had failed them. after rearrangement?
(F) hysteria in TV news debates and the slugfest between (1) C (2) D (3) B
rival ideological camps, one common thread running
through all of these suicides  (4) A (5) F
Q.1.  Which of the following would be the LAST sentence Q.3.  Which of the following should be the FIFTH sentence
after rearrangement? after rearrangement?

(1) D (2) E (3) F (1) D (2) B (3) A

(4) A (5) C (4) E (5) F

Q.2.  Which of the following would be the FIRST sentence Q.4. Which of the following should be
after rearrangement? the FOURTH sentence after rearrangement?
(1) A (2) B (3) F
(1) B (2) F (3) C
(4) E (5) D
(4) D (5) A
Q.5.  Which of the following should be the LAST sentence
Q.3.  Which of the following would be the FOURTH
after rearrangement?
sentence after rearrangement?
(1) A (2) B (3) C
(1) A (2) B (3) D
(4) D (5) E
(4) F (5) E
Q.4.  Which of the following would be the FIFTH sentence EXERCISE-13
after rearrangement? Q.1-5.  Rearrange the following sentences (A),(B),(C),
(1) A (2) C (3) D (D), (E), (F) and (G) in the proper sequence to
form a meaningful paragraph, then answer the
(4) B (5) E questions given below them.

44 ENGLISH LANGUAGE
stportal.mahendras.org
(A) Though the Common Man character in his cartoons (D) In most cases, it leads to infighting and dispute in the
was always silent, the intended message was very family
loud and clear. 
(E) Among the challenges that the heritage hotels face are
(B) If a picture is worth a thousand words ,a cartoon is that most of them are in the rural areas and their titles
worth a thousand pictures are not clear as per the law. 
(C) One feels sad to read in the leading dailies that the (F) That would in times to come, benefit people who own
legendary cartoonist RK Laxman is no more.  forts, havelis and other heritage structures, sans a clear
(D) R.K.Laxman will always be remembered for his early title. 
morning dose of laughter.  Q.1. Which of the following would be the FOURTH
(E) He always cared for the plight of common man sentence after rearrangement?
through his immortal cartoons. 
(1) D (2) E (3) B
(F) His cartoons would bring a smile even on the faces of
(4) C (5) A
people who didn’t know to read or were in a hurry. 
(G) He embodied the hopes, aspirations and troubles of Q.2. Which of the following would be the FIRST sentence
Indians for over half a century.  after rearrangement?

Q.1. Which of the following would be (1) C (2) F (3) E


the SECOND sentence after rearrangement? (4) B (5) D
(1) D (2) F (3) B Q.3. Which of the following would be the SECOND
(4) G (5) C sentence after rearrangement?
Q.2. Which of the following would be the LAST sentence (1) E (2) B (3) A
after rearrangement? (4) D (5) C
(1) G (2) B (3) A Q.4. Which of the following would be the SIXTH sentence
(4) E (5) D after rearrangement?
Q.3. Which of the following would be the THIRD sentence (1) B (2) A (3) F
after rearrangement?
(4) C (5) D
(1) B (2) G (3) E
Q.5. Which of the following would be the THIRD
(4) D (5) F sentence after rearrangement?
Q.4. Which of the following would be the FIRST sentence (1) A (2) B (3) F
after rearrangement?
(4) D (5) E
(1) C (2) A (3) B
(4) E (5) D
EXERCISE-15
Q.5. Which of the following would be the FIFTH sentence Q.1-10. If the (A) part of the sentences is fixed at its place
after rearrangement? then what will be the correct rearrangement of the
sentences given below and in case you feel the given
(1) G (2) B (3) D
sentences are in their correct sequence then mark
(4) A (5) F option (5) i.e., no change required as your answer.
EXERCISE-14 Q.1. The reason blood pressure tends to rise (A)/ tribal
Q.1-5.  Rearrange the following sentences (A),(B),(C), communities that has been (B)/ concludes a study
(D), (E) and (F) in the proper sequence to form a of remote(C)/ with age may be a Western diet and
meaningful paragraph, then answer the questions lifestyle, (D)/ recently published in the journal.(E)
given below them.
(1) BCED (2) CBDE (3) DCBE
(A) Now there is hope for their restoration as the
(4) EBCD (5) No change required
government has proposed to amend the Municipal Act 
Q.2. When I was in Chhattisgarh 30 years ago,(A)/ the
(B) With legal documents not in place, the property often
tribal people didn’t suffer(B)/ was frugal yet more
faces problems in land conversion
or less balanced. (C)/ diabetes or cholesterol and
(C) Hundreds of heritage properties in the state are lying their diet(D)/ from any lifestyle disorder — high
in a dilapidated condition for want of funds to be blood pressure, (E)
restored. 
ENGLISH LANGUAGE 45
stportal.mahendras.org
(1) BCED (2) CBDE (3) BEDC flinch when charging (C)/ while he smokes with
(4) EBCD (5) No change required abandon in his clinic (D)/ for treating their imagined
illnesses.(E)
Q.3. The Anganwadi Services scheme, (A)/ caters to
children up (B)/ which incorporates the Integrated (1) BCED (2) CBDE (3) DBEC
Child Development Services, (C)/ to age six, and to (4) EBCD (5) No change required
(D)/ pregnant and lactating women.(E) Q.10. The grass can be used to(A)/ purify polluted water
(1) BCED (2) CBDE (3) DBEC bodies,(B)/ especially temple tanks, and to arrest
(4) EBCD (5) No change required soil erosion(C)/ in Kerala and Kodaikanal (D)/ in
ghat sections, especially.(E)
Q.4. By studying nine different pharmaceutical active
compounds in Yamuna river,(A)/ and to humans (1) BCED (2) CBDE (3) DBEC
who use (B)/ this water for drinking purposes (C)/ (4) EBCD (5) No change required
researchers have now pointed out that it can (D)/ Q11-15. The sentences (A, B, C, D, E, F, G and H) given in
“possibly cause chronic toxicity” to aquatic life.(E) the question are the divided parts of a paragraph.
(1) BCED (2) CBDE (3) DBEC In order to make a coherent paragraph, rearrange
(4) EBCD (5) No change required the sentences except B & D and G (highlighted) as
these are correctly fixed at the second, fourth and
Q.5. The mass protests have, unwittingly,(A)/ to meet the seventh place respectively.
the Paris climate agreement targets (B)/ would
have been(C)/ hit by the higher levy against the A. stands at the head of a wide approach.
imperative(D)/ pitted the majority who.(E) B. sculptor, artist and the Lincoln Memorial
(1) BCED (2) CBDE (3) DBEC Commission,

(4) EBCD (5) No change required C. with the expenditure of more than $2,000,000,
has brought to
Q.6. The health, longevity and well-being of
Indians(A)/ has improved since Independence, D. completion the splendid Lincoln Memorial
and the high levels of economic growth(B)/ over at Washington which will be dedicated soon.
the past two-and-half-decades (C)/ to spend on the E. is visible from a distance.
social sector (D)/ have made more funds available. F. Ten years of the combined effort of architect,
(E)
G. Within the portico the great statue of Lincoln,
(1) BCED (2) CBDE (3) DBEC by Daniel Chester French,
(4) EBCD (5) No change required H. This Memorial, a truly splendid achievement of
Q.7. What prompted the U.S. and China to arrive modern art,
at an unexpected,(A)/ important to see if any Q.11. Which of these would be the first sentence after
compromise (B)/ albeit temporary, compromise is rearranging the given sentences?
unclear and it will be (C)/ between the two trade
giants will include a complete rollback of the (1) A (2) C (3) E
tariffs(D)/ imposed on each other over the year.(E) (4) F (5) H
(1) BCED (2) CBDE (3) DBEC Q.12. Which of these would be the third sentence after
(4) EBCD (5) No change required rearranging the given sentences?

Q.8. The best source of entertainment(A)/ from a (1) A (2) C (3) E


neighbourhood video library pirate,(B)/ brought (4) F (5) H
into the country(C)/ by friends and family or Q.13. Which of these would be the fifth sentence after
borrowed(D)/ was watching movies recorded on rearranging the given sentences?
video cassettes.(E)
(1) A (2) C (3) E
(1) BCED (2) CBDE (3) DBEC
(4) F (5) H
(4) EBCD (5) No change required
Q.14. Which of these would be the sixth sentence after
Q.9. Then there’s Dr. Kulkarni, Bhaskar’s friend rearranging the given sentences?
(A)/ rich patients an exorbitant fee (B)/ who doesn’t
46 ENGLISH LANGUAGE
stportal.mahendras.org
(1) A (2) C (3) E G. which rang with the profoundest measures of
(4) F (5) H freedom and justice.

Q.15. Which of these would be the eighth sentence after H. no one can dream that these men had not got
rearranging the given sentences? the most sublime ideal of democracy which
had ever dawned upon the souls of men.
(1) A (2) C (3) E
Q.16. Which of these would be the first sentence after
(4) F (5) H rearranging the given sentences?
Q16-20. The sentences (A, B, C, D, E, F, G and H) (1) A (2) C (3) E
given in the question are the divided parts
of a paragraph. In order to make a coherent (4) F (5) G
paragraph, rearrange the sentences except B, D Q.17. Which of these would be the third sentence after
and H (highlighted) as these are correctly fixed rearranging the given sentences?
at the first, fourth and the last place respectively. (1) A (2) C (3) E
A. in which the people would be the supreme (4) F (5) G
power, and so inspired by this vision men
Q.18. Which of these would be the fifth sentence after
B. dawned upon your ancestors, what it would rearranging the given sentences?
mean for men to be free. They got the vision
of a government (1) A (2) C (3) E

C. never in the history of the world did it dawn (4) F (5) G


upon the human mind as it Q.19. Which of these would be the sixth sentence after
D. wrote such documents as were went from the rearranging the given sentences?
Massachusetts legislature, (1) A (2) C (3) E
E. from the New York legislature and from the (4) F (5) G
Pennsylvania group over to the Parliament of Q.20. Which of these would be the seventh sentence after
Great Britain, rearranging the given sentences?
F. They did not equivocate in a single word when (1) A (2) C (3) E
they wrote the Declaration of Independence;
(4) F (5) G

ENGLISH LANGUAGE 47
stportal.mahendras.org

explanation
is mentioned in 'A'.
EXERCISE 1
‘C’ is the third statement as it is directly connected to the
Q.1.(3) Q.2.(4) Q.3.(2)
second statement ‘D’
Q.4.(2) Q.5.(5) Q.6.(3)
‘D’ is the second statement as it is continuing the idea.
Q.7.(2) Q.8.(4) Q.9.(1)
‘B’ is the last statement as it ends the idea.
Q.10.(4) Q.11.(2) Q.12.(3)
Q.1.(4) Q.2.(4) Q.3.(2)
Q.13.(3) Q.14.(4) Q.15.(3)
Q.4.(4) Q.5.(5)
Q.16.(3) Q.17.(2) Q.18.(4)
Q.19.(2) Q.20.(3) EXERCISE 5
The correct arrangement is ACEBD
EXERCISE 2
‘A’ is the first statement as it starts the conversation about
The correct re-arrangement is CABED
the study of speech disorder.
‘C’ is the first statement as it starts the story which is about
‘C’ is the second statement as it is used as a example for the
a Chairman.
first statement.
‘A’ is the second statement as it is directly connected to ‘C’
‘E’ is the third statement as it is directly connected to ‘C’
‘B’ is the third statement as it is talking about the responsibility statement.
of the chairman.
‘B’ is the fourth statement as it represent a continous idea.
‘E’ is the fourth statement as it is completing the third
‘D’ is the last statement as it ends the conversation.
statement.
Q.1.(3) Q.2.(2) Q.3.(1)
‘D’ is the last statement as it ends the conversation.
Q.4.(2) Q.5.(5)
Q.1.(2) Q.2.(4) Q.3.(4)
Q.4.(1) Q.5.(4) EXERCISE 6
The arragement is “BDEAC”
EXERCISE 3
Q.1.(2) Q.2.(5) Q.3.(4)
The correct re-arrangement is CBFAED
Q.4.(1) Q.5.(3)
‘C’ is the first statement because it is an independent
statement which starts the conversation. EXERCISE 7
‘B’ is the second statement as in the statement, ‘It’ is used for The correct arrangement is “DFBECHGA” “D” is the
the force which is mentioned in first statement. first statement which is followed by ‘F’ and ‘B’ to make a
‘E’ is the fifth statement as it starts concluding the idea. complete sense.
‘D’ is the last statement as it is directly connected to ‘E’ ‘E’ is the fourth statement as it proceeds the idea.
because in both the statement the conversation is going on ‘C’ is the fifth statement as it starts with an auxiliary verb,
‘The Mass’. which is clearly connected to the ‘E’ statement.
‘F’ is the third statement as here this gravitational force ‘H’ is the sixth statement which is followed by “G & A” to
is related to “The force of attraction ” which is the fourth make a complete sense.
statement.
Q.1.(2) Q.2.(5) Q.3.(4)
‘A’ is the fourth statement as it tells something about the earth.
Q.4.(1) Q.5.(3)
Q.1.(3) Q.2.(4) Q.3.(1)
EXERCISE 8
Q.4.(5) Q.5.(1)
Q.1.(4) He is planning to race ahead of e-retailers and catch
EXERCISE 4 the hinterland consumer while he is still fiddling
The correct re-arrangement is EDCAFB with his first smartphone.
‘E’ is the first statement as it is an independent statement Q.2.(5)
which starts the conversation. Q.3.(2) Several states have reduced VAT we will appeal
‘A’ is the fourth statement as it gives the example of the again States, which have high VAT component,
they should reduce for consumers interest," the
philosopher in reference to 'C'.
minister told reporters.
‘F’ is the fifth statement as here the thought of the philospher
Q.4.(2)
48 ENGLISH LANGUAGE
stportal.mahendras.org
Q.5.(4) There is a real and heightened risk of investment
EXERCISE 15
bubble of the type seen in Ponzi schemes which
can result in sudden and prolonged crash exposing Q.1.(3) The reason blood pressure tends to rise with age
investors, especially retail consumers losing their may be a Western diet and lifestyle, concludes a
study of remote tribal communities that has been
hard-earned money.
recently published in the journal.
Q.6.(5)
Q.2.(3) When I was in Chhattisgarh 30 years ago,
Q.7.(3) Mount Sinabung roared back to life in 2010 for the tribal people didn’t suffer from any lifestyle
the first time in 400 years, after another period of disorder — high blood pressure, diabetes or
inactivity it erupted once more in 2013, and has cholesterol and their diet was frugal yet more or
remained highly active since. less balanced.
Q.8.(2) Users and traders of VCs have already been Q.3.(2) The Anganwadi Services scheme, which
cautioned three times, by the Reserve Bank of incorporates the Integrated Child Development
India about the potential financial, and security Services, caters to children up to age six, and to
pregnant and lactating women.
related risks that they are exposing themselves to
by investing in Bitcoin. Q.4.(3) By studying nine different pharmaceutical
active compounds in Yamuna river, researchers
Q.9.(5) have now pointed out that it can “possibly cause
Q.10.(4) As internet penetrates Bharat and phones and data chronic toxicity” to aquatic life and to humans who
become cheaper, the e-retailers wait for Indians in use this water for drinking purposes.
the hinterland to get online and buy stuff from them. Q.5.(4) The mass protests have, unwittingly, pitted the
Q.11.(5) majority who would have been hit by the higher
levy against the imperative to meet the Paris
EXERCISE 9 climate agreement targets.
Q.1.(1) D Q.2.(2) F Q.3.(5) E Q.6.(1) The health, longevity and well-being of Indians
Q.4.(4) B Q.5.(5) C has improved since Independence, and the high
levels of economic growth over the past two-and-
The rearrangement is FACEBD half-decades have made more funds available to
EXERCISE 10 spend on the social sector.
Q.1.(5) E Q.2.(4) A Q.3.(1) D Q.7.(2) What prompted the U.S. and China to arrive
at an unexpected, albeit temporary, compromise
Q.4.(3) F Q.5.(2) B is unclear. It will be important to see if any
The rearrangement is ECAFDB compromise between the two trade giants will
include a complete rollback of the tariffs imposed
EXERCISE 11 on each other over the year.
Q.1.(3) F Q.2.(1) B Q.3.(3) D Q.8.(4) The best source of entertainment was watching
Q.4.(5) E Q.5.(4) A movies recorded on video cassettes, brought into
the country by friends and family or borrowed
The rearrangement is BCADEF
from a neighbourhood video library pirate.
EXERCISE 12 Q.9.(2) Then there’s Dr. Kulkarni, Bhaskar’s friend
Q.1.(5) F Q.2.(1) C Q.3.(3) A who doesn’t flinch when charging rich patients an
exorbitant fee for treating their imagined illnesses
Q.4.(4) E Q.5.(4) D
while he smokes with abandon in his clinic.
The rearrangement is FBCEAD Q.10.(1) The grass can be used to purify polluted water
EXERCISE 13 bodies, especially temple tanks, and to arrest soil
erosion in ghat sections, especially in Kerala and
Q.1.(4) Q.2.(2) Q.3.(5)
Kodaikanal.
Q.4.(1) Q.5.(3) Q.11.(4) Q.12.(2) Q.13.(5)
The arrangement would be CGEADFB. Q.14.(1) Q.15.(3)
EXERCISE 14 The correct sequence is: FBCDHAGE.
Q.1.(2) Q.2.(1) Q.3.(3) Q.16.(2) Q.17.(1) Q.18.(3)
Q.4.(5) Q.5.(3) Q.19.(5) Q.20.(4)
The arrangement would be CAFEBD.  The correct sequence is : CBADEGFH

ENGLISH LANGUAGE 49
stportal.mahendras.org

CHAPTER

8 PREPOSITION
Scan the QR code to get video of this chapter.

What is a Preposition?
A Preposition is a word or phrase placed before a Noun or Pronoun or Noun Phrase or Pronoun Phrase. Prepositions are
used to indicate various kinds of relations, such as, place, time, method and manner, reason and purpose, possession and
direction and motion.
A preposition usually indicates the temporal, spatial or logical relationship of its object to the rest of the sentence as in the
following examples:
The book is on the table. The book is beneath the table.
The book is leaning against the table. The book is beside the table.
She held the book over the table. She read the book during the class.
Problems with Prepositions

Rule 1.
You may end a sentence with a preposition. Just do not use extra prepositions when the meaning is clear without them.
Examples:
Correct: Where did he go? Incorrect: Where did he go to?
Correct: Where did you get this? Incorrect: Where did you get this at?
Correct: I will go later. Incorrect: I will go later on.
Correct: Take your shoes off the bed. Incorrect: Take your shoes off of the bed.
Correct: Lets cope with stress Incorrect: Lets cope up with stress
Correct: Cut it into small pieces. Incorrect: Cut it up into small pieces.
Rule 2. Example:
Use on with expressions that indicate the time of an occur- Divide the candies between the two of you.
rence. Divide the candies among the three of you.
Example: Rule 5.
He was born on December 23. The word like may be used as a preposition and in informal
We will arrive on the fourth. writing, as a conjunction. In formal writing, use as, as if, or
as though rather than like as the conjunction.
Rule 3.
Examples:
Of should never be used in place of have.
Prepositional usage -
Correct: I should have done it.
Incorrect: I should of done it. You look so much like your mother.
Rule 4. Conjunction usage - You look like you are angry.
OR
Between refers to two. Among is used for three or
more. You look as if you are angry.

50 ENGLISH LANGUAGE
stportal.mahendras.org

Types Of Preposition
Prepositions – Time

Clip Art Preposition at Example


at
(a time) “I got to work at 8.00 this morning.”
on “I’ll see you on Tuesday.”
(a day)
on “I have an appointment on 31st.”
(a date)
in “My birthday is in June”
(a month)
in
(a year) “I was born in 1959”
in “I get up in the morning.”
(the morning)
in “I have lunch in the afternoon.”
(the afternoon)
in “I go home in the evening.”
(the evening)
at “I go to bed at night.”
(night)
in (a season) “It usually snows in the winter.”

Preposition English Usage Example


in days of the week on Monday
in months / seasons time of day, year after in August / in winter/ in the morning in
a certain period of time (when?) 2006/ in an hour
at for night, for weekend, a certain point at night, at the weekend, at half past
of time (when?) nine
since from a certain point of time (past till since 1980
now)
for over a certain period of time (past till for 2 years
now)
ago a certain time in the past 2 years ago
before earlier than a certain point of time before 2004
to telling the time ten to six (5:50)
past telling the time ten past six (6:10)
to / till / until marking the beginning and end of a From Monday to/till Friday
period of time
till / until in the sense of how long something is He is on holiday until Friday.
going to last
by In the sense of at the latest I will be back by 6 o’clock.
up to a certain time By 11 o’clock, I had read five pages

ENGLISH LANGUAGE 51
stportal.mahendras.org

Prepositions – Place (Position and Direction)


Where is it?

The ball is in the box The ball is on the box. The ball is under the box.
house
Jai’shouse Priti’s house Pragati’s house

Jai’s house is next


behindto Priti’s house is between Pragati’s Pragati’s house is next to
Priti’s house. and Jai’s house. Priti’s house.

The climbers stood


climber stood The man stood The enemies stood The gardners The man stood
on top of the between the opposite
opposite to each
each stood behind next to the
mountain. two enemies. other. the pumpkins. the umbrella
gopher and held over it

The man looked The man wrote The man looked The man looked The manager
through the the address at the mail in at the clock on sat at his desk
telescope. on the package. the post box. the wall. on his chair his hands.

The following table contains rules for some of the most frequently used prepositions in English:
The following examples will give you an idea about the use of prepositions:-
1. abide by : Everyone must abide by the judgment of the court.
2. abide with : We must abide with our friends through thick and thin.
3. abound in : The sea abounds in wealth.
4. abscond from : He absconds from classes only when he is ill.
5. absent from : The clerk is absent from his section.
6. absolve of : The judge absolved the official of charges of corruption.

52 ENGLISH LANGUAGE
stportal.mahendras.org

7. absorb in : Lata is absorbed in singing.


8. abstain from : Postal employees abstained from their duties protesting against the government’s
indifference towards their problems.
9. abundance of : There is abundance of oil and natural gas in the Godavari basin.
10. abundant in : Rice is abundant in the Godavari district of Andhra Pradesh.
11. accede to : The Government acceded to the demands of the striking employees.
12. accompanied by : Bill Clinton, the President of the United States of America, accompanied by his
daughter and other high-level officials, visited India.
13. according to : According to Darwin, man has evolved out of the ape.
14. account for : The accountant must account for the shortage of cash in the treasury.
15. accuse of : The man was accused of smuggling and murder.
16. accustomed to : The boy is accustomed to reading till midnight.
17. acquit of : The judge acquitted him of all charges of corruption.
18. acquaint with : After returning from South Africa, Gandhi went round the country to acquaint himself
with the people and their problems.
19. adapt to : We must adapt ourselves to different environments.
20. admit to : He admitted his son to a public school.
21. adhere to : Government employees must adhere to a code of conduct.
22. adjacent to : My office is adjacent to my house.
23. adjourn to : For want of quorum the committee meeting was adjourned to the next day.
24. afraid of : Children are afraid of dogs.
25. agree to : The Government agreed to the demands of the employees.
26. agree with : I agree with you that afforestation brings ecological balance.
27. agree on : All the members of the teachers’ association agreed on the need to observe punctuality.
28. ail from : The nation is ailing from disunity.
29. aim at : The hunter aimed his shot at the deer.
30. akin to : Man is akin to other animals in almost all matters.
31. argue against The leaders of the opposition parties argued with the Prime Minister against
..with : revising the Constitution.
32. ashamed of : Ashamed of her failure in the examinations, a girl committed suicide.
33. ask for : He asked me for a sheet of paper.
34. aspire for : Avaricious persons aspire for more and more money.
35. associate with : Vinoba Bhave was associated with the Sarvodaya Movement.
36. attend to : I have urgent work to attend to.
37. attend on : Doctors and nurses attend on patients.
38. attentive to : Snakes are said to be attentive to music.
39. authority on : Dr. S.Radhakrishnan was an authority on Eastern Philosophy.
40. authority over : In India the executive has no authority over the judiciary.
41. avail of : We must avail ourselves of every opportunity to prosper in life.
42. aware of : In a democratic country the people must be aware of their duties and respon sibilities
43. bear with : As a teacher I cannot bear with indiscipline.
44. believe in : Indians believe in superstitions.
45. beloved of : The last child is generally the most beloved of the parents.
46. benefit from : We must be benefited from the experience and example of others.
47. benefit of : There is no benefit of sending sub-standard sports team abroad.
48. bend before : We should not bend before traitors.

ENGLISH LANGUAGE 53
stportal.mahendras.org

49. bend on : The University is bent on introducing information technology courses.


50. bend to : Weak persons bend to pressure.
51. bestow on : God bestowed his favour on me.
52. binding on : The resolutions passed by the majority of the cabinet, are binding on the entire Cabinet.
53. bless with : I am blessed with four daughters.
54. blind in : The beggar seems to be blind in both the eyes.
55. blind to : Parents generally are blind to the faults of their children.
56. blush at : The newly married couple blush at each other.
57. boast of : One should not boast of one’s own virtues.
58. boil with : Gopal boiled with anger when Ravi called him a rogue.
59. born of : Michael Faraday was born of poor parents.
60. brim with : On the eve of the examinations, the boy is brimming with confidence.
61. bristle with : An honest man’s path is always bristled with difficulties.
62. brood over : There is no use of brooding over our past misfortunes.
63. busy in : I am busy in preparing notes.
64. busy with : The Principal is busy with examinations.
65. capable of : Bears are capable of climbing trees.
66. care about : Selfish rulers do not care about the welfare of toiling masses.
67. care for : Some persons do not care for breakfast.
68. careful of : The student was careful of his words in front of the Principal.
69. cash in on : Unscrupulous people try to cash in on our ignorance.
70. cause for : Although there is political instability in the country, there is no cause for panic.
71. charge against : An opposition leader leveled charges of corruption against a minister.
72. charge with : An official of the Air Force was charged with spying for a foreign country.
73. claim to : Both sons and daughters have a claim to ancestral property.
74. clash of : The clash of interests between America and Russia took the world to the brink of disaster.

75. clash with : The Border Security Force jawans clashed with some cross country smugglers.
76. climb up…down : Cats can climb up and down the trees very quickly.
77. cling to : One must cling to the truth at all times.
78. close to : Himanshu and Jaspreet are close to each other.
79. Collide with : Many people died when a passenger bus collided with a transport lorry.
80. collude with : Terrorists and extremists colluded with smugglers to destroy the country.
81. combine into : Paes and Bhupati have combined into a fine doubles team in Tennis.
82. commensurate Wages must be commensurate with the volume of work.
with:
83. commit to : Democracy is committed to the welfare of the people.
84. commune with : Spiritualists commune with supernatural powers.
85. compare to : Sardar Patel is compared to Bismarck of Germany.
86. compare with : Mumbai can be compared with Hollywood in some respects.
87. compensate for : The Management must compensate in cash for the loss of life or limb suffered by the workers.

88. eliminate from : Corrupt officials must be eliminated from public life.

54 ENGLISH LANGUAGE
stportal.mahendras.org

exercise
Q.5. Mohan will never pass his SSC examination 1 ...........
EXERCISE-1
he works hard.
Complete the exercise according to the picture.
(A) if (B) unless (C) since (D) because
Q.1. ----------The picture, I can see a woman.
Q.6. If you live in a corrupt society, you cannot easily
Q.2. She is sitting--------a chair. rise ..... the prevailing corruption.
Q.3. There is another chair ------------the woman. (A) upon (B) over (C) above (D) beyond
Q.4. Her feet are -----------the table Q.7. It was the first time he had eaten a square meal........
Q.5. The woman is holding a cup ------------her hands. he had left the village.
EXERCISE-2 (A) since (B) for
Fill the correct prepositions in the blank. (C) before (D) although
Q.1. Ravi is playing tennis ---------- Sunday. Q.8. They are very grateful............. your kindness.
Q.2. My brother's birthday is ----- the 5th of November. (A) for (B) to
Q.3. My birthday is ------------ May. (C) with (D) towards
Q.4. We are going to see my parents ------ the weekend. Q.9. His mother was 45 when she......
Q.5. ---------1666, a great fire broke out in London. (A) had died (B) died
Q.6. I don't like walking alone in the streets------- night. (C) was dying (D) has died
Q.7. What are you doing ------------ the afternoon? Q.10. There was nothing he could do ............... wait.
Q.8. My friend has been living in Canada ------- two years. (A) and (B) but
Q.9. I have been waiting for you--------- seven o'clock. (C) otherwise (D) than
Q.10. I will have finished this essay ------------ Friday. EXERCISE-5
EXERCISE-3 Find the errors and justify your answers
Fill in the blanks with correct preposition Q.1. He was sleeping (A)/ in his room when a thief (B)/
Q.1. The first McDonald's restaurant was opened ------- entered into his house (C)/ and took away a lot of
-------- Dick and Mac McDonald --------- the 15th things (D)/ No Error (E)
---------- May 1940. Q.2. In her concluding speech (A)/ she said (B)/ almost
Q.2. The best selling products --------- their restaurant nothing (C)/ worth listening to (D)/ No Error (E)
were hamburgers. Q.3. It was apparent for (A)/ everyone present (B)/ that if
Q.3. So the McDonald brothers thought ------------- a way the patient did not receive (C)/ immediate medical
to produce hamburgers more quickly. aid he would die (D)/ No Error (E)
Q.4. This was introduced ------------- 1948 and became Q.4. He proposed me (A)/ that we should go to the Disco
known ------------ the Speedee Service System. (B) and then have (C)/ dinner at a resturant (D)./ No
Error (E)
Q.5. The first franchised McDonald's restaurant was
opened ------------ 1953, and today you can find Q.5. There appears (A)/ to be a little liaison (B)/ among
the (C)/ two groups of the society (D) ./ No Error
McDonald's restaurants ---- more than 100 countries. (E)
EXERCISE-4 Q.6. The team (A)/ complained to the manager (B)/
Fill in the blanks. against the captain (C)/ and the poor facilities
Q.1. The shopkeeper does not have the toys, I was provided in the hotel (D). / No Error (E)
looking.......... Q.7. Yesterday I met (A)/ a man (B)/ who was blind (C)/
(A) up (B) about (C) for (D) to with the right eye (D)/ No Error (E)
Q.2. Books are very often compared.............. agranary. Q.8. The principal distributed (A)/ the sweet among our
(A) with (B) to (C) by (D) at friends (B)/ who bade (C)/ him farewell (D)/ No
Error (E)
Q.3. Divide twelve mangoes ............. three boys.
Q.9. As per the invitation (A)/ Rahim marries (B)/ with
(A) to (B) for Sayra (C)/ on 13 th December, Monday (D) / No
(C) between (D) among Error (E)
Q.4. To reach their village, they have to change Q.10. The debacle of the congress party (A)/ admit no
............. a small train at the junction. other explanation (B)/ than its (C)/ poor performance
(A) on (B) to (C) over (D) into during the last five years (D)/ No Error (E)
ENGLISH LANGUAGE 55
stportal.mahendras.org

Explanation
Q.3. So the McDonald brothers thought of a way to
EXERCISE 1
produce hamburgers more quickly.
Q.1. In the picture, I can see a woman.
Q.4. This was introduced in 1948 and became known
Q.2. She is sitting on a chair. as the Speedee Service System.
Q.3. There is another chair in front of the woman. Q.5. The first franchised McDonald’s restaurant
Q.4. Her feet are under the table was opened in 1953, and today you can find
McDonald’s restaurants in more than 100
Q.5. The woman is holding a cup in her hands. countries.
EXERCISE 2 EXERCISE 4
Q.1. Ravi is playing tennis on Sunday. Q.1.(C) Q.2.(B) Q.3.(D)
Q.2. My brother’s birthday is on the 5th of November. Q.4.(D) Q.5.(B) Q.6.(C)
Q.3. My birthday is in May. Q.7.(A) Q.8.(A) Q.9.(B)
Q.4. We are going to see my parents at the weekend. Q.10.(B)
Q.5. In 1666, a great fire broke out in London. EXERCISE 5
Q.6. I don’t like walking alone in the streets at night. Q.1.(C) Delete ‘into’ (enter does not take preposition in
case of physical presence.
Q.7. What are you doing in the afternoon?
Q.2.(D) Delete ‘to’
Q.8. My friend has been living in Canada for two years.
Q.3.(A) Change it to ‘apparent to’
Q.9. I have been waiting for you since seven o’clock.
Q.4.(A) Change it to ‘proposed to’
Q.10. I will have finished this essay by Friday.
Q.5.(C) Change among to between. Between is used for
EXERCISE 3 two.
Q.1. The first McDonald’s restaurant was opened by Q.6.(D) and about the poor
Dick and Mac McDonald on the 15th of May Q.7.(D) Correct phrase is ‘blind in’
1940.
Q.8.(B) Replace ‘among’ by ‘amongst’
Q.2. The best selling products at their restaurant were
hamburgers. Q.9.(B) Delete with. (marry does not take a preposition)
Q.10.(B) Place of after admits.

56 ENGLISH LANGUAGE
stportal.mahendras.org

CHAPTER

9 ADJECTIVES
Scan the QR code to get video of this chapter.

Adjectives are describing words which add details about the Examples:
nouns in a sentence. Adjectives are usually placed BEFORE 1. The hand has five fingers.
the nouns they modify. However, adjectives can occasionally
be found AFTER nouns and pronouns. 2. Sunday is the first day of the week.

Example sentences: Adjectives of Number are of two kinds:-

He is a strong boy. 4 (a). Definite Numeral Adjectives, which denote an exact


number as,
He has big blue eyes.
One, two, three, etc. - These are called Cardinals.
The new car broke down.
First, second, third, etc. - These are called Ordinals.
The old lady was talking in a quiet voice.
A Cardinal denotes how many, and an Ordinal denotes
Examples: the order of things in a series.
Sports car Orange juice 4 (b). Indefinite Numeral Adjectives,
Television station which do not denote an exact
number.
Coffee shop, Book cover
All, many, few, some, any, several, certain.
Types of Adjectives.
5. Distributive Adjectives which refer to each one of a
1. Adjectives of Quality (Descriptive Adjective) show number as
the kind or quality of a person or thing.
Each boy must take his turn.
Adjectives of Quality answer the question: Of what
kind? England expects every man to do his duty.
1. London is a large city. Either pen will do.

2. He is an honest man. 6. Demonstrative Adjective point out which person or


thing is meant.
3. The foolish old crow tried to sing.
Examples:
2. Proper Adjective: Adjectives formed from Proper
Nouns (e.g., French wines, Indian tea, Turkish tobacco) That boy is clever.
are sometimes called Proper Adjectives. These mangoes are sour.
3. Adjectives of Quantity show how much of a thing is I hate such things.
meant. It can be noticed that this and that are used with Singular
Adjectives of Quantity answer the question: How Nouns, these and those with Plural Nouns.
much?
7. Interrogative Adjectives - What, which and whose
Examples: when they are used with nouns to ask questions.
1. I ate some rice. Examples:
2. He has little intelligence. What manner of man is he ?
3. Take great care of your health. Whose book is this?
4. He showed much patience. Which way shall we go?
5. He has lost all his wealth. Comparison of adjectives:
4. Adjectives of Number (Numeral Adjective) show how Positive degree – the simplest form.
many persons or things are meant, or in what order a
Comparative degree – comparing two things or persons.
person or thing stands.
Superlative degree – comparing three or more things or
Adjectives of Number answer the question: How many?
persons.

ENGLISH LANGUAGE 57
stportal.mahendras.org

Positive Comparative Superlative Rules related to Adjectives


large larger largest Never use - er or - est at the same time you use more or most.
These are incorrect: more shorter, most alertest
horrible more horrible most horrible
Certain comparatives taken from Latin language have no
How to compare adjectives
positive or superlative degree.
1. Add - er and - est to short adjectives of one syllable
They all end in “or” but not in “er”. They are twelve
(sometimes two syllables)
in all.
Positive Comparative Superlative Five of them lost their comparative meaning, and are used
short shorter shortest as positive forms.
great greater greatest They are: exterior, interior, ulterior, major & minor
small smaller smallest e.g. The exterior wall of the house is made of stone; the
old older oldest interior walls are of wood.
2. If a short adjective ends with consonant y, change the Her age is a matter of minor importance.
y to i and add - er and - est. I have no ulterior motive in offering you my help.
Positive Comparative Superlative The other seven are used as comparative forms but
lucky luckier luckiest are followed by “to” instead of ‘than’.
heavy heavier heaviest They are: Inferior, superior, prior, anterior, posterior,
senior & junior
happy happier happiest
silly sillier silliest e.g. Raman is inferior to Ajay in intelligence.
ugly uglier ugliest Ajay’s intelligence is superior to Raman’s
3. If a short adjective ends with e, add - r and - st. He is junior to all his colleagues.
All his colleagues are senior to him.
Positive Comparative Superlative
fine finer finest Points to Remember
4. Put more(less) and most(least) in front of longer Each of the two words gives a different meaning; therefore,
adjectives (with two or more syllables) to show it is best to know them well:-.
comparison. Some/Any
Positive Comparative Superlative SOME : We use "some" in positive sentences. We use some
for both countable and uncountable nouns.
appealing less appealing least appealing
remarkable more remarkable most remarkable Example: I have some friends.
beautiful more beautiful most beautiful ANY: We use "any" in negative sentences or questions. We
use any for both countable and uncountable nouns.
alert more alert most alert
Example: Do you have any cheese? - He doesn't have any
5. Some adjectives are irregular adjectives and don't
friends in Chicago.
follow these rules. They change form considerably
from one degree to the next. You need to study these EXCEPTION!: We use "some" in questions when offering
changes carefully in order to recognize them easily. or requesting something that is there.
List of irregular adjectives. Example: Would you like some bread? (offer) - Could I have
some water? (request)
Positive Comparative Superlative
ANY: We use "any" in negative sentences or questions. We
bad worse worst use any for both countable and uncountable nouns.
far farther farthest
Example: Do you have any cheese? - He doesn't have any
good better best friends in Chicago.
many more most Foremost/first
little less least
These two words are synonyms, i.e. either word can be used.
old elder eldest However, there is some difference in their usage.
well/good better best “Foremost” means ‘the best or the most important; in a top
(healthy) or leading position in a group of people or things’.

58 ENGLISH LANGUAGE
stportal.mahendras.org
For example The last chapter is boring. (order of position)
Mr. Kapoor is one of the foremost authorities on animal Much/Many:
protection plans. "Much" modifies only uncountable nouns.
Mr. Kapoor is the first person to start a Trust (zoo) to protect
e.g. "They have so much money in the bank."
the endangered species of animals from around the world
e.g. "The horse drinks so much water."
Less/lesser
"Many" modifies only countable nouns.
These two words are the comparative forms of the word
‘little’ e.g. "Many Americans travel to Europe."
The difference is: “less” suggests ‘amount’, and “lesser” e.g. "I collected many sources for my paper”.
suggests degree showing some ‘negative’ sense in a choice Little/Few:
of two.
"Little" modifies only uncountable nouns.
For example,
e.g. "He had little food in the house."
She has less money than he (has).
e.g. "When I was in college, there was little money to
Which is the lesser of the two evils, drinking or smoking?
spare."
[Both ‘drinking alcohol’ and ‘smoking tobacco’ are evils, but
"Few" modifies only countable nouns.
we’d like to compare and decide which one is more harmful
– ‘more negative’ -- in this choice of two!] e.g. "There are a few doctors in town."
Elder, older; eldest, oldest e.g. "He had few reasons for his opinion."
“Elder and eldest” are used only for persons. Few, A Few, The Few
(they are used with members of the same family.) Few, when used without a preceding "a", means "very few"
“Elder” is not used with conjunction ‘than’. or "none at all". On the other hand, "a few" is used to indicate
"not a large number". But some or hand full. While "the Few"
“Older and oldest” are used for both persons and things –
is used for a special limited number (not many but all).
time (age)
1. Few artists live luxuriously.
e.g. Rahul is my elder brother. Ahmed is his eldest son.
2. Send me a few books.
Town Hall is the oldest building in our town.
3. That music appeals to the few.
Later, latter; latest, last
Enough:
Later and latest refer to time
Latter and last refer to position Enough modifies both countable and uncountable nouns.

E.g. He came later than I expected. e.g. "There is enough money to buy a car."

(he has come late) I have not heard the latest news. (recent news) e.g. "I have enough books to read."

The latter chapters of the book are interesting. (order of


position)

ENGLISH LANGUAGE 59
stportal.mahendras.org

exercise
Q.10. Yesterday Alok told me a joke. This joke was the
EXERCISE-1
............... joke I’ve ever heard.
Pick the correct word from the brackets to make the sentence
meaningful. EXERCISE-3
Q.1. My father is as (strong, stronger, strongest) as his Fill in the blanks with the following adjectives, each of which
father. is to be used only once.
Q.2. She is (pretty, prettier, prettiest) than her sister. Little; wild; dark; angry; eight; muddy; honest;
heavy; clumsy; much; proud; skinny; brave; narrow;
Q.3. You are not so (tall, taller, tallest) as your brother. blind.
Q.4. That pond is the (shallow, shallower, shallowest) in Q.1. I cannot lift this _____ metal box.
this area.
Q.2. That ___ boy knocked down the vase again.
Q.5. That has to be the (interesting, more interesting,
most interesting) film I have seen. Q.3. The thin beggar raised his _____ hand.
Q.6. Which University offers (the good, the better, the Q.4. The ____ soldier was awarded a medal.
best) degree courses? Q.5. Cars are too big to use this _____ path.
Q.7. This clown is not so (funny, funnier, funniest) as Q.6. Tigers are _____ animals.
the other one.
Q.7. He is poor. He hasn’t _____ money.
Q.8. He is easily the (bad, worse, worst) player in the
Q.8. He is an _____ man. You can trust him.
team.
Q.9. They helped the ___ man cross the road.
Q.9. The second half of the play was (little, less, the
least) interesting. Q.10. Look at the _____ sky. It is going to rain.
Q.10. What is (far, farther, the farthest) distance you have EXERCISE-4
ever run?
Find the errors and justify your answer
EXERCISE-2 Q.1. Kolkata is further (A) /from Alwar (B) /than Jaipur
Fill the correct degree(comparative or superlative) by changing (C) /the capital of Rajasthan (D).
the words given in bold in the first sentence.
Q.2. Ramesh is smarter (A) /enough to get (B)
Example: I have a fast car, but my friend has a __ /selected for this (C) /post, without any
car. recommendations(D).
Answer: I have a fast car, but my friend has a faster Q.3. He said, ‘Priyanka (A) /is the most unique (B] /
car.
singer of (C) /our college (D).’
Q.1. This is a nice cat. It’s much ……………. than my
Q.4. This shirt is (A) /comparatively better (B) /than that
friend’s cat.
(C) /we saw in corner shop yesterday (D).
Q.2. Here is Neelam. She’s six years old. Her brother is
nine, so he is ……………….. Q.5. Everyone was surprised to note (A) /that Rahim
married a girl (B) /who was more beautiful and
Q.3. This is a difficult exercise. But the exercise with an
more tall (C) /than he (D).
asterisk (*) is the ………exercise on the worksheet.
Q.4. He has an interesting hobby, but my sister has Q.6. A lots of books (A) /on English grammar are
the…………. hobby in the world. (B) /available with me but (C) /this one is the
best(D).
Q.5. In the last holidays I read a good book, but father
gave me an even ………….one last weekend. Q.7. I told her (A) /that it would be all (B) /the more
Q.6. School is boring, but homework is …….. than better (C) /if she herself talked to the groom (D).
school. Q.8. She does not have (A) /some money to buy (B) /a
Q.7. Skateboarding is a dangerous hobby. Bungee new refrigerator (C) /so she is worried (D).
jumping is ……… than skateboarding. Q.9. Now a days (A) /the weather (B) /is getting more
Q.8. This magazine is cheap, but that one is… cold (C) /and colder (D).
Q.9. We live in a small house, but my grandparents’ Q.10. All the books (A) /were indeed interesting (B) /but
house is even ……………. than ours. that one was (C) /the more interesting (D).

60 ENGLISH LANGUAGE
stportal.mahendras.org

Explanation
Q.10. Yesterday Alok told me a joke. This joke was the
EXERCISE 1
funniest joke I’ve ever heard.
Q.1. My father is as strong as his father.
EXERCISE 3
Q.2. She is prettier than her sister.
Q.1. Heavy
Q.3. You are not as tall as your brother
Q.2. Clumsy
Q.4. That pond is the shallowest in this area
Q.3. Dirty
Q.5. That has to be the most interesting film I have seen.
Q.4. Brave
Q.6. Which University offers the best degree courses?
Q.5. Narrow
Q.7. This clown is not as funny as the other one.
Q.6. Wild
Q.8. He is easily the worst player in the team.
Q.7. Much
Q.9. The second half of the play was less interesting.
Q.8. Honest
Q.10. What is the farthest distance you have ever run?
Q.9. Blind
EXERCISE 2
Q.10. Dark
Q.1. This is a nice cat. It’s much nicer than my friend’s
cat. EXERCISE 4
Q.2. Here is Neelam. She’s six years old. Her brother Q.1.(A) Change further to farther. Farther is the comparative
is nine, so he is elder. degree of far.
Q.3. This is a difficult exercise. But the exercise with Q.2.(A) Replace smarter by smart.
an asterisk (*) is the most difficult exercise on the Q.3.(B) Delete most. Some words like unique, full, whole,
worksheet. square, universal are used as superlatives so we
Q.4. He has an interesting hobby, but my sister has the should not use most here.
most interesting hobby in the world. Q.4.(B) Write either ‘comparatively good’ or ‘better'.
Q.5. In the last holidays I read a good book, but father Q.5.(C) Replace ‘more tall’ by taller.
gave me an even better one last weekend.
Q.6.(A) Replace ‘A lots of ’ by ‘ Lots of ’ or ‘A lot of ’.
Q.6. School is boring, but homework is more boring
Q.7.(C) more better is incorrect. Avoid double comparative.
than school.
Delete more.
Q.7. Skateboarding is a dangerous hobby. Bungee
Q.8.(B) Replace some money by any money. Any is used
jumping is more dangerous than skateboarding.
in negative sentences.
Q.8. This magazine is cheap, but that one is cheaper.
Q.9.(C) Replace more cold by colder.
Q.9. We live in a small house, but my grandparents’
Q.10.(D) Replace more by most.
house is even smaller than ours.

ENGLISH LANGUAGE 61
stportal.mahendras.org

CHAPTER

10 CLOZE TEST
Scan the QR code to get video of this chapter.

A cloze test (also cloze deletion test) is an exercise, test, 4. Read the text again, trying to fill the gap as you come
or assessment consisting of a portion of text with certain to it by imagining what the correct answer should
words removed (cloze text), where the candidate is asked be.
to replace the missing words. The Cloze test measures
5. Read the text another time, this time choose the
the students' comprehensive abilities by giving them a
correct answer from the five(sometimes four)
short text, with blanks where some of the words should be
filled. This version of the test resembles the directions to answers given.
a computer game, describing the game's rules. This task 6. If you are unsure of any given answer, try reading the
requires students to build an internal representation of the sentence with each of the five possibilities.
text, to put the words together in a meaningful way, so that
7. Try to eliminate two of the obvious false answers
they will be able to interpolate (guess, based on the given
(usually two relatively false answers are given) When
information) what words belong to the blanks.
stuck between two answers, go with the one that sounds
Cloze tests require the ability to understand context and right.
vocabulary in order to identify the correct words or type of
words that belong to the blanks of the passages of a text. Tips:
1. Do not stop to answer the questions on your first reading
Here's How:
or just at the blank.
1. Read the text throughly trying to understand the general
2. Always think about the overall meaning of the text (i.e.,
meaning.
whether the text is negative, positive, etc.) to make sure
2. Look at each missing word gap and try to imagine what that your answer choice fits the context.
the correct word should be.
3. Trust your intuition. If you feel a word is right
3. Decide which part of speech (adjective, noun, gerund, instinctively, it is probably correct.
etc.) needs to be used to fill in each gap.

62 ENGLISH LANGUAGE
stportal.mahendras.org

exercise
In the following passage there are bold words each the government’s cybersecurity arm Computer Emergency
of which has been numbered, these numbers are Response Team-India (CERT-In), are now investigating the
printed below the passage and against each five words data breach. “Banks are yet to report the matter to the cyber
have been suggested, one of which fits the blanks cell, despite it being mandatory.
appropriately. Find out the appropriate word in each
Q.1. (1) created (2) formulated (3) breached
case but if the given word is correct and requires no
(4) reached (5) No change required
change then mark (5) i.e. no change required as your
answer. Q.2. (1) anonymity (2) liability (3) adapt
(4) acceptability (5) No change required
EXERCISE- 1 Q.3. (1) urgency (2) vehemently (3) maturity
The RBI has called a meeting with all stakeholders involved (4) variety (5) No change required
in the largest data breach in India’s banking system, said
Q.4. (1) besmirch (2) despite (3) of
two people with direct knowledge of the development. The
meeting will be chaired ______ (1) by a deputy governor of (4) instead (5) No change required
the central bank and will be attended by executives from banks Q.5. (1) revealing (2) simply (3) managed
and payment network service providers.  The central bank will (4) happen (5) No change required
ask all lenders to report cyber security issues on a real-time
Q.6. (1) free (2) desperate (3) savagely
basis, an RBI official, one of the two people cited above, said
(4) separately (5) No change required
on condition of fidelity_____ (2). The regulator may also ask
all banks to centralize their cyber security operations and put a Q.7. (1) path (2) view (3) location
proper team in place instead of outsourcing these functions, this (4) cite (5) No change required
person said.  The matter gains myth_____ (3) in view of the
EXERCISE- 2
large-scale data breach. The RBI official said that banks took
a month to bring this issue to its notice beside ______ (4) its In the following passage there are bold words each
latest notification on cyber security.  “It is observed that banks of which has been numbered, these numbers are
are hesitant to share cyber-incidents faced by them. However, printed below the passage and against each five words
the experience gained globally indicates that collaboration have been suggested, one of which fits the blanks
among entities in sharing the cyber-incidents and the best appropriately. Find out the appropriate word in each
practices would facilitate timely measures in containing case but if the given word is correct and requires no
cyber-risks. It is reiterated that banks need to report all unusual change then mark (5) i.e. no change required as your
cyber-security incidents to the Reserve Bank,” read the answer.
central bank’s notification issued. Article had reported that Rock-smashing monkeys make stone flakes that look a lot
it took three months for India’s banking system to become like tools made by our old ancestors. Scientists watched as
aware of the large-scale data breach. Card data of 3.2 million Capuchin monkeys in a national park pounded stones against
customers was stolen between May and July from a network each other, splitting off sharp-edged flakes that resemble
of Yes Bank Ltd ATMs happening _________ (5) by Hitachi cutting tools used by the hounds _____ (1) of humans.
Payment Services Pvt. Ltd, but it was only in September that
The monkeys ignored the flakes, focusing on the damaged
banks and payments services providers became aware of the
stones instead. So they clearly weren’t deliberately making
extent of the breach. Yes Bank and Hitachi released statements
them as tools. But if ancient monkeys did the same thing, their
denying there was a breach in their respective systems.  The
unintentional handiwork could be mistaken for deliberate
regulator has a broad sense of the issue but it is also likely to
tool-making by human ancestors, researchers said. The
mysteriously _________ (6) investigate the matter, said the
scientists are not suggesting that any stone tools attributed
first person cited above. The regulator will conduct annual
so far to human forerunners were instead made by monkeys.
cyber audits of banks starting next year.  A detailed forensic
Those tools, which date back as far as 3.3 million years ago,
report is already being conducted by SISA Information
are more complex than what the Brazilian monkeys make.
Security Pvt. Ltd, a global payments security specialist firm. 
But as scientists look with _____ (2) earlier and earlier tools,
“We are working with various stakeholders. We will also work their findings may begin to resemble the monkey flakes
with various other agencies required for this investigation. more strongly, said Proffitt, lead author of a study choose
The investigation is on track ________ (7). The necessary _____ (3) by the journal Nature. And the new work shows
results will be shared with the concerned stakeholders once that such flakes are not exclusively the calling card of our
we have completed the investigation,” said, chief executive ancient ancestors, called hominins, he said. If somebody finds
of SISA. The report is expected in November.  Multiple very old simple flakes, you can’t assume it is hominin. You
government organizations, including the cyber cell of the have to say it might be produced by an diseased ______ (4)
Mumbai Police’s crime branch, the ministry of finance and monkey or ape.

ENGLISH LANGUAGE 63
stportal.mahendras.org
Our African ancestors used sharp-edged stone flakes for doing something monotonous. If you make it interesting
butchering and skinning animal carcasses, as well as cutting for them, tell them stories, or tell them they can listen to a
up tough plant material. To show such flakes were human- song if they finish a task, then they do it."
made tools, scientists seek win _______ (5) like wear marks If a child is in a bad mood, the obvious solution
on the edges or nearby animal bones with marks from is to ask them about it, and that is what ____ (4) she also
butchering. Proffitt studied capuchin monkeys in the Serra recommends: "My students are a bit older, so I ask them what
da Capivara National Park. They examined the dearth _____ the matter is, and if something is wrong. Most of the time
(6) and damaged rocks and compared them to artifacts from they tell me, and I give them a solution. Then the matter is
human ancestors. It’s not clear why the monkeys smash rocks resolve _____ (5). Not all kids lie. Some are genuinely upset.
together, he said. That is why we have to talk to them and find out."
Scientists long thought tool-making was confined to our Q.1. (1) behave (2) chose (3) need
branch of the evolutionary family tree, the Homo group. But (4) refer (5) no change required
scientists says ______ (7) finding 3.3-millon-year-old tools
much older than any known member of Homo, last year. Q.2. (1) regards (2) garner (3) context
(4) farce (5) no change required
Q.1. (1) monkeys (2) students (3) forerunners
(4) pounded (5) no change required Q.3. (1) in (2) by (3) for
Q.2. (1) for (2) in (3) on (4) of (5) no change required
(4) to (5) no change required Q.4. (1) why (2) who (3) when
(4) which (5) no change required
Q.3. (1) show (2) happen (3) express
Q.5. (1) settled (2) grievance (3) pass
(4) released (5) no change required (4) fatal (5) no change required
Q.4. (1) loose (2) loss (3) distant
EXERCISE- 4
(4) extinct (5) no change required
In the following passage there are bold words each of
Q.5. (1) evidence (2) evident (3) demonstrate which has been numbered, these numbers are printed
(4) amalgamate (5) no change required below the passage and against each five words have been
suggested, one of which fits the blanks appropriately.
Q.6. (1) fluke (2) family (3) flakes
Find out the appropriate word in each case but if the
(4) waver (5) no change required given word is correct and requires no change then mark
Q.7. (1) tell (2) exasperated (3) reported (5) i.e. no change required as your answer.
(4) knows (5) no change required March 4, will be a turning point for Europe. On the same day
EXERCISE- 3 as an important____(1) general election in Italy, we'll find
out whether an internal referendum of the German Social
In the following passage there are bold words each Democratic Party members has framed ____ (2) a 'yes'
of which has been numbered, these numbers are for the 'grand coalition' government in Berlin, continuing
printed below the passage and against each five words their current partnership with Angela Merkel's Christian
have been suggested, one of which fits the blanks Democrats. Conventional wisdom says this would be a good
appropriately. Find out the appropriate word in each nature____ (3) for Europe. I think the conventional wisdom
case but if the given word is correct and requires no is wrong. Like putting on a medical corset to alleviate a
change then mark (5) i.e. no change required as your serious back condition, and then carrying on with your life
answer. just as before, a grand coalition ____ (4) would be good
There are two sides to every story. Adults often attribute in the short term but bad in the long. You need to address
their harsh behaviour to the fact that it is often necessary the causes, not just the epitome ____ (5) . And there is an
in order to discipline them because they tend ____ (1) to alternative.
lie or be lazy and stubborn. We asked a lady, who has been Q.1. (1) internal (2) insane (3) innocuous
teaching play-school children at Kidzee School for five
years, and is a mother of two now-grown "children" herself, (4) embodiment (5) No change required
to elaborate on this per her experience as a teacher and Q.2. (1) produced (2) says (3) epoch
parent. She said that children can lie and be manipulative (4) plopped (5) No change required
about small things. However, in the way____ (2) of the
Q.3. (1) outcomes (2) result (3) concussion
video of the little girl being abused by her mother, she
reiterated that not all children are "stubborn." Many, due (4) repercussion (5) No change required
to the burden of studies at such an early age, get genuinely Q.4. (1) situation (2) jurisdiction (3) partition
stressed or distracted like the child in the video probably
(4) petrification (5) No change required
was: "Sometimes they do get tired at ____ (3) writing or
64 ENGLISH LANGUAGE
stportal.mahendras.org
Q.5. (1) result (2) symptoms Q.6. (1) apathetic (2) dedicated (3) lazy
(3) circumstance (4) entourage (4) vigorous (5) No change required
(5) No change required Q.7. (1) reimbursement (2) penalty (3) dismissal
EXERCISE- 5 (4) remuneration (5) No change required
In the following passage there are bold words each Q.8. (1) contact (2) joint (3) seem
of which has been numbered, these numbers are (4) cut (5) No change required
printed below the passage and against each five words
Q.9. (1) mean, (2) selfish (3) generous
have been suggested, one of which fits the blanks
appropriately. Find out the appropriate word in each (4) stingy (5) No change required
case but if the given word is correct and requires no Q.10. (1) making (2) building (3) composing
change then mark (5) i.e. no change required as your
answer. (4) shaping (5) No change required

The National Pension System (NPS), which invests EXERCISE- 6


in equity, generates superior answers ____ (1) for its In the following passage there are blanks each of which
subscribers. The EPFO should confer ____ (2) the job of has been numbered, these numbers are printed below
investing equity to the NPS. There are multiple benefits. the passage and against each five words have been
The NPS has the lowest asset management fees, and suggested, one of which fits the blanks appropriately.
EPFO subscribers will get this benefit. The NPS survey Find out the appropriate word in each case.
_____ (3) to civil servants and worker leaders cannot say
We know several things about the Indian education system.
that what is good for civil servants is not good enough
First, it is extraordinarily inefficient. Public expenditures,
for workers.
regardless of whether they are adequate in aggregate, do
The corpus ____ (4) available to fund managers at the not  (1)  results. Second, Indians desire education. They
NPS will grow, conceding ____ (5) greater diversification recognize its importance, and are willing to (2) for quality.
and risk-taking. It is worth considering if EPFO and NPS Third, quality is difficult to (3), and public policy fails
should further diversify their portfolios of asset classes to to help overcome this problem. Fourth, access to quality
private equity and real estate. In which case, it would make education is very unequal.
sense for the NPS/EPFO to employ actual ____ (6) fund
By now, we do not need more studies documenting
managers for these segments with an appropriately structured
these problems, unless they go beyond proximate causes
provision _____ (7) structure. Effective oversight to prevent
and start to develop concrete solutions. We know that
insider trading is also a must. One way is to link ____ (8) the
teachers in government schools often do not show up
compensation and incentives of asset managers with sound
to (4), especially in rural areas. We know that the incentive
performance. Canada’s biggest public pension fund groups
systems in government bureaucracy, extending beyond
pay a basic salary, an attractive annual bonus and a miserly
the ministry of human resource development (HRD),
_____ (9) long-term performance bonus that accounts for
are major (5) to the dysfunctions of the Indian education
the largest component of the payout. The remuneration
system. Under the  (6)  of maintaining standards and
structure drives the asset managers to take a long-term view
combating inequality, the education bureaucracy makes
of investments as they benefit in forging ____ (10) the assets
it harder for private providers to fill the gaps created by
grow. A large portion of the remuneration for pension fund
government failures.
managers in India must be linked to profitability in the long
term. We also know that India has little time to  (7)  these
problems. The supposed demographic dividend is on the
Q.1. (1) retreat (2) returns (3) recompense
cusp of being a disaster, unless the education system is
(4) lottery (5) No change required fixed. This fix cannot be done in a conventional manner.
Q.2. (1) allot (2) deliver (3) trust Hiring more school teachers or professors without (8) the
entire incentive system will not work. That overhaul
(4) entrust (5) No change required
will take too long, especially for the extensive primary
Q.3. (1) furnishes (2) gratify (3) caters school system—without dealing with the formative years
(4) procures (5) No change required of education, nothing else can be fixed. There is more
scope for rapid change at the university level, especially
Q.4. (1) bulk (2) substance (3) work by bringing in talent from abroad, but even there, India’s
(4) mass (5) No change required politics will make change difficult.
Q.5. (1) allowing (2) managing (3) creating The most promising solution lies in giving Indians digital
(4) giving (5) No change required access to knowledge, and to structured learning. There is
nothing sacred about the conventional classroom. Indeed,

ENGLISH LANGUAGE 65
stportal.mahendras.org
we know that it has always been just a part of the necessary maintenance system for central government employees
learning (9) that included peers and parents. The digital to begin with, and is in the process of (5) it with police
revolution has reached the point where (10) can provide and land records and voter ID cards. Bringing both
more than passive information that has to be processed Aadhaar and Digital India under the same roof will, no
unaided by the individual to turn into knowledge. Instead, doubt, help the government effort.
there are learning games, video lessons and myriad Aadhaar (6) have already crossed 75 crore, schemes like
possibilities for interaction, that fundamentally change cash transfers for LPG have already  (7)  the Aadhaar-
the way education can take place. based subsidy delivery model, and the Jan Dhan Yojana
Q.1. (1) Achieve (2) negotiate (3) fulfill will also be based on it. Encouragingly, most states now
(4) dispatch (5) slander have also embraced Aadhaar and enrollments in 15 states
have (8) 90% already—Rajashan’s social scheme delivery
Q.2. (1) Handle (2) diminish (3) confuse platform Bhamashah and Madhya Pradesh’s Samagra
(4) pay (5) settlement are two big examples of its  (9). With the IT ministry
acting as the central point, it will be easier to integrate
Q.3. (1) Derived (2) judge (3) values
the Aadhaar-based data universe for delivery of all
(4) destined (5) finds government money related to social sector schemes. The
Q.4. (1) Advice (2) exercise (3) teach government must also now move on providing Aadhaar
the necessary statutory backing so that there is no (10) on
(4)edited (5) insulate
its usage. The government had introduced the Bill for this
Q.5. (1) Beginners (2) fans (3) hurdles in Parliament but the Parliamentary Standing Committee
(4) angel (5) contributors had struck it down at that time. It has a better chance of
getting passed now.
Q.6. (1) Posture (2) literacy (3) aspect
Q.1. (1) Lethargy (2) keen (3) vigil
(4) guise (5) presence
(4) pathetic (5) accomplish
Q.7. (1) Eased (2) fix (3) wander
Q.2.  (1)Tribute (2) intervention (3) contribute
(4) mess (5) replicate
(4) subsidies (5) advocacy
Q.8. (1)Ruining (2) sharpened (3) overhauling
Q.3. (1)Transaction (2) restoration (3) convert
(4) connected (5) overtaking
(4) legislation (5) replacement
Q.9. (1)Aura (2) barrage (3) status
Q.4.   (1)Efficiency (2) desperation (3) competent
(4) property (5) environment
(4) liberty (5) economic
Q.10. (1)Teaches (2) everyone (3) Indians
Q.5. (1)Veiling (2) endurance (3) linking
(4) technology (5) focus
(4) juggling (5) vital
EXERCISE- 7
Q.6. (1) Engagement (2) rally (3) exertion
Q.1-10.  In the following passage there are blanks each
(4) receipt (5) enrollments
of which has been numbered, these numbers are printed
below the passage and against each five words have been Q.7. (1) Alienated (2) adopted (3) intimated
suggested, one of which fits the blanks appropriately. Find (4) traced (5) shunned
out the appropriate word in each case.
Q.8.  (1)Surpassed (2) distanced (3) eclipsed
Given that the government is (1) to adopt the Aadhaar
platform to deliver  (2), it is a good idea to bring the (4) overcome (5) ranked
Unique Identification and Development Authority of Q.9. (1)Realize (2) descend (3) confession
India (UIDAI) under the ministry of communications (4) acceptance (5) comply
and information technology. The fact that the UIDAI
was placed under the Planning Commission could be Q.10.  (1)Follow (2) alter (3) discussed
one of the reasons why it faced the kind of problems (4) controversy (5) concurrence
it did. With Planning Commission now scrapped and
its (3) NITI Aayog primarily a think tank, UIDAI would, EXERCISE- 8
in any case, need to be put under an administrative Q.1-10.  In the following passage there are blanks each
ministry. While the ministry is also the administrative of which has been numbered, these numbers are printed
ministry for the Digital India program, it has also below the passage and against each five words have been
succeeded in displaying the  (4)  of Aadhaar-based suggested, one of which fits the blanks appropriately. Find
identification model by developing the attendance out the appropriate word in each case.

66 ENGLISH LANGUAGE
stportal.mahendras.org
The ambient air quality in Indian cities has (1) to dangerous Q.5. (1) Cause (2) function (3) matter
levels over the last two decades. The people are  (2)  to (4) make (5) aim
extreme health risks due to increasing particulate matter,
hazardous airborne agents in indoor spaces, and emission Q.6. (1) Ordinary (2) makeshift (3) frequent
of noxious gases from industries and automobiles and (4) various (5) monotonous
open sewerage systems. Outdoor air quality is affecting
Q.7. (1)Economically (2) timidly (3) awfully
the indoor air quality too.
(4) periodically (5) haphazardly
Delhi, Patna, Gwalior Raipur and Ahmedabad have been rated
as the most polluted cities in India as far as air pollution is Q.8. (1) Employ (2) remove (3) inject
concerned. The Word Health Organisation (WHO) designates (4) shed (5) amputate
any air that carries more than 10 microgram (i.e., a millionth
Q.9. (1) Repudiate (2) downsize (3) adopt
of a gram) of pollutants in a cubic metre of air as (3).
(4) tolerate (5) seize
The cities mentioned above show presence of (4) 10 to 15
times more than the minimum. Compare them with Fresno, Q.10. (1)Fought (2) regularize
Riverside (both in Canada), Los Angeles-Long Beach, (3) transformed (4) sought
Hanford-Corcoran (both in California) and Fairbanks (Alaska)
as the most polluted cities in the North American continent (5) Leapt
where presence of pollutants in air is two to five times higher EXERCISE- 9
than the minimum.
Q.1-5. In the passage given below words given in bold
The main pollutants in the urban air in India are particulate are followed by a number given in the brackets
matter (i.e., dust, fine and ultrafine); industrial gases such for every word in bold five alternatives are given.
as sulphur dioxide (SO2), nitrogen dioxide (NO2) and Find out the word which best fits the place. If the
Ozone (O3); chullahs and open fire cooking; and waste. given word already fits in the blank, mark (5) as
These airborne agents (5) respiratory diseases such as asthma, the answer.
acute bronchitis, allergy and irritation of the respiratory tract, Quintessential (1) as one of India’s most luxurious
and can even lead to cancer of lung and bladder. trains, the Deccan Odyssey gives its guests a first-
Environmental engineers therefore advise building owners hand experience of the country’s most abhorrent
to opt for balanced air filters and ventilation systems, (2) cultural and historical highlights. Each of its six
dehumidifiers and appropriate solar shading. Non- routes has been specially designed to take you across
operational fans, pumps, fan coil units are to be removed India’s diverse locales that radiate timeless traditions.
and  (6)  servicing of H-VAC (heating, ventilation and Get ready for an exceptional journey through the
cooling systems) has to be taken up. heart of a vibrant country as you enjoy Royalty On
Rails. The train journeys showcase the austere (3)
Air-conditioning ducts have to be cleaned  (7). There
of palaces, culinary wonders of cities and villages.
are companies that deploy robots to penetrate deep
and (8) cobwebs and dust from these ducts. Ripe with spirit (4) and style, each luxury journey
is an once-in-a-lifetime experience. The Deccan
Building owners are advised to  (9)  a holistic approach
Odyssey comprises of 21 royal coaches with 4
towards IAQ maintenance. Key IAQ measurements
spacious cabins each and 4 resplendent (5) suites.
(e.g., temperature, RH, velocity, particulate matter, CO2,
SO2, NO2, O3) have to be obtained. Technical review of Q.1. Choose the correct option for (1)
ventilation and cooling system must be (10) and targets (1) Applauded (2) Appreciative
should be set in sync with professional advice.
(3) Lauded (4) Rambled
Q.1. (1) Affiliated (2) chased (3) indulged
(5) No correction required.
(4) tormented (5) degraded
Q.2. Choose the correct option for (2)
Q.2. (1)Evident (2) veiled (3) defined
(1) Obnoxious (2) Invidious
(4) sheltered (5) exposed
(3) Enthralling (4) Fascinating
Q.3. (1)Vestige (2) hazardous (3) predictable
(5) No correction required.
(4) omnipresent (5) operational
Q.3. Choose the correct option for (3)
Q.4. (1) Stability (2) contaminated
(1) Opulence (2) Deep pockets
(3) pollutants (4) quality
(3) Grandiose (4) Spartan
(5) pressure
(5) No correction required.

ENGLISH LANGUAGE 67
stportal.mahendras.org
Q.4. Choose the correct option for (4) (3) Active, acquiescence
(1) Quest (2) Expedition (4) Thought, assent
(3) Adventure (4) Exploration (5) Dealt, predicament
(5) No correction required Q.7. Choose the correct option for (3) & (4)
Q.5. Choose the correct option for (5) (1) Apparently, spark
(1) Caliginous (2) All the rage (2) Indeed, boost
(3) Drab (4) Refulgent (3) Dubiously, catalyzer
(5) No correction required. (4) Absolute, hindrance
Q.6-9. In the passage given below there are 5 blanks. (5) Hardly, restraint
Every blank has four alternative each containing Q.8. Choose the correct option for (5) & (6)
two words (1), (2), (3) and (4).Choose the option
which will best suit the respective blank and (1) Meagre, coverage
give it complete meaning to the sentences in the (2) Deliberate, mission
passage. Mark (5) as your answer if the work
(3) Pace, recover
given in bold after the blank is your answer i.e.
“No change required.” (4) Abundant, protection
As far as the vaccination programme is ____________ (5) Bounteous, plan
(1), India faces a threefold _____________ (2): Low Q.9. Choose the correct option for (7) & (8)
full immunization coverage (65 per cent), limited
(1) Slowdown, named
basket of vaccines and, issues regarding quality and
logistics of vaccine management for such a vast and (2) Accelerate, valued
diverse country. It is __________ (3) heartening to (3) Dawdle, dedicated
know that the government has taken multiple steps
to _________ (4) the scope of immunization. (4) Hasten, christened

India’s full immunization coverage (FIC), which (5) Retard, used


used to be 61 per cent in 2009, improved to 65 per Q10-14. In the passage some blanks are given and
cent in 2013 at a ___________ (5) increase rate against each blank five alternatives consisting
of 1 per cent per year. It was then realized that of a pair of words are suggested. Choose the
with the prevailing 1 per cent annual increase in correct alternative to complete the passage in a
immunization, it would take a long time to cover meaningful way.
the whole country. At that tardy pace, India would Journalism originated as the reportage of current
have taken 25 years more to achieve 90 per cent full events, specifically in printed form, i.e. newspapers.
immunization ____________ (6). However, ____ (1) ____ by the massive growth in
To __________ (7) the full coverage to at least technology and electronic communication over the
90 per cent till 2020, the Ministry of Health & past decade, its scope has expanded and its definition
Family Welfare launched Mission Indradhanush has evolved into an all-encompassing domain of
(_____________ (8) after the seven colors of the information. The current stage on that evolutionary
rainbow, termed as Indradhanush in Hindi) in ladder — following print, radio, and television — is
2014. Under this, seven vaccines would be given digital journalism, the most ___ (2) ___ used and
to all those children and pregnant women who popular driver of news consumption in today’s
have missed out or are left out under the routine world.
immunization rounds. It would cover all far-flung As of now, as many as eighty percent of Indians
areas. MI has used the Annual Health Survey consume news on the Internet, primarily through
(2011-12), District Level Health Survey, Coverage the social media and instant-messaging services. The
Evaluation study, Rapid Survey on Children large-scale use of services like Facebook, Twitter,
(RSOC), Integrated Child Health and Immunization Instagram, LinkedIn, and YouTube has ____(3)____
Survey (INCHIS) data and risk analysis methods to news organizations to offer regular updates to
identify and monitor the progress in the initial 201 consumers on current events, bypassing barriers of
high-focus districts. time and accessibility. In fact, advancements made
Q.6. Choose the correct option for (1) & (2) in chat-bots have also made it ____ (4) _____ to
(1) Concerned, challenges deliver curated news, stock market updates, and
other relevant information, tailored specifically
(2) Considerate, policies
68 ENGLISH LANGUAGE
stportal.mahendras.org
for the individual consumer, on WhatsApp and (1) Entitled, Allowed
Messenger. (2) Ratified, accredited
Artificial Intelligence has played a key role in ____ (3) Inhibited, constrained
(5) _____ the emerging potential of technology in
automating news delivery on digital mediums. (4) Both (A) & (B)
Q.10. Choose the correct option for (1) (5) All (A), (B) & (C)
(1) Dissuaded, thwarted Q.13. Choose the correct option for (4)
(2) Propelled, impelled (1) Viable, forlorn
(3) Impelled, dissuaded (2) Abject, doable
(4) Both (A) & (B) (3) Feasible, possible
(5) All (A), (B) & (C) (4) Both (A) & (B)
Q.11. Choose the correct option for (2) (5) All (A), (B) & (C)
(1) Widely, concisely Q.14. Choose the correct option for (5)
(2) Expansively, exiguously (1) Accentuating, Highlighting
(3) Immensely, enormously (2) Emphasizing, deflecting
(4) Both (B) & (C) (3) Estimating, surmising
(5) All (A), (B) & (C) (4) Both (A) & (B)
Q.12. Choose the correct option for (3) (5) All (A), (B) & (C)

ENGLISH LANGUAGE 69
stportal.mahendras.org

Explanation
Q.7.(4) Q.8.(5) Q.9.(3)
EXERCISE 1
Q.10.(1)
Q.1.(5) Q.2.(1)
EXERCISE 6
Q.3.(1) For other options
Q.1.(1) Achieve
Vehemently(Adv)- in a forceful, passionate, or
intense manner. Q.2.(4) Pay
Q.4.(2) For other options Q.3.(2) Judge
Besmirch(V)-Sully Q.4.(3) Teach
Q.5.(3) Managed Q.5.(5) Contributors
Q.6.(4) For other options Q.6.(4) Guise
Savagely(Adv.)- in a fierce, violent, and Q.7.(2) Fix
uncontrolled manner. Q.8.(3) Overhauling
Q.7.(5) Q.9.(5) Environment
EXERCISE 2 Q.10.(4) Technology
Q.1.(3) Q.2.(1) EXERCISE 7
Q.3.(4) Q.4.(4) Q.1.(2) Keen
Q.5.(1) For other options Q.2.(4) Subsidies
Demonstrate(V)-reveal Q.3.(5) Replacement
Amalgamate(V)-Combine Q.4.(1) Efficiency
Q.6.(3) For other options Q.5.(3) Linking
Fluke(N)-chance Q.6.(5) Enrollments
Q.7.(3) Q.7.(2) Adopted
EXERCISE 3 Q.8.(1) Surpassed
Q.1.(5) Q.9.(4) Acceptance
Q.2.(3) For other options Q.10.(4) Controversy
Garner(V)-gather EXERCISE 8
Farce(N)-absurdity
Q.1.(5) degraded
Q.3.(4) Q.4.(5) Q.5.(1)
EXERCISE 4 Q.2.(5) exposed
Q.1.(5) For other options: Q.3.(2) hazardous
Innocuous(Adj)- harmless Q.4.(3) Pollutants
embodiment(N)-symbol Q.5.(1) Cause
Q.2.(1) For other options:
Q.6.(3) Frequent
Epoch(N)-era
plop(V)-fall with a plop Q.7.(4) Periodically
Q.3.(2) For other options: Q.8.(2) Remove
Concussion(N)- temporary unconsciousness Q.9.(3) Adopt
Repercussion(N)-consequence Q.10.(4) Sought
Q.4.(5) EXERCISE 9
Q.5.(2) For other options:
Q.1.(3) lauded-highest honor
Epitome(N)-personification
Entourage (N)-retinue Quintessential- representing the most perfect
or typical example of a quality or class.
EXERCISE 5
Q.1.(2) Q.2.(4) Q.3.(3) Q.2.(4) Fascinating- extremely interesting
Q.4.(5) Q.5.(1) Q.6.(2) Obnoxious- Extremely pleasant

70 ENGLISH LANGUAGE
stportal.mahendras.org
Invidious- unpleasant Dissuade, thwart (v): persuade (someone) not
Enthralling- Holding one’s attention. to take a particular course of action

Q.3.(1) Opulence- great wealth or luxuriousness. Q.11.(3) Immense, enormous (adj.): extremely large or
great, especially in scale or degree.
For other options- deep pockets: wealthy
For other options:
Spartan: lack of comfort or luxury
Expansively (adv.) : in a extremely large or great
Grandiose-magnificent or monumental way
Q.4.(3) For other options-quest- a long search for some- Exiguously (adv.) : in very small or concise way.
thing
Concisely (adv.): in short but comprehensive
Expedition- a journey for a particular purpose way
Exploration- the action of roaming an unknown Q.12.(4) Entitle, Allow, Ratify, accredit (v): to give
area. authority or consent.
Q.5.(5) Resplendent- having a very bright or beautiful For other options:
appearance.
Inhibit, constrain (v): To restrict, prohibit or
For other options-caliginous –dark, dim or check.
misty.
Q.13.(2) For other options:
All the rage- to be very popular at a particular
time. Viable (adj.): possible

Drab- lacking brightness or interest Forlorn (adj.): pitifully sad and abandoned or
lonely.
Refulgent – shining very brightly
Abject (adj.): present to the maximum degree.
Q.6.(1) Q.7.(2) Q.8.(1)
Q.14.(1) For other options:
Q.9.(4)
Deflect (v): cause (something) to change direc-
Q.10.(2) Propel, impel (v): drive or push something tion; turn aside from a straight course.
forwards.
Surmise (v): suppose that something is true
For other options:
without having evidence to confirm it.

ENGLISH LANGUAGE 71
stportal.mahendras.org

CHAPTER

11 ARTICLES
Scan the QR code to get video of this chapter.

Definition Before a word which begins


An article is a limiting word, with a silent ‘h’. Example: an
not descriptive, which cannot hour, an honest man, an heir,
be used alone, but always joins an honour, an honourable
to a substantive word to denote man, etc
a particular thing, or a group or Before a singular, countable
class of things, or any individual noun which begins with a vowel
of a group or class. or silent ‘h’. Example: an orange
Kinds. ‘The’ is used:
Articles are either definite or indefinite. “I want the red apple.”
The is the definite article, since it points out a particular In the above sentence we find the article “the.” It shows us
individual, or group, or class. that the speaker wants a specific apple.
An or a is the indefinite article, because it refers to any one 1. When the same thing or person is mentioned again, that
of a group or class of things. is, a particular thing or person. Example: I bought an
‘A’ is used: orange. The orange is sweet..

“I need a chair.” 2. When there is only one such thing. Example: the earth,
the sun, the moon.
In the above sentence we find
the article “a”. It shows us 3. Before the names of famous buildings, etc. Example:
that the speaker does not need The Eiffel Tower, The Great Wall of China.
a specific chair. He can have 4. When a singular noun is used to point out a whole
any chair class, race, group, etc. Example: The bear is a strong
Before a word which begins animal.
with a consonant sound. 5. Before the special names of rivers, seas, oceans, mountain
Example: a woman ranges, groups of islands, certain organizations, political
Before a singular, countable parties, and countries such as the U.S.A., the U.K., the
noun. Example: a banana U.S.S.R. and the U.A.E., The Nile, The Dead Sea, The
Pacific Ocean, The Himalayas, The United Nations,
When we mention something The Republican Party, etc.
for the first time. Example: I
saw a dog. 6. Before the names of holy or important books. Example:
The Quran, The Bible.
Before a word with a long
sound of ‘u’. Example: a university, a uniform, a useful 7. Before an adjective when the noun is understood.
book, a European, etc. Example: The poor need help.

Before the word one. Example: a one-way street, a one-eyed Articles are not used
man, a one-year course, a one-day holiday, etc 1. Before proper nouns: Example: I am a fan of Sachin
‘An’ is used: Tendulkar. (not A or The Sachin Tendulkar)
I want an apple.” 2. Before the name of a place, town, country, street, or
road. Example: Bangalore is a beautiful city. (not A or
In the above sentence we find the article “an.” It shows us
The Bangalore)
that the speaker does not want a specific apple. He can have
any apple. 3. Before names of materials used in a general sense.
Example: Gold is found in Australia. (not A or The gold)
Before a noun which begins with a vowel sound. Example:
an apple. 4. Before abstract nouns used in a general sense. Example:
We love beauty. (not a beauty or the beauty)

72 ENGLISH LANGUAGE
stportal.mahendras.org

Difference in Meaning with Use of Articles What is the difference between “a” and “the”?
It helps to observe the following rules: “The” is used to talk about specific or known things. These
When we use two or more adjectives to describe the same are usually things that have been mentioned before or that
person or thing, we use the article only before the first the listener is familiar with.
adjective. ”A” (or “an”) is used to talk about things which are not
Example: specific. These are usually things that haven’t been mentioned
before or that the listener is unfamiliar with.
He talked to a tall and blonde woman. (He talked to a (one)
woman who was tall and blonde.) Let’s say I tell you: “I went to see a doctor last week.”

When we use two or more adjectives to refer to more than Explanation: I went to see some doctor. I didn’t mention him
one person or thing, we use the article before each adjective. before, and you are not familiar with him. Another option
is that it is not important who he is. So I use the word “a”.
Example:
Then I say: “The doctor said I should get more rest.”
He talked to a tall and a blonde woman. (He talked to two
women, one of which was tall and the other was a blonde.) Now you already know which doctor I refer to. I refer to the
doctor I went to see. So I use the word “the.”
What is the difference between “a” and “an”?
Next I tell you: “Do you remember the movie we watched
“A” and “an” have the same meaning. together?”
We use “a” before a consonant sound. I use the word “the” because I mention a specific movie – you
Examples: know which movie I am talking.
a dog a building a country a professor Then I tell you: “Well, I’ve seen a better movie since!”
a university You are not familiar with the new movie, so I use the word “a”.
We use “an” before a vowel sound. I also tell you: “A man on the street stepped on my foot.”
Examples: You don’t know who exactly stepped on my foot, I’ve never
an apple an umbrella an eye an hour mentioned him before. So I use the word “a”.
Note : We use “a” and “an” only before a singular noun. We Finally, I tell you: “I am a singer.”
can’t use “a” and “an” before a plural noun. This sentence simply means that I sing for a living. If I would
Examples: say: “I am the singer” you would understand that I am that
singer that was mentioned before.
Correct: a car. Incorrect: a cars.
For example:
Correct: an orange.
- “Wow! I heard a great singer is coming to town.”
Incorrect: an oranges.
- “Oh, it’s me. I am the great singer.”
An = a, e, i, o, u (sounds)
A = b, c, d, ....z (sounds)
This is an apple. This is a banana. This is a coat.

This is a dress. This is an egg. This is an ice cream.

This is an orange. This is an umbrella.

A/An or The

This is a house. This is the house where I live.

This is a bed. This is the bed in my bedroom.

ENGLISH LANGUAGE 73
stportal.mahendras.org
This is a dress.
Now that we understand the general idea of English grammar articles, here are some more specific details:
Specific Uses of English Grammar Articles
In this case Example Sentence
Use “a” / “an” You mention something I have a problem.
for the first time.
You want to say that something This is a table.
belongs to a certain group.
You want to say that someone She is a designer.

belongs to a certain group.


I’ve built a strong ship.
You want to say that something
is that kind of thing.
He is a nice guy.
You want to say that someone
is that kind of person.
Use “the” You talk about a specific thing. The feeling I got was very strange. I passed the test!
It is clear which thing you are
talking about. Look at the sun.
There is only one such thing.
No article You talk about something in general. Pigs can’t fly. Diving can be difficult.
You talk about cities, countries, We visited Mumbai.
streets, etc.

74 ENGLISH LANGUAGE
stportal.mahendras.org

exercise
Q.9. ......... dinner hosted by Ram was superb.
EXERCISE- 1
Q.10. ......... measles is ......... contagious disease.
Use ‘a’ or ‘an’ as required
Q.1. … ashtray Q.2. …. movie
EXERCISE- 5
In the following sentence, five parts i.e. (a), (b), (c), (d) and
Q.3. ……… hoot Q.4. …… appointment
(e) are given, there can be errors in any part of the sentence,
Q.5. ………. emblem Q.6. ……..thing you will have to choose the incorrect part of the sentence
Q.7. …….. envelope Q.8. …… poor and mark your answer accordingly.
Q.9. ……. investment Q.10. ……… tree Q.1. Empirical evidence for a group of six countries (a)/
strongly suggests that the use of (b)/ an underlying
EXERCISE- 2 inflation as (c)/ an indicator of trend inflation should
Q.1. ……. computer Q.2. …….. eye be avoided (d)/ no error (e)
Q.3. ………. man Q.4. ……….. umbrella Q.2. Last date for entries in athletics for the Asian Games
Q.5. ……… leg Q.6. ….. example (a)/ in Jakarta from August 18 to September 2 (b)/ is
June 30 but the AFI (c)/ had scheduled the inter-State
Q.7. ……. uproar Q.8. ……. hand Nationals in Kolkata (d)/ No Error (e)
Q.9. …. king Q.10. …….. end Q.3. A letter of credit is a (a)/ guarantee issued (b)/ by the
EXERCISE- 3 importer’s bank that it will honor(c)/ payment up to
Use ‘a’, ‘an’ or ‘the’ as required certain amount of export bills (d)/ no error (e)

Q.1. We are very surprised by… words he knows. Q.4. Ahead of Modi's visit of America, external affairs
minister said (a)/ agreement on the contentious is-
Q.2. Will you take … picture of the church ? sue (b)/ of sharing river waters including that of (c)/
Q.3. They have… loveliest garden I have ever seen. Eminem would not be on the agenda (d)/ no error (e)
Q.4. It's really… interesting piece of news, don't you think Q.5. India was among first nations / (a) having the high-
so ? est quota with IMF and due to this status/ (b) India
was allotted / c) a permanent place in the Executive
Q.5. That's… incredible story !
Board of Directors / d) No error. (e)
Q.6. They were hidden in this village till…. end of the
Q.6. The textile industry in Indian economy / (a) is mani-
war.
fested in its significant contribution to / b) industrial
Q.7. He gave me … excellent piece of advice. production, employment generation, / c) and foreign
Q.8. I have emptied the content of the bottle in … sink. exchange earnings. / d) No error (e)
Q.9. We have… theatre and several cinemas in the town. Q.7. Women riding pillion on two-wheelers / a) in the
capital have to compulsorily / b) wear helmets, ac-
Q.10. His father,… bank clerk, will be retired at the end
cording to the final notification / c) issued by Delhi
of the year.
government. / d) No error (e)
EXERCISE- 4
Q.8. The plan of the Sixth Pay Commission / (a) in
Fill in the blanks with articles where ever necessary. 2006-07 and resultant wage increase / (b) have had
Q.1. He went to ............ college to meet ......... class a positive rub-off / (c) on sales in urban markets. /
teacher. (d) No error (e)
Q.2. I will take ......... tram from ......... church. Q.9. With sharp rise in the number of large-scale financial
crimes, / (b) the CBI and Indian Institute of Manage-
Q.3. He came and sat on ......... bed.
ment have initiated / (c) a first-of-its-kind course to
Q.4. He has already gone to ......... bed. train sleuths / (d) from CBI to probe financial frauds.
Q.5. Ram was appointed ......... clerk. / (e) No error

Q.6. Suresh was declared ....... captain of our team. Q.10. Answer to our population problems  / (a) lies in edu-
cation and not in coercive and criminal sterilization
Q.7. Ramesh invited me to......... dinner yesterday. / (b) drives with unrealistic targets and / (c) difficult
Q.8. He organised ......... nice lunch in honour of......... to resist monetary incentives / (d) No error (e)
President.

ENGLISH LANGUAGE 75
stportal.mahendras.org

EXPLANATION
Q.10. His father, a bank clerk, will be retired at the end
EXERCISE 1
of the year.
Q.1. An ashtray Q.2. A movie
EXERCISE 4
Q.3. A hoot Q.4. An appointment
Q.1. the, the
Q.5. An emblem Q.6. A thing Q.2. a, the
Q.7. An envelope Q.8. A poor Q.3. the
Q.9. An investment Q.10. A tree Q.4. x
EXERCISE 2 Q.5. x
Q.1. A computer Q.2. An eye Q.6. x
Q.3. A man Q.4. An umbrella Q.7. x
Q.5. A leg Q.6. An example Q.8. a, the
Q.7. An uproar Q.8. A hand Q.9. The
Q.9. A king Q.10. An end Q.10. x, a
EXERCISE 3 EXERCISE 5
Q.1. We are very surprised by the words he knows. Q.1.(c) Remove ‘an’ before underlying
Q.2. Will you take a picture of the Church ? Q.2.(a) use ‘ the’ before ‘last’
Q.3. They have the loveliest garden I have ever seen. Q.3.(d) Insert “a” before “certain”
Q.4. It's really an interesting piece of news, don't you Q.4.(d) Insert “ the” before “Eminem”
think so ? Q.5.(a) Insert “the” before “first”
Q.5. That's an incredible story ! Q.6.(a) Insert “the” before “Indian”
Q.6. They were hidden in this village till the end of the Q.7.(d) Insert “the” before “Delhi”
war.
Q.8.(b) Insert “the” before “resultant”
Q.7. He gave me an excellent piece of advice.
Q.9.(a) Add “a” before “sharp”
Q.8. I have emptied the content of the bottle in the sink.
Q.10.(a) Add “The” before “answer”
Q.9. We have a theatre and several cinemas in the town.

76 ENGLISH LANGUAGE
stportal.mahendras.org

CHAPTER

12 ADVERBS
Scan the QR code to get video of this chapter.

The word “adverb” comes from the Latin word ‘ad’-(in I will do it later.
addition) and ‘verbum’(word). He promised to write back soon.
It is a word that adds more to the meaning of a verb, an
What are you doing tomorrow?
adjective or another adverb. (in general)
We haven’t met before.
I dreamt about you last night. (dreamt = verb; last
night=adverb) I phoned you yesterday.
The monster was incredibly ugly. (ugly=adjective; I saw her a few months ago.
incredibly=adverb) Last week I saw them walking together.
The heart patient collapsed quite suddenly. Adverb of Place —This shows
(suddenly=adverb; quite=adverb) where an action or something is done
An adverb is usually placed after the verb when it is used or happens. It answers the question
in a sentence as follows: “Where?”

He called yesterday. Such adverbs are placed after the


verb.
The train will arrive soon.
He is home.
They struggled hard to reach the top.
Examples:-
The patient is sleeping soundly.
We are here. He went home.
Unlike adjectives, adverbs do not modify nouns.
We found him outside.
CORRECT: The woman has a beautiful daughter.
(Adjective) She looked up.
INCORRECT: The woman has a beautifully daughter. I live here. / He fell down. / They are talking outside. / He
(Adverb) walked in.
CORRECT: She was still sad about it. (Adjective) We met her in the zoo.
INCORRECT: She was still sadly about it. (Adverb) They like to walk along the river.
Some words can be both adverbs and adjectives as follow: Adverb of Manner :
far, hard, and long. It is important to distinguish how they
This shows how an action or something is
are used.
done. It answers the question “How?” The
I don’t live far away from here. (Adverb) adverb is placed just after the verb when
Where I live isn’t far from here. (Adjective) it is used in a sentence.
She worked quite hard. (Adverb) They run happily.
She found the work quite hard. (Adjective) Examples:-
If we exercise regularly, we may live longer. (Adverb) He handled the situation well.
Types Of Adverbs She listened secretly to their conversation.
Adverb of Time : This shows when an action or something The children ran happily to their father.
is done or happens. It answers the question “When?” It is She cried loudly. / He drives quickly. /
placed either at the beginning or at the end of a sentence. She speaks softly.
When they are at the beginning, they are often emphasized.
Adverb of Degree or Quantity : This
They are having dinner now. answers the questions, “To what degree?”
Examples :- or “How much?” It is usually placed before the adjective
and the adverb.He answered perfectly.
Let’s talk now.
ENGLISH LANGUAGE 77
stportal.mahendras.org
Examples;- indeed, by all means, no, not at all, by no means.
He is very concerned for you. Examples:-
You are totally right. I hope my parents just for once will say yes to my latest
We almost made it to the train. idea. You must have heard about the haunted house surely?

It is too dark for us to see anything. Certainly we’ll try to rid this place of the foul odour. It would
(Before adjective) indeed help if I had a bodyguard.

Last night it rained very heavily. By all means eat whatever you want, but I think you will
(Before adverb) not be able to finish all the food.

Adverb of Frequency : This answers the question “How Oh no, not another breakdown at the traffic lights ! It is not
often?” Adverbs of frequency are very important because at all certain that the match will take place.
we often use them. Examples of these adverbs include: never, It is by no means easy that we will finish it soon.
rarely, seldom, hardly ever, occasionally, sometimes, Interrogative Adverb (Question): When? Where? How?
generally, usually, frequently, nearly always, often and Why? How much/often?
ever. Also included are: quite, just, already, almost and Examples:-
nearly. I will never do that!
When was the last time you saw the accused?
Examples:-
Where have you been all the while?
I always brush my teeth after a meal.
How could you have overlooked all these mistakes?
We often meet and chat.
Why do you have to do such a stupid thing?
He is usually here on time.
He will never have finished in time.
Relative Adverb:
when, where, how, why these words are the same in form
Jyoti is rarely late for work.
as Interrogative Adverbs; but they are not questions.
Praveen seldom reads the Bible.
Examples:-
Does he ever come to play chess ?
The time when he arrived is still unknown.
Adverbs of duration: The scene where the accident occurred is close to the
Adverbs of duration show us the hospital.
length of the action or state. He is the only one who knows how to do it.
They answer the following question: Nobody knows why he left in such a hurry.
For how long? He will forever be her In many cases (but not always!) adverbs have the
little boy. following form:
Examples:- Adjective + “-ly”
He works there temporarily. Examples:
We spoke briefly. Quick + ly = quickly Strange + ly
I will be forever grateful. = strangely
The world is constantly changing Dead + ly = deadly Sudden + ly
Adverbs of probability- = suddenly
Adverbs of probability show us the chances for the action or Clever + ly = cleverly Brave + ly
state to happen. They answer the following questions:
= bravely
How likely? He is probably in trouble.
Real + ly = really
Examples:-
When an adjective ends with “y” replace the “y” with
She will certainly forget about it. an “i”:
Maybe we’ll come after all. Heavy + ly = heavi + ly = heavily
It will probably not work. Happy + ly = happi + ly = happily
Surely you are not serious! When the adjective ends with an “e” drop the “e”:
Affirmative Adverb (yes) and Adverb of negation (No) True + ly = tru + ly = truly
- examples of this adverb includes: yes, surely, certainly,
78 ENGLISH LANGUAGE
stportal.mahendras.org
An easy way to identify adverbs, or to distinguish them from She speaks English well. (NOT She speaks well English.)
adjectives, is to look at the ending. Most adverbs are formed He walked slowly.
by adding - ly to the adjective, such as: He worked quickly.
Adverbs of place (e.g. here, there, everywhere, nowhere,
Comparative adverbs on the roof etc.) and adverbs of time (now, then, today,
“Comparative” means “comparing something to something tomorrow, next week etc.) are usually placed after the verb
else.” or after the object if there is one.
Comparative adverbs show us which action or state is better, I saw him yesterday.
worse, stronger, weaker, and so forth. I looked everywhere but couldn’t find anything.
Examples: Hang the picture there.
more, less, better, worse, faster, slower, farther, They are coming next week.
closer.
When two or more adverbs modify the same verb, they
She writes faster than most people. usually come in the following order: adverbs of manner,
Example sentences: adverbs of place, adverbs of time.
Madhu works out more seriously than Divya. We will go there tomorrow evening.
She eats less than her friends. He performed well at the concert last night.
You are better than this. Adverbs of frequency which answer the question ‘how
We couldn’t go slower even if we wanted to. often’ (e.g. always, often, rarely, frequently etc.) and some
other adverbs like hardly, almost, nearly, just, quite etc.,
Let’s gets closer.
are normally put between the subject and the verb if the verb
Superlative adverbs consists of only one word. If there is more than one word in
the verb, the adverb comes after the first word.
“Superlative” means “of the highest degree.”
You never visit us.
Superlative adverbs show us which action or state is the best,
the strongest, and so forth. I have always wanted to be a writer.
Examples: I have often told him to mend his ways.
Best, most, least, worst, strongest, fastest, slowest. We usually have breakfast at night.
They like each other best. If the verb is a form of be (is/am/are/was/were) these adverbs
are placed after the verb.
Example sentences:
I am never late for office.
He knows best.
We are just off.
It was the most boring experience.
Adverbs are usually placed before the auxiliaries have to
He shouted the strongest so he won.
and used to.
He ran the slowest so he lost.
He always used to agree with me.
Position of Adverbs I often have to wake up early in the morning.
Position Adverbs of manner which answer the question An adverb modifying an adjective or another adverb normally
‘how?’ normally come immediately after the verb or after comes before the word it modifies.
the object if there is one.
She was quite tired.
It is raining heavily.
He is a rather lazy boy.
She combed her hair gently. (NOT She combed gently her
hair.)

ENGLISH LANGUAGE 79
stportal.mahendras.org

exercise
Q.8. He finished. (late, often)
EXERCISE- 1
Q.9. We reached the station. (quickly, consequently)
Rewrite each of the following sentences, placing the adverb
of frequency given in brackets in the middle position of the Q.10. You speak. (loudly, never)
main clause.
EXERCISE- 3
For example:
Frame the following sentences in the correct order by placing
## Have you visited New York? (ever) the adverbs and adverb phrases given in brackets, at the end
## Have you ever visited New York? position of each sentence.
## I do not go to the library on the weekend. (always) For example:

## I do not always go to the library on the weekend. ## He lived. (for six years, happily, in Copenhagen)

Q.1. He did not arrive on time. (ever) ## He lived happily in Copenhagen for six years.

Q.2. Do you visit Boston? (often) ## They returned. (from Holland, last week, unexpectedly

Q.3. Are they surprised at the results? (frequently) ## They returned from Holland unexpectedly last week.
Q.1. They stood. (at the bus stop, for twenty minutes,
Q.4. The children do not follow our instructions. (always)
patiently)
Q.5. Do you wonder what will happen next? (sometimes)
Q.2. We arrived. (here, last night, on foot)
Q.6. Did they find the missing information? (ever)
Q.3. The young child walked. (by herself, this morning, to
Q.7. We do not stay out after dark. (usually) school)
Q.8. The facts are not known. (generally) Q.4. They were waiting. (at seven o’clock, eagerly, outside
Q.9. I want to analyze the book. (carefully) the fairgrounds)

Q.10. We arrived. (early) Q.5. She arrived. (in a black limousine, at the hotel)

EXERCISE- 2 Q.6. Chickadees build their nests. (in dense evergreens, in


the early spring, secretively)
In each of the following sentences, place the adverbs
Q.7. The waves crashed. (against the shore, loudly)
given in brackets in their most usual positions in the
sentence. Place connecting adverbs in the starting Q.8. I walked. (in the rain, to work, yesterday)
position, place adverbs of frequency in the middle Q.9. He sat. (until the announcements were finished, on
position, and place adverbs of manner and adverbs of time the edge of his chair, expectantly)
in the end position. Adverbs of manner should precede
Q.10. We left. (this morning, home, in a hurry)
adverbs of time.
For example:
EXERCISE- 4
Fill in the blanks with the correct form of the words given
## They left. (early, usually)
in brackets.
## They usually left early.
Example: Prem works ______ (slow).
## We proceeded. (cautiously, therefore)
Answer: Prem works slowly.
## Therefore, we proceeded cautiously.
Q.1. He ––––reads a book. (quick)
## We will review our options. (tomorrow, carefully)
Q.2. Mandeep is a––––– girl. (pretty)
## We will review our options carefully tomorrow.
Q.3. The class is ––––––loud today. (terrible)
Q.1. We pick the flowers. (carefully, usually)
Q.4. Maya is a ––––singer. (good)
Q.2. She answers. (correctly, rarely)
Q.5. You can ––––open this tin. (easy)
Q.3. He is wrong. (however, seldom)
Q.6. It’s a ––––––day, today. (terrible)
Q.4. We will attend the concert. (therefore, tonight)
Q.7. She sings the song–––––. (good)
Q.5. We found the hotel. (easily, nevertheless)
Q.8. He is a –––––driver. (careful)
Q.6. They left. (quietly, this morning)
Q.9. He drives the car –––––. (careful)
Q.7. She wins first prize. (always, furthermore)
Q.10. The dog barks ––––. (loud)

80 ENGLISH LANGUAGE
stportal.mahendras.org
Q.8. They guessed what was about to happen. (little)
EXERCISE- 5
For each of the following sentences, add the negative Q.9. I am entirely satisfied with my situation. (seldom)
expression given in brackets at the beginning of the sentence, Q.10. One comprehends a complex situation immediately.
and make any other changes that are necessary. (rarely)
For example:-
EXERCISE- 6
## I had reached home when I remembered the message.
Read the words in bold letter mentioned in below given
(hardly)
sentences carefully and rectify the sentences.
## Hardly had I reached home when I remembered the
Q.1. When my parents reached home they were too tired
message.
Q.2. The new candidates did not know hardly anyone
## We had the opportunity to do whatever we wanted.
in college and so felt lonely
(seldom)
Q.3. We rarely find something in the modern movies
## Seldom did we have the opportunity to do whatever
that is worth remembering
we wanted.
Q.4. You are supposed to have acted nobler than any
Q.1. We had entered the room when the telephone rang.
other candidate
(scarcely)
Q.5. After leaving the school we should go home
Q.2. I have seen a more beautiful ballet than that one.
directly
(never)
Q.6. He has no time to read newly published books and
Q.3. We realized that a dangerous stretch of road lay ahead no desire neither
of us. (little) Q.7. They have not seldom visited their school teachers
Q.4. I have worked as hard as I could. (never before) since they left it.
Q.5. A writer can express his exact feelings in words. Q.8. The weather was much hot yesterday hence we
(rarely) didn’t go out
Q.9. She meets him often near the railway station
Q.6. We perceive everything that is around us. (hardly
Q.10. I told her as blunt as I could but she was not
ever)
convinced
Q.7. One can find a more striking example of erosion than
the Grand Canyon. (nowhere)

ENGLISH LANGUAGE 81
stportal.mahendras.org

EXPLANATION
EXERCISE 1 EXERCISE 4
Q.1. He did not ever arrive on time. Q.1. He quickly reads a book.
Q.2. Do you often visit Boston? Q.2. Mandeep is a pretty girl.
Q.3. Are they frequently surprised at the results? Q.3. The class is terribly loud today.
Q.4. The children do not always follow our instructions. Q.4. Maya is a good singer.
Q.5. Do you sometimes wonder what will happen next? Q.5. You can easily open this tin.
Q.6. Did they ever find the missing information? Q.6. It’s a terrible day today.
Q.7. We do not usually stay out after dark. Q.7. She sings the song well.
Q.8. The facts are not generally known. Q.8. He is a careful driver.
Q.9. I want to analyze the book carefully. Q.9. He drives the car carefully.
Q.10. We arrived early. Q.10. The dog barks loudly.
EXERCISE 2 EXERCISE 5
Q.1. We usually pick the flowers carefully. Q.1. Scarcely had we entered the room when the
Q.2. She rarely answers correctly. telephone rang.

Q.3. However, he is seldom wrong. Q.2. Never have I seen a more beautiful ballet than that
one.
Q.4. Therefore, we will attend the concert tonight.
Q.3. Little did we realize that a dangerous stretch of road
Q.5. Nevertheless, we found the hotel easily. lay ahead of us.
Q.6. They left quietly this morning. Q.4. Never before have I worked as hard as I could.
Q.7. Furthermore, she always wins first prize. Q.5. Rarely can a writer express his exact feelings in
Q.8. He often finished late. words.
Q.9. Consequently, we reached the station quickly. Q.6. Hardly ever do we perceive everything that is around
us.
Q.10. You never speak loudly.
Q.7. Nowhere can one find a more striking example of
EXERCISE 3 erosion than the Grand Canyon.
Q.1. They stood patiently at the bus stop for twenty Q.8. Little did they guess what was about to happen.
minutes.
Q.9. Seldom am I entirely satisfied with my situation.
Q.2. We arrived here on foot last night.
Q.10. Rarely does one comprehend a complex situation
Q.3. The young child walked to school by herself this immediately.
morning.
Q.4. They were waiting eagerly outside the fairgrounds
EXERCISE 6
at seven o’clock. Q.1. Replace ‘too’ by ‘very’
Q.5. She arrived at the hotel in a black limousine. Q.2. Replace ‘did not know hardly’ by ‘hardly knew’
Q.3. Replace ‘something’ by ‘anything’
Q.6. Chickadees build their nests secretively in dense
evergreens in the early spring. Q.4. Replace ‘nobler’ by ‘more nobly’
Q.5. Replace ‘directly’ by ‘direct’
Q.7. The waves crashed loudly against the shore.
Q.6. Replace ‘neither’ by ‘either’
Q.8. I walked to work in the rain yesterday.
Q.7. Delete ‘not’
Q.9. He sat expectantly on the edge of his chair until the Q.8. Replace ‘much’ by ‘very’
announcements were finished.
Q.9. Replace ‘meets him often’ by ‘often meets him’
Q.10. We left home in a hurry this morning. Q.10. Replace ‘blunt’ by ‘bluntly’

82 ENGLISH LANGUAGE
stportal.mahendras.org

CHAPTER

13 ERROR DETECTION Scan the QR code to get video of this chapter.

In the following sentences, three parts i.e. (A), (B) and (C) Q.6. She said that she had chosen a selection of brand-new
are given, there can be error(s) in one or all parts of the releases,(A)/ timeless favourites and also got some
sentence, you will have to choose the incorrect part(s) of previews (B)/ for the travelers to lust over.(C)
the sentence and mark your answer accordingly. (1) Only (A) (2) Only (B)
EXERCISE- 1 (3) Both (A) and (C) (4) Only (C)
Q.1. Despite of the obvious health benefits, wearing a (5) No Error
mask can be uncomfortable,(A)/ while some users Q.7. A research suggests smaller particulate matter(A)/
complain that they restrict the amount of oxygen(B)/ including dust, dirt, soot and smoke(B)/ is respon-
which was possible to inhale in each breath.(C) sible for a most adverse health effects.(C)
(1) Only (A) and (B) (2) Only (B) (1) Only (A) (2) Only (B)
(3) Both (A) and (C) (4) Only (C) (3) Both (A) and (C) (4) Only (C)
(5) No Error (5) No Error
Q.2. Lecturers will increase contributions(A)/ and define Q.8. The most recent statistics suggests around 1.2 million
benefit pensions guarantee-(B)/ an income in retire- people in the UK(A)/ have go meat-free, with the
ment, based on final salaries.(C) food on offer for vegetarians (B)/ becoming ever-
more varied, flavourful and adventurous.(C)
(1) Only (A) (2) Only (B)
(1) Both (A) and (B) (2) Only (B)
(3) Both (A) and (C) (4) Only (C)
(3) Both (A) and (C) (4) Only (C)
(5) No Error
(5) No Error
Q.3. To have music all between your home once meant
Q.9. Those with a sweet tooth are bound to love(A)/ the
endless cables(A)/ squirrelled away under carpets
new cook book healthier peanut butter chocolate
(B)/ but now, with wireless speakers you can be chip cookies(B)/ and German chocolate cake.(C)
achieve the same end with minimal fuss.(C)
(1) Both (A) and (B) (2) Only (B)
(1) Only (A) (2) Only (B) and (C)
(3) Both (A) and (C) (4) Only (C)
(3) Both (A) and (C) (4) Only (C)
(5) No Error
(5) No Error
Q.10. Many MPs are faced allegations(A)/ of bullying after
Q.4. A huge range of new models have been release(A)/ one of the party’s frontbenchers(B)/ was accused of
for the beginning of 2016 including ones to suit the slapping a cancer survivor.(C)
increased interest in wireless headphones,(B)/ along (1) Only (A) (2) Only (B)
with some fantastic new designs.(C)
(3) Both (A) and (C) (4) Only (C)
(1) Only (A) (2) Only (B)
(5) No Error
(3) Both (A) and (C) (4) Both (A) and (B)
EXERCISE- 2
(5) No Error Q.1. The Centre government has been, for good or wrong
Q.5. Buying the right monitor for your PC is crucial,(A)/ reasons, quite concerned (A)/ with the task of labour
especially if you spend many hours in front of them laws and governance systems, primarily on the
(B)/ either because you work from home or you’re ground (B)/ that labour laws enacted in the planned
an avid gamer.(C) economic regime does not suit the market regime
of the present times(C)
(1) Only (A) (2) Only (B)
(1) Both (A) and (B) (2) Only (B)
(3) Both (A) and (C) (4) Only (C)
(3) Both (A) and (C) (4) Only (C)
(5) No Error
(5) No Error
ENGLISH LANGUAGE 83
stportal.mahendras.org
Q.2. For a decentralised cryptocurrencies, (A)/ alienating Q.9. Many people who have cut back on (A)/ sugar says
users are the greatest danger, (B)/ because exit costs that they have found their (B)/ new eating habits
in cyberspace are very low. (C) more pleasure than their old ones. (C)
(1) Both (A) and (B) (2) Only (B) (1) Both (A) and (B) (2) Only (B)
(3) Both (A) and (C) (4) Only (C) (3) Both (A) and (C) (4) Both (B) and (C)
(5) No Error (5) No Error
Q.3. The reason, most scientists don’t believe about (A)/ Q.10. Farmers in villages around the country have felt (A)/
astrology is that it is not consistent with our (B)/ demeaned and distributed by the insensitive (B)/
theories who have been tested by experiment. (C)
of successive governments at the Centre and in the
(1) Both (A) and (B) (2) Only (B) states. (C)
(3) Both (A) and (C) (4) Only (C) (1) Both (A) and (B) (2) Only (B)
(5) No Error (3) Both (A) and (C) (4) Only (C)
Q.4. The sugar industry has been going through an ex- (5) No Error
tended phase of distress because (A)/ various factors
such that failure of monsoon, varietal degeneration, Q.11. About 5000 farmers reached Mumbai, (A)/ walking
(B)/ reduced recovery, decline in areas under sugar- 165 km in the hope that their (B)/ elected representa-
cane and the resultant of sugar mill. (C) tives would listen when they speak.(C)
(1) Both (A) and (B) (2) Only (B) (1) Both (A) and (B) (2) Only (B)
(3) Both (A) and (C) (4) Only (C) (3) Both (A) and (C) (4) Only (C)
(5) No Error (5) No Error
Q.5. We have opted a multi-channel strategy (A)/ and our Q.12. When you walk into an ice cream manufacturing
promoters and sales partners are Swiss Tourism, (B)/ unit, you might except (A)/ to see hordes of workers
representatives, tour operators and travel companies. busy all day long on the production (B)/ line like
(C) mixing, churning, freezing, packaging and transport-
(1) Only (A) (2) Only (B) ing. (C)
(3) Both (A) and (C) (4) Only (C) (1) Both (A) and (B) (2) Only (B)
(5) No Error (3) Both (A) and (C) (4) Only (C)
Q.6. The CBI court, on the application (A)/ for premier (5) No Error
agency, has allowed the (B)/ police remand for the
purpose of interrogation in their custody.(C) EXERCISE- 3
(1) Both (B) and (C) (2) Only (B) In the following sentences, three parts i.e. (A),
(B) and (C) are given, there can be error(s) in
(3) Both (A) and (C) (4) Only (C) one or all parts of the sentence, you will have to
(5) No Error choose the incorrect part(s) of the sentence and
Q.7. The Lieutenant governor informed to the committee mark your answer accordingly.
that allocation (A)/ of insufficient funds to meet li- Q.1. The insects that live and feed on the cactus pear
abilities of routine maintenance, making up (B)/ of provide dye for textiles,(A)/ foods and cosmetics
deficiencies, emergency procurements will definitely while its seeds, fruits, and(A)/ stem has high levels
have a negative affect. (C) of nutrients, vitamins, minerals and antioxidants.(C)
(1) Both (A) and (B) (2) Only (B)
(1) Both (A) and (B) (2) Only (B)
(3) Both (A) and (C) (4) Only (C)
(3) Both (A) and (C) (4) Only (C)
(5) No Error
(5) No Error
Q.8. India needs to conduct a largescale genomic study
to (A)/ identify country’s specify biomarkers, which Q.2. A whale are a real gift for big acquatic hunters,(A)/
are compounds (B)/ or genefragments associated in An adult whale is several tens of tonnes that(B)/
a particular condition. (C) much bears can feed on for several months.(C)
(1) Both (A) and (B) (2) Only (B) (1) Both (A) and (B) (2) Only (B)
(3) Both (B) and (C) (4) Only (C) (3) Both (A) and (C) (4) Only (C)
(5) No Error (5) No Error

84 ENGLISH LANGUAGE
stportal.mahendras.org
Q.3. The cactus is already a well-established ingredient(A)/ Q.10. If I would have (A)/worked regularly I would have
in Latin American cuisine, where it is eaten fresh, passed (B)/the examination. No error.(C)
cooked, or pickled,(B)/ however its use as fodder is (1) Both (A) and (B) (2) Only (B)
less widespread.(C)
(3) Both (A) and (C) (4) Only (C)
(1) Both (A) and (B) (2) Only (B)
(5) No Error
(3) Both (A) and (C) (4) Only (C)
EXERCISE- 4
(5) No Error
In the following sentences, three parts i.e. (A), (B)
Q.4. As climate change brings erratic rainfall and pro- and (C) are given, there can be error(s) in one or
longed droughts,(A)/ countries should looks to all parts of the sentence, you will have to choose
the cactus pear,(B)/ which can grow in desert-like the incorrect part(s) of the sentence and mark your
conditions.(C) answer accordingly.
(1) Both (A) and (B) (2) Only (B) Q.1. The smartphones private label would be targeting
(3) Both (A) and (C) (4) Only (C) the local,(A)/ low-brand and mid-brand strength of
smartphones(B)/ which makes up almost 20% of the
(5) No Error
market.(C)
Q.5. Climate change means melting ice, where polar
(1) Both (A) and (B) (2) Only (B)
bears(A)/ mostly live(B)/ and so polar bear have to
spend longer on land.(C) (3) Both (A) and (C) (4) Only (C)

(1) Both (A) and (B) (2) Only (B) (5) No Error

(3) Both (A) and (C) (4) Only (C) Q.2. The vice-president of Amazon India said that(A)/
“We leverage the customer data and feedback to
(5) No Error understand the gaps in the selection,(B)/ pricing and
Q.6. The bears had come to feast on the carcass(A)/ of a reach out to our vendors or OEMs to manufacture
bowhead whale that wash ashore,(B)/ latter resting relevant products.”(C)
around the food source.(C) (1) Both (A) and (B) (2) Only (B)
(1) Both (A) and (B) (2) Only (B) (3) Both (A) and (C) (4) Only (C)
(3) Both (B) and (C) (4) Only (C) (5) No Error
(5) No Error Q.3. The airport serves a average of 275,000 passengers
Q.7. It is spiky, alien-looking and can be found(A)/ deco- daily,(A)/ according to its website,(B)/ near 2,500
rating homes around the world,(B)/ but experts say planes arrive and depart each day.(C)
the prickly pear cactus could help alleviate hunger (1) Both (A) and (B) (2) Only (B)
in arid regions.(C) (3) Both (A) and (C) (4) Only (C)
(1) Both (A) and (B) (2) Only (B) (5) No Error
(3) Both (A) and (C) (4) Only (C) Q.4. Traditionally, investors identification an emerging
(5) No Error technology trend,(A)/ align themselves with the
Q.8. After the brief appearence before the waiting crowds, startups best positioned to ride the wave(B)/ and cash
out when the company goes public or sells out.(C)
(A)/he was taken to the Governor chamber (B)/for
the swearing in ceremony./No error. (C) (1) Both (A) and (B) (2) Only (A)
(1) Both (A) and (B) (2) Only (B) (3) Both (A) and (C) (4) Only (C)
(3) Both (A) and (C) (4) Only (C) (5) No Error
(5) No Error Q.5. In the past, the Supreme Court has emphasised that
a married woman are a “victim” and the man(A)/
Q.9. Congress dissidents and (A)/a wide range among the
is “the author of the crime” and it has treat the
opposition has mounted a (B)/campaign to have the
exemption given to women(B)/ as a special provision
President renominated. No error. (C) that has the protection of Article 15(3).(C)
(1) Both (A) and (B) (2) Only (B) (1) Both (A) and (B) (2) Only (B)
(3) Both (A) and (C) (4) Only (C) (3) Both (A) and (C) (4) Only (C)
(5) No Error (5) No Error
ENGLISH LANGUAGE 85
stportal.mahendras.org
Q.6. Apple has appointed an expert(A)/ to lead the Indian (C) It had nothing to do with Mr. Patel's stepping
operations after its erstwhile country head (B)/ put down.
in his papers.(C) (1) Only (A) (2) Both (A) and (B)
(1) Both (A) and (B) (2) Only (B) (3) Only (C) (4) Both (B) and (C)
(3) Both (A) and (C) (4) Only (C)
(5) All of the above
(5) No Error
Q.2. (A) The incoming Governor is bound to be judged.
Q.7. Whether it’s at a interview, a presentation or simply
(B) The RBI cannot be treated as if it is just another
coffee with the team, your body language(A)/ may
government department.
decide how others perceive you. Yet most people
don’t realize that posture,(B)/ or the way you place (C) Anything less will not go down well in both the
your hands, can even set you back at work.(C) investors and the markets.
(1) Both (A) and (B) (2) Only (B) (1) Only (A) (2) Both (A) and (B)
(3) Both (A) and (C) (4) Only (A) (3) Only (C) (4) Both (B) and (C)
(5) No Error (5) All of the above
Q.8. In the salad days of bitcoin, venture capital firms Q.3. (A) There is little incentive for witnesses to turns up
bought into the craze in a roundabout way. Instead in the court.
of simply purchasing bitcoins,(A)/ they invested in (B) The need to protect witnesses has been
companies like Coinbase, which makes(B)/ a wallet emphasised by Law Commission reports.
for digital currencies which aimed to improve the
process of mining coins(C) (C) A witness protection order will be passed by a
competent authority.
(1) Both (A) and (B) (2) Only (B)
(1) Only (A) (2) Both (A) and (B)
(3) Both (A) and (C) (4) Only (C)
(3) Only (C) (4) Both (B) and (C)
(5) No Error (5) All of the above
Q.9. When it comes to investing my money,(A)/ I go for Q.4. (A) The U.S. will refrain of raising the tariff on
options that are not meant for social media,(B)/ and Chinese goods
the Victoria Beckham Collection is too refined to be (B) China will purchase agricultural and other
make an impact online.(C) goods from the U.S.
(1) Both (A) and (B) (2) Only (B) (C) If talks fail, however, increased tariff rates are
scheduled to come into force immediately.
(3) Both (A) and (C) (4) Only (C)
(1) Only (A) (2) Both (A) and (B)
(5) No Error
(3) Only (C) (4) Both (B) and (C)
Q.10. If we had to do our faces according to Instagram,(A)/ (5) All of the above
then we’d be wearing false eyelashes every day(B)/
Q.5. (A) The Economic Survey 2017-18 put social
since when did they become an essential?(C) services spending at 6.6% of GDP.
(1) Both (A) and (B) (2) Only (B) (B) The latest report on stunting and wasting should
(3) Both (A) and (C) (4) Only (C) convince the Centre.
(C) The fact that the global average of process foods
(5) No Error
scored only 31%.
EXERCISE- 5 (1) Only (A) (2) Both (A) and (B)
In the following questions, three sentences are (3) Only (C) (4) Both (B) and (C)
given i.e., (A), (B), and (C), choose which among
(5) All of the above
them is/are grammatically correct, if all are
correct choose option (5) i.e., all of the above as Q.6. (A) In the current system, there is little incentive
your answer. with witnesses.
Q.1. (A) The controversy did not affect the outcome of (B) Besides threats to their lives, they experience
the extradition proceedings. hostility and harassment while attend courts.
(B) It was a period during which the Centre and the (C) The tardy judicial process seldom takes into
RBI were engaged in an unseemly tussle. account the distance they have travelled.
86 ENGLISH LANGUAGE
stportal.mahendras.org
(1) Only (A) (2) Both (A) and (B) Q.1. The plan to bring all liquid affluent discharge
(3) Only (C) (4) Both (B) and (C) from textile units (A)/and tanneries to zero has
to be pursued vigorously, giving industries (B)/
(5) All of the above the assistance to help them choose the best
Q.7. (A) The introduction of the scheme marks a leap technologies(C)/ against the recovery of waste water
forward. for reuse. (D)/No Error (E)

(B) Until now, there have been ad hoc steps. (1) A (2) B & A (3) C
(4) A & D (5) E
(C) A few dedicated courtrooms for vulnerable
witnesses are also functional. Q.2. The decoration of the new office block (A)/ including
the furniture and curtains (B)/ are most pleasing and
(1) Only (A) (2) Both (A) and (B)
it suits the (C)/ambient of the place (D)/No Error (E)
(3) Only (C) (4) Both (B) and (C)
(1) A & D (2) B & C (3) E
(5) All of the above
(4) C & D (5) D
Q.8. (A) The astronomers have observed sunspots on the Q.3. Learning the arts and crafts don’t just foster the (A)/
surface for the sun for nearly 400 years. creative development of a child, but also assists in
(B) They find that the sun’s activity would not dip advancing and refining his (B)/ or her core skills,
during the next cycle. which in fact went towards (C)/boosting academic
achievement as well. (D)/No error (E)
(C) They expect the cycle to peak around 2024.
(1) D & B (2) B (3) A & B
(1) Only (A) (2) Both (A) and (B)
(4) E (5) C & A
(3) Only (C) (4) Both (B) and (C)
Q.4. The evidences which were present at the (A)/ court
(5) All of the above against the allegations of prosecution (B)/are the
Q.9. (A) The researchers simulate the behaviour of the fleet of liars(C)/turned the case towards them (D)/
sun using magnetic field evolution models. No error (E)

(B) They predict solar activity over next cycle, (1) A & C (2) B & D (3) C
about ten years into the future. (4) E (5) D
(C) An important reason to understand sunspots is Q.5. The Maharashtra and Union governments have sought
that they affect space weather. (A)/to defend the arrest and prosecution, contending
that the case (B)/is based on incrimination evidence
(1) Only (A) (2) Both (A) and (C) seized during the probe (C)/and has nothing to do
(3) Only (C) (4) Both (B) and (C) with the ideology or the political views of those
(5) All of the above under investigation.(D)/ No error (E)
(1) A (2) C (3) B
Q.10. (A) Mars could have had an environment with
oceans, and potentially life, much earlier than (4) E (5) D
Earth. EXERCISE- 7
(B) Some parts of Europe and North America Q.1-5. In the following questions, each of the two sentences
experienced lower-than-average temperatures. divided into two parts (A),(B) and (C),(D)
(C) Water is considered to be an essential precursor respectively is given . Any two of these parts
for life, at least as we know it. have grammatical errors. Choose the alternative
consisting of the number of grammatically wrong
(1) Only (A) (2) Both (A) and (B) parts. If both the sentences are grammatically
(3) Only (C) (4) Both (B) and (C) correct, choose option(5)i.e. No error.
(5) All of the above Q.1. i. For long, most people were under the assumption
that cigarette filters are biodegradable (A)/ as
EXERCISE- 6
they consist cellulose acetate at least in a part,
Q.1-5. Read the sentences to find out whether there is any which is itself a natural product.(B)
error in it. The error, if any, will be in one or more
than one part of the sentence. Select the part/s ii. Cellulose acetate fibres used as the predominant
which has/have an error. If the given sentence is filter (C)/ material do not ready biodegrade due
correct as it is, mark the answer as ‘No error’. to the acetyl groups on the cellulose backbone,

ENGLISH LANGUAGE 87
stportal.mahendras.org
which itself can be degraded.(D) (1) A and B (2) A and C (3) B and D
(1) A and B (2) A and C (3) B and D (4) C and D (5) No Error
(4) C and D (5) No Error Q.4. i. The program seeks innovative ideas for small
Q.2. i. Space weather is important because it can and cost-constrain missions (A) that can help
have profound impacts affecting technology unravel the mysteries of the universe and
and (A)/ astronauts in space, disrupting radio explore our place in it.(B)
communications and, at it’s most severe, ii. Four video projectors are positioned at 90 degree
overwhelming power grids.(B) increments around the sphere which acts as (C)/
ii. The new experiment will obtain global the screen and each projector is responsible
observations of an important driver of space for one quadrant of screen space and projects
(C) / weather in dynamic region of upper images seamlessly.(D)
atmosphere that can cause interference with (1) A and B (2) A and C (3) B and D
radio and GPS communications.(D) (4) C and D (5) No Error
(1) A and B (2) A and C (3) B and D Q.5. i. The proposed mission would investigate how
(4) C and D (5) No Error giant weather storms from the Sun space (A)/
Q.3. i. Recently the scientists have learned that solar call solar particle storms, are accelerated and
variability is not enough to (A)/ drive the released into planetary space.(B)
changes observed and Earth’s weather also ii. The Science on a Sphere is an educational tool
must be having an effect.(B) in which real-time planetary or space data are
ii. To help unravel that connection, the system (C)/ transformed to visual images and shown on
will investigate how do waves in the lower a suspended globe rather than a flat screen. (D)
atmosphere, (C)/ caused by variations in the (1) A and B (2) A and C (3) B and D
densities of different packets of air, impacts the (4) C and D (5) No Error
upper atmosphere.(D)

88 ENGLISH LANGUAGE
stportal.mahendras.org

EXPLANATION
Q.10.(1) In part (A)- across the country
EXERCISE 1
In part (B)- insensitivity
Q.1.(3) In part (A)- remove 'of' after despite because
despite= in spite of Q.11.(4) In part (C)- spoke
In part (C)-replace ‘was’ with ‘is’ because the Q.12.(2) In part (A)- expect
sentence is in present tense In part (C)- in mixing
Q.2.(5) All parts are grammatically sound. EXERCISE 3
Q.3.(3) In part (A)- use ‘all between’ with ‘all around’ Q.1.(4) In part C: have high levels
In part (C)-remove ‘be’ between ‘can and achieve’ Q.2.(3) In part A: A whale is
Q.4.(4) In part (A)- have been released In part C: many bears
In part (B)- since the beginning of 2016 Q.3.(5)
Q.5.(2) In part (B)-use ‘in front of it’ in place of ‘in front of Q.4.(2) In part B: Should look
them’, because the subject of reference is monitor.
Q.5.(4) In part C: so polar bears have to
Q.6.(5) All parts are grammatically sound.
Q.6.(3) In part B: that washed ashore
Q.7.(4) In part (C)- a most adverse should not be used,
In part C: later resting around
indeed use the before most, because the is used
before the superlative degree of an adjective. Q.7.(5)
Q.8.(1) In part (A)- statistics ‘suggest’ should be used, Q.8.(3) Replace ‘Governor chamber’ by ‘Governor’s
because here statistics is a plural noun, such as the chamber’. This is the question of use of apostrophe.
statistics show that the crime rate has increased. Q.9.(3) Replace ‘has’ by ‘have’. Subject ‘Congress
In part (B)- have ‘gone’ should be used because dissidents and a wide range’ is plural.
have+v3. Q.10.(1) Replace ‘would have’ by ‘had’. This is a
Q.9.(5) All parts are grammatically sound. Conditional sentence with specific structure : If
+ Subject + had + V-III + Would have +V-III
Q.10.(1) In part (A)-are facing because subject is plural.
EXERCISE 2 EXERCISE 4
Q.1.(4) Replace ‘which makes up’ with ‘which make up’
Q.1.(3) In part (A)- central
in part (C)
In part (C)- do not
Q.2.(5)
Q.2.(1) In part (A)- crypto currency
Q.3.(3) Replace ‘serves a average’ with ‘serves an
In part (B)- is the average’, in part (A).
Q.3.(3) In part (A)- believe in Replace ‘near 2,500 planes arrive’ with ‘nearly
In part (C)- which 2,500 planes arrive’ in part (C).
Q.4.(1) In part (A)- because of Q.4.(2) Replace ‘investors identification’ with ‘investors
identify’ in part (A)
In part (B)- such as
Q.5.(1) Replace ‘a married woman are’ with ‘a married
Q.5.(1) In part (A)- opted for a
woman is’ in part (A)
Q.6.(1) In part (B)- of premier
Replace ‘it has treat’ with ‘it has treated’ in part
In part (C)- its custody (B)
Q.7.(3) In part (A)- remove ‘to’ Q.6.(5)
In part (C)- effect Q.7.(4) Replace ‘at a interview’ with ‘at an interview’ in
Q.8.(3) In part (B)- specific part (A)
In part (C)-associated with Q.8.(5)
Q.9.(4) In part (B)- say Q.9.(4) Remove ‘be’ from part (C)
In part (C)- pleasurable Q.10.(5)

ENGLISH LANGUAGE 89
stportal.mahendras.org
Q.2.(4) Is most pleasing should be used and ‘ambience’
EXERCISE 5
at the place of ‘ambient’
Q.1.(4) 'did not affect' should replace "did not effect"
Q.3.(5) ‘does not’ at the place don’t and ‘goes’ in place of
Q.2.(2) ‘go down well in’ should be replaced with ‘go ‘went’ as the subject is ‘learning’
down well with’.
Q.4.(3) Were presented and Pack of liars should be used.
Q.3.(4) ‘to turns up in the court’ is wrong, to takes v1 with
Q.5.(2) Incriminating of evidence should be used as after
it so ‘to turn up in the court’ is correct.
a verb should be used after on – V(ing) form is
Q.4.(4) will refrain of is wrong because refrain takes from required.
with it.
EXERCISE 7
Q.5.(2) process foods is wrong because adjective qualifies
Q.1.(3) In part B: add ‘of’ after consist.
noun so processed food is correct.
In part D: ‘readily’ at the place of ‘ready’
Q.6.(3) ‘incentive with witnesses’ is wrong ‘incentive for
witnesses’ is correct. Q.2.(3) In part B : its should be used at the place of it’s
while attend is wrong while attending is correct In part D: add ‘a’ before ‘dynamic region’
because preposition takes gerund after it. Q.3.(4) In part C: remove ‘do’ after ‘how’
Q.7.(5) In part D: ‘impact’ should replace ‘impacts’
Q.8.(4) ‘on the surface for the sun’ is wrong. Q.4.(2) In part A: ‘cost-constrained missions’ should be
‘on the surface of the sun’ is correct. written.
Q.9.(2) ‘over next cycle’ is wrong. In part C: ‘act’ should replace ‘acts’
‘over the next cycle’ is correct. Q.5.(3) In part B: ‘called’ at the place of ‘call’
Q.10.(5) In part D: ‘transformed into’ at the place of
‘transformed to’
EXERCISE 6
Q.1.(4) ‘effluent’ should be used in place of ‘affluent’ and
‘for’ at the place of ‘against’

90 ENGLISH LANGUAGE
stportal.mahendras.org

CHAPTER

14 VOCABULARY
Scan the QR code to get video of this chapter.

adjectives, adverbs or preposition), as long as both members


Synonyms And Antonyms
of the pair are the same part of speech. More examples of
The Synonyms and Antonyms form an integral part of the
English.
English Language. Acquaintance with vocabulary of the
Noun
English language is a necessity for effective expression either
in the written or in the oral form. Convenience and comfort
Synonym is nothing but the similar meaning of a particular Verb
word or its semantic relation. So, it is a word or a phrase "buy" and "purchase"
that means the same as another word or a phrase in the same Adjective
language. "sick" and "ill"
Antonyms are the negative connotation of a particular word. Adverb
An Antonym is a word or phrase that is opposite in meaning
"quickly" and "speedily"
to a particular word or a phrase in the same language.
Preposition
Synonyms are different words with almost identical or
similar meanings. Words that are synonyms are said to be "on" and "upon"
synonymous, and the state of being a synonymous is called An antonym is a word that means the opposite of another.
synonyms. Example 1 : 'fat' is an antonym of 'thin'
Synonyms can be any part of speech (Eg. nouns, verbs, Example 2 : The words 'open' and 'closed' are antonyms.

ENGLISH LANGUAGE 91
stportal.mahendras.org

exercise
Q.10. Fatal
EXERCISE- 1
(A) terrible (B) deadly
Tell whether each pair of words are synonyms or antonyms.
(C) poisonous (D) wrong
1. agree, disagree ____________
Q.11. Concealed
2. cold, freezing ___________
(A) covered (B) closed
3. easy, difficult___________
(C) sealed (D) hidden
4. argue, squabble ____________
Q.12. Relish
5. guess, estimate ____________
(A) realize (B) taste
6. bottom, top ___________
(C) enjoy (D) reveal
7. tired, energetic __________
Q.13. Acute
8. huge, gigantic ____________
(A) dull (B) drowsy
9. sink, float __________
(C) unpleasant (D) sharp
10. windy, calm ______
Q.14. Solicit
11. noisy, quiet ___________
(A) command (B) request
12. unhappy, sad __________
(C) sympathize (D) agree
EXERCISE- 2 Q.15. Surmount
Q.1-20. In the following questions out of four alternatives, (A) discount (B) surround
choose the one which best expresses the meaning
(C) overcome (D) capture
of the given word and mark it in the Answer Sheet.
Q.16. Sycophant
Q.1. Successive
(A) supplement (B) leader
(A) rapid (B) victorious
(C) flatterer (D) blessing
(C) beneficent (D) consecutive
Q.17. disparage
Q.2. Ravage
(A) praise (B) compliment
(A) destroy (B) break
(C) exalt (D) underestimate
(C) demolish (D) abolish
Q.18. docile
Q.3. Superstitious
(A) idle (B) gentle
(A) pious (B) traditional
(C) stubborn (D) foolish
(C) irrational (D) sacred
Q.19. asperity
Q.4. Mentor
(A) harshness (B) mild
(A) guide (B) genius
(C) polite (D) civil
(C) stylist (D) philosopher
Q.20. soporific
Q.5. Garnish
(A) happy (B) terrifying
(A) paint (B) adorn
(C) lethargic (D) alert
(C)garner (D) banish
Q.6. Infructuous EXERCISE- 3
(A) meaningless (B) unnecessary Q.1-20. In the following questions choose the word opposite
(C) redundant (D) fruitless in meaning to the given word and mark it it the
Answer Sheet.
Q.7. Fidelity
Q.1. Exodus
(A) affection (B) allegiance
(A) influx (B) home-coming
(C) accuracy (D) loyalty
(C) return (D) restoration
Q.8. Gruff
Q.2. Inquisitive
(A) hard (B) rough
(A) insincere (B) indifferent
(C) tough (D) sturdy
(C) insensitive (D) insulting
Q.9. Doleful
Q.3. Candid
(A) mournful (B) sober
(A) outspoken (B) frank
(C) regretful (D) cheerless
(C) devious (D) disguised
92 ENGLISH LANGUAGE
stportal.mahendras.org
Q.4. Nadir (C) worth (D) dignity
(A) modernity (B) zenith Q.20. debilitate
(C) liberty (D) progress (A) exhaust (B) spoil
Q.5. Culpable (C) strengthen (D) enfeeble.
(A) defendable (B) blameless EXERCISE- 4
(C) careless (D) irresponsible
Q.1. Find the antonyms of incite 
Q.6. Facilitate (A) Encourage  (B) Provoke 
(A) help (B) propogate (C) Urge  (D) Block 
(C) hinder (D) reject Q.2. Find the antonyms of atrophy 
Q.7. Criticise (A) Decline  (B) Diminution 
(A) finish (B) recommend (C) Downfall  (D) Betterment 
(C) praise (D) request Q.3. Find the antonyms of glimmer 
Q.8. Conformity (A) Dullness  (B) Flash 
(A) deviation (B) dilution (C) Sparkle  (D) Gleam 
(C) distraction (D) diversion Q.4. Find the antonyms of scatter 
(A) Discard  (B) Distribute 
Q.9. Affirm
(C) Divide  (D) Hold 
(A) refuse (B) negate
Q.5. Find the antonyms of eject 
(C) neglect (D) avoid
(A) Dismiss  (B) Accept 
Q.10. Autonomy (C) Dump  (D) Eliminate 
(A) slavery (B) subordination
Q.6. Find the antonyms of engage 
(C) dependence (D) submissiveness (A) Employ  (B) Shun 
Q.11. abandon (C) Enlist  (D) Appoint 
(A) regain (B) retain Q.7. Find the antonyms of rapture 
(C) remain (D) revive (A) Cheer  (B) Cool 
Q.12. humble (C) Glory  (D) Depression 
(A) rich (B) powerful Q.8. Find the antonyms of hallucination 
(C) haughty (D) strong (A) Dream  (B) Delusion 
(C) Reality  (D) Mirage 
Q.13. exemptions
(A) generalisation (B) liberalization Q.9. Find the antonyms of wraith 
(A) Vision  (B) Shadowy 
(C) exclusions (D) inclusion
(C) Spirit  (D) Reality 
Q.14. prominent
Q.10. Find the antonyms of apparition 
(A) unknown (B) eminent (A) Haunt  (B) Visitant 
(C) renowned (D) important (C) Fact  (D) Chimera 
Q.15. betrayal EXERCISE- 5
(A) deception (B) treason Q.1. Choose the synonym of ‘DIRE’ 
(C) loyalty (D) distrust
(a) perturb  (b) horrible 
Q.16. autonomous (c) patient  (d) kind 
(A) dependent (B) uncontrolled
Q.2. Choose the antonym of ‘NETTLE’ 
(C) manual (D) free (a) repress  (b) separation 
Q.17. zeal (c) annoy  (d) please 
(A) fervor (B) eagerness Q.3.  Choose the synonym of ‘CLUMSY’ 
(C) apathy (D) initiative (a) regain  (b) burdensome 
Q.18. predilection (c) clutch  (d) waste 
(A) dislike (B) likeness Q.4. Choose the antonym of ‘DECIPHER’ 
(C) aptitude (D) inclination (a) coercion  (b) construe 
(c) expropriate  (d) confuse 
Q.19. probity
(A) honesty (B) deception Q.5. Choose the synonym of ‘BONA FIDE’ 

ENGLISH LANGUAGE 93
stportal.mahendras.org
(a) redeem  (b) results  (1) Only (A) (2) Both (A) & (B)
(c) actual  (d) union  (3) Only (C) (4) Both (C) & (D)
Q.6.  Choose the antonym of ‘VIGILANT’  (5) All of the above
(a) move  (b) careful 
(c) negligent  (d) chaos  Q.4. DISGUISE
Q.7.  Choose the synonym of ‘FAVOUR’  (A) The children looked at her without disguise.
(a) crucifying  (b) backing  (B) You have a disguise grip on reality.
(c) purifying  (d) crying  (C) Does holding a handkerchief over the
Q.8. Choose the synonym of ‘FACULTATIVE’  mouthpiece really disguise your voice?
(a) mix  (b) unrestricted  (1) Only (A) (2) Both (A) & (C)
(c) use  (d) decrease 
(3) Only (C) (4) Both (C) & (D)
Q.9.  Choose the synonym of ‘GENUFLECTION’ 
(a) expression  (b) monotony  (5) All of the above
(c) conversation  (d) tier  Q.5. LOOK OFF
Q.10.  Choose the antonym of ‘MELIORATE’  (A) The gameplay is great, but the graphics on the
(a) correct  (b) bemoan  characters' faces are looking off.
(c) familiar  (d) repress
(B) You're looking a little off, Jim. Are you feeling
EXERCISE- 6 all right?
Q.1-10. In the following questions a word is given and
(C) With a stadium of fans looking off, the pressure
its usages are given just below it. So now you
was on for him to make the field goal.
have to choose which of the following usages
of the word in the sentence are correct, both (1) Only (A) (2) Both (A) & (B)
grammatically and contextually. If all the (3) Only (C) (4) Both (C) & (D)
three usages are correct choose option (5) i.e.,
all of the above as your answer. (5) All of the above

Q.1. PLUNGE Q.6. FICKLE

(A) They whooped as they plunged into the sea. (A) The fickle weather soon had me changing from
my shorts into my thick slacks.
(B) Gaurang plunged his hands into his pockets.
(B) Once we arrived at the movie theatre, my fickle
(C) Shares in the company plunged 18p on news. sister could not decide which movie she wanted
(1) Only (A) (2) Both (A) & (B) to see.
(3) Only (C) (4) Both (C) & (D) (C) Sally is so fickle that it takes nearly an hour for
(5) All of the above her to choose an item from the restaurant menu.
Q.2. REVERBERATE (1) Only (A) (2) Only (B)
(A) Her deep booming laugh reverberated around (3) Only (C) (4) Both (A) and (B)
the room. (5) All of the above
(B) The statements by the professor reverberated Q.7. SACRILEGE
through the Capitol. (A) It is considered an act of sacrilege for anyone
(C) He flashed his most reverberated smile to touch the sacred statue.
(1) Only (A) (2) Both (A) & (B) (B) Most Christians on this ground repudiate the
(3) Only (C) (4) Both (C) & (D) application of the term to the sacrilege of Jesus
Christ.
(5) All of the above
Q.3. CHERISH (C) Remembered that this service was primarily
regarded not as an act of sacrilege but as a
(A) He needed a person he could cherish. meeting for instruction in the law.
(B) He had long cherished a secret fantasy about (1) Only (A) (2) Only (B)
his future.
(3) Only (C) (4) Both (A) and (B)
(C) I cherish the poem she wrote.
(5) All of the above
94 ENGLISH LANGUAGE
stportal.mahendras.org
Q.8. CONSUMMATE ii. This comic novel, though (A) antique / (B)
(A) Because my sister is a consummate liar, she antic rather than earnest, very different in style
can make anyone believe practically anything. and tone from Naipaul, is serious about race,
social class, immigrants, and outsiders.
(B) My charity organization is looking for a
consummate fundraiser to help our group raise iii. Though we believe we have a chance of
much-needed funds. remaining independent, we have to work
together and would like to have an (A)
(C) It is a tournament that is open to both amicable / (B) amiable relationship.
consummate and professionals.
(1) A A A (2) A B B
(1) Only (A) (2) Only (B)
(3) A B A (4) B A B
(3) Only (C) (4) Both (A) and (B)
(5) B B A
(5) All of the above
Q.2. i. At that time the viewers had come to wonder,
Q.9. GENTEEL
she wished so badly to serve as a mere (A) aid
(A) Since this is supposed to be a genteel meeting, / (B) aide while according to all her instructors
I will refrain from calling anyone mean names. she had excelled well in the art of sagecraft.
(B) When one attends a genteel social event, he or ii. The bride slowly bedecked in a glowing white
she should always bring a gift for the host. gown made her way down the (A) isle / (B)
(C) The judge was pleased by the genteel way the aisle carefully, as three hundred people sat
husband and wife treated each other during watching her every move.
their divorce hearing. iii. The quality of the athletes, always impressive,
(1) Only (A) (2) Only (B) seemed to take a quantum leap forward, a happy
(A) augur / (B) auger for the future of the sport
(3) Only (C) (4) Both (A) and (B)
in this Eastern European nation.
(5) All of the above
(1) A A A (2) A B B
Q.10. TORPID
(3) A B A (4) B A B
(A) According to the preference settings, the laptop
(5) B B A
will go to sleep when it is has been torpid for
five minutes. Q.3. i. The issue largely depends on whether the
(A) auricle / (B) oracle normally gave her
(B) The player was dropped from the team because
responses in glossolalia or in intelligible
he was torpid and refused to put forth effort
speech.
during practice.
ii. The king, hearing the hiss of the serpents,
(C) The players stay torpid during the cold winter
scrambled as fast as he could to the window,
months by skiing and ice skating.
and (A) balled / (B) bawled out to the
(1) Only (A) (2) Only (B) abominable enchantress never to come back.
(3) Only (C) (4) Both (A) & (B) iii. I t is predominantly a terrestrial species,
(5) All of the above although it has been known to climb up banks
EXERCISE- 7 and into low bushes in order to (A) bask / (B)
basque or search for prey.
Q.1-3. In the following questions, three sentences with
two highlighted words are given and denoted as (1) A A A (2) A B B
(A) & (B). Below them, five options are suggested (3) A B A (4) B A B
consisting of the correct set of words used in the (5) B B A
sentences respectively.
Q.4-5. In the following questions two sentences are
Q.1. i. While the military commander is unable to given below. Each sentence has a pair of words
(A) accede / (B) exceed to his troop’s request that are highlighted. From the highlighted
for a forty-eight hour leave, he can give words select the most appropriate word (A or
them a twenty-four hour leave for rest and B) to form correct sentences. Choose the most
recuperation. appropriate one.

ENGLISH LANGUAGE 95
stportal.mahendras.org
Q.4. i. Hopefully the movie trailer piqued (A) / C. Affinity iii. amicable
peaked (B) the interest of moviegoers and
motivate them to buy tickets to see the film. (1) Only A is defined
(2) Only B is defined
ii. The criminal forced his wife to be his consort
(A)/ concert (B) in crime. (3) Both A and B are defined
(1) AA (2) BB (4) Both B and C are defined
(3) AB (4) BA (5) All are defined/ All are not defined
(5) None of the above Q.3.
Q.5. i. The mountain trek will exhilarate(A)/ Column 1 Column 2
accelerate(B)tourists who love to challenge A. Lurch i. make an uncontrolled
themselves. move
ii. His reactions to unpleasant situations tended B. Unprecedented ii. never done before
to aggregate (A)/ aggravate (B) everyone’s C. Impuissant iii. powerless
nerves. (1) Only A is defined
(1) AA (2) BB (2) Only B is defined
(3) BA (4) AB (3) Both A and B are defined
(5) None of the above (4) Both B and C are defined
EXERCISE- 8 (5) All are defined/ All are not defined
Q.1-5. In the question three different words are given Q.4.
followed by three phrases which may or may
not define the words mentioned in column 1. Column 1 Column 2
Choose the option/s which is/are available as A. Cantankerous i. bad tempered
the meaning of the words. If the meaning is B. Afflict ii. cause pain or trouble
‘available’ or ‘not available’ for all the words,
C. Contemplate iii. contemporaneous
mark option (5).
Q.1. (1) Only A is defined
(2) Only B is defined
Column 1 Column 2
(3) Both A and B are defined
A. Deterrent i. something that discourages
(4) Both B and C are defined
B. Envisage ii. to gain advantage by
manipulating unfairly (5) All are defined/ All are not defined
C. Hitherto iii. upto a level Q.5.
(1) Only A is defined Column 1 Column 2
(2) Only B is defined A. Entrenched i. enumerable
(3) Both A and B are defined B. Curmudgeon ii. a bad tempered person
(4) Both B and C are defined C. Ambidextrous iii. a person who can
write with both hands
(5) All are defined/ All are not defined
Q.2. (1) Only A is defined
(2) Only B is defined
Column 1 Column 2
(3) Both A and B are defined
A. Garrulous i. causing quarrels
(4) Both B and C are defined
B. Preternatural ii. beyond what is normal
or natural (5) All are defined/ All are not defined

96 ENGLISH LANGUAGE
stportal.mahendras.org

EXPLANATION
happy- pleased
EXERCISE 1
terrifying- make greatly afraid.
1. agree, disagree-antonyms
alert- attentive
2. cold, freezing -synonyms
3. easy, difficult -antonyms EXERCISE 3
4. argue, squabble -synonyms Q.1.(C) Q.2.(B) Q.3.(C) Q.4.(B)
5. guess, estimate -synonyms Q.5.(B) Q.6.(C) Q.7.(C) Q.8.(A)
6. bottom, top -antonyms Q.9.(A) Q.10.(C)
7. tired, energetic -antonyms Q.11.(B) Abandon : give up with the intent of never claiming
8. huge, gigantic -synonyms again
9. sink, float -antonyms Q.12.(C) Humble:marked by meekness or modesty
10. windy, calm -antonyms Q.13.(D) Exemptions : immunity from an obligation or duty
11. noisy, quiet -antonyms Q.14.(A) Prominent : conspicuous in position or importance
12. unhappy, sad -synonyms Q.15.(C) Betrayal : the quality of aiding an enemy
Q.16.(A) autonomous means independent so its antonym is
EXERCISE 2 dependent.
Q.1.(D) Q.2.(A) Q.3.(C) Q.4.(A)
For other options:-
Q.5.(B) Q.6.(D) Q.7.(D) Q.8.(B)
uncontrolled- without a check or restrain manual-
Q.9.(A) Q.10.(B) done or operated by hands rather than by an
Q.11.(D) Concealed : not accessible to view electronic device.
Q.12.(C) Relish : to enjoy free- enjoying personal rights or liberty.
Q.13.(D) Acute : extremely sharp or intense Q.17.(C) zeal means enthusiasm, its antonym is apathy- lack
Q.14.(B) Solicit : request urgently or persistently of interest.
Q.15.(C) Surmount : deal with successfully For other options:-
Q.16.(C) sycophant means flatterer i.e. a person who tries fervor- great warmth and earnestness or feeling.
to please by complimentary remarks or attention eagerness- enthusiasm
For other options:- Q.18.(A) predilection means a tendency to think favourably
supplement- something added to complete a thing of something in particular so its antonym here is
leader- a person/thing that leads. dislike.
blessing- a special favour, mercy or benefit. For other options:-
Q.17.(D) understimate means to make too low an estimate likeness- the condition of being alike.
of or to think insufficiently highly of. aptitude- capability
For other options:- inclination- a tendency towards a certain condition.
praise- to express commendation, admiration, etc. Q.19.(B) probity means honesty, its antonym is deception
compliment- an expression of praise, admiration, which means to cheat.
etc For other options:-
exalt- to raise in rank or honor. honesty- synonym of probity
Q.18.(B) gentle- amiable or kind worth- good or important enough to justify
For other options:- dignity- nobility or elevation of character.
idle- not working or active Q.20.(C) debilitate means to make weak or feeble so its
stubborn- unreasonably obstinate, one who refuses antonym is strengthen
to comply, agree, or give in. For other options:-
foolish- showing a lack of sense. exhaust- to drain of strength or evergy.
Q.19.(A) harshness- rude in behaviour spoil- to damage severely or harm enfeeble- make
For other options:- weak.
mild- amiably gentle or temperate in feeling or behaviour EXERCISE 4
towards others. Q.1.(D) Q.2.(D) Q.3.(A) Q.4.(D) Q.5.(B)
polite- showing good manners towards others. Q.6.(B) Q.7.(D) Q.8.(C) Q.9.(D) Q.10.(C)
civil- polite or courteous.
EXERCISE 5
Q.20.(C) lethargic- lazy
Q.1.(B)  Q.2.(D)  Q.3.(C)  Q.4.(D)  Q.5.(C) 
For other options:-
Q.6.(C)  Q.7.(B)  Q.8.(B) Q.9.(A)  Q.10.(D)
ENGLISH LANGUAGE 97
stportal.mahendras.org
Reverberate : have continuing and serious effects.
EXERCISE 6
Q.3.(5) Auricle (n): Outer part of the ear
Q.1.(5) Plunge : jump or dive quickly and energetically.
Oracle (n): a priest or priestess acting as a medium
Plunge : suffer a rapid decrease in value.
through whom advice or prophecy was sought
Plunge : push or thrust quickly.
from the gods in classical antiquity.
Q.2.(2) Reverberate : (of a loud noise) be repeated several
Balled (v) : clench (one's fist) tightly / wrap the
times as an echo.
root ball of (a tree or shrub) to protect it during
Reverberate : have continuing and serious effects. transportation
Q.3.(5) Cherish : protect and care for (someone) lovingly. Bawl (v): shout
Cherish : keep (a hope or ambition) in one's mind. Bask (v): lie exposed to warmth and light, typically
Cherish : hold (something) dear. from the sun, for relaxation and pleasure.
Q.4.(2) Disguise : the concealing of one's true intentions Basque (n): tight fitting dresses.
or feelings. Q.4.(1) In 1st sentence pique (v) is used which means
Disguise : make (something) unrecognizable by arouse (interest or curiosity) and peaked (adj.)
altering its appearance, sound, taste, or smell. means rising to a point
Q.5.(2) Look off : To appear somewhat strange, incorrect, In 2nd sentence consort (n) is used which means
inaccurate, or substandard. partner, companion
Look off : To look sickly, unwell, or out of sorts, Concert (n) means a musical performance given
either physically or mentally. in public, typically by several performers or of
For other options- several compositions.
Look on : To watch some action or activity as a Q.5.(4) In 1st sentence exhilarate (v) is used which means
spectator. to make (someone) feel very happy and accelerate
With a stadium of fans looking on, the pressure (v) means increase in rate, amount, or extent.
was on for him to make the field goal. In 2nd sentence aggravate (v) is used which means
Q.6.(5) to make (a problem, injury, or offence) worse or
Q.7.(1) more serious and aggregate (n) means total.
Q.8.(4) EXERCISE 8
Q.9.(5) Q.1.(1) Hitherto (adverb) – until now
Q.10.(4) Envisage (v) – form a mental picture of (something
EXERCISE 7 not yet existing or known).
Q.1.(3) Accede (v): agree to something. Q.2.(2) For other options-
Antic (adj.)- odd, strange , antique (adj)- ancient Garrulous (adj) – talkative
Amicable (adj): friendly , amiable (adj): lovable Affinity (n) – empathy
Q.2.(5) Aid (v): to assist Q.3.(5) All are defined.
Aide (n): an assistant Q.4.(3) For other options-
Aisle(n): walkway Contemplate (v) – look thoughtfully at something
Isle(n): Island Contemporaneous (n) – existing at or occurring
Augur (n): a religious official who observed in the same period of time
natural signs, especially the behaviour of birds, Q.5.(4) For other options-
interpreting these as an indication of divine Entrenched (adj) – established/ unlikely to change
approval or disapproval of a proposed action. Enumerable (adj) – able to be counted by one-to-
Auger (n): a tool for drilling one

98 ENGLISH LANGUAGE
stportal.mahendras.org

CHAPTER
READING
15 COMPREHENSION
Scan the QR code to get video of this chapter.

Questions on Reading Comprehension measure the ability 6. Read the questions carefully , making sure that you
to understand, analyze and apply information and concepts understand what is being asked . if need better refer
presented in the written form. All questions are to be back to the passage for finding the answer.
answered on the basis of what is stated or implied in the 7. The choice may expand or extend the idea or contrast
given passage. Reading comprehension, therefore evaluates the idea running in the paragraph.
your ability to :
8. Read all the alternatives very carefully . without first
Understand words and statements in the given passage. reading all the alternatives , do not assume that you
Understand the logical relationships between points and have selected the best answer.
concepts in the given passage. 9. Many a times the last sentence of the paragraph can
Draw inference from facts and statements in the given also provide a clue .
passage. 10. Usually the correct choice will not deviate from the
Read the questions first before you read the passage. subject matter too much.
Guidelines for answering the reading comprehension 11. Don't jump to conclusions with fact questions using
questions:- Roman numerals to identify answer choices:- You
1. Read the question first before you read the passage. will recognize this style of question as soon as you
see it:
2. Identify the type of passage you are reading .
a. I only b. II only c. III only
3. Identify the core idea or the theme of the paragraph.
d. I and II only e. II and III only
4. Look for the clues in the choices , normally the
correct is the one which falls under the theme. The catch is that, oftentimes, facts I and II will be presented
very close to each other in the passage, but fact III will be
5. Answer all the questions on the basis of what is stated buried much further in the text. Take the time to review and
or implied in the passage itself. consider each fact on its own merits.

ENGLISH LANGUAGE 99
stportal.mahendras.org

exercise
Read the following passages and Answers the questions (3) Ditch (4) Hill
given below (5) Mountain
PASSAGE - 1 Q.7. What did the turtle tell the frog about ‘The Ocean’ ?
There was a frog that lived in a shallow well. “ Look how (1) Its more than a hundred miles across and more
well off I am here ! “ he told a big turtle from the Eastern than eight thousand feet deep.
Ocean. “ I can hop along the coping of the well when I go (2) Its more than a thousand miles across and more
out, and rest by a crevice in the bricks on my return. I can than ten thousand feet deep.
wallow to my heart’s content with only my head above water,
(3) The ocean is only twelve thousand feet deep.
or stroll ankle deep through soft mud. No crabs or tadpoles
can compare with me. I am master of the water and lord of (4) The ocean is only twenty thousand feet deep.
this shallow well, What more can a fellow ask ? Why don’t (5) Not mentioned in the passage.
you come here more often to have a good time ? “ Q.8. Which of the following statement(s) is/are true in
Before the turtle from the Eastern Ocean could get his left foot context of the passage ?
into the well, however, he caught his right claw on something. (1) In ancient times there were floods nine years
So he halted and stepped back then began to describe the
out of ten.
ocean to the frog. “ It’s more than a thousand miles across
and more than ten thousand feet deep. In ancient times there (2) There were droughts seven years out of eight.
were floods nine years out of ten yet the water in the ocean (3) A frog lived in a deep well.
never increased. And later there were droughts seven years (4) Both 1 & 2
out of eight yet the water in the ocean never grew less. It has
(5) The frog lived in the Western Ocean.
remained quite constant throughout the ages. That is why I
like to live in the Eastern Ocean. “ Q.9. Which of the following has a similar meaning to the
word ‘abashed’ ?
Then the frog in the shallow well was silent and felt a little
abashed. (1) Amused (2) Ashamed
Q.1. Give a suitable title to the passage. (3) Embarrassed (4) Both 2 & 3
(1) The frog in the well. (2) The frog. (5) Pleased
(3) The well (4) Shallow well. Q.10. Which of the following statement(s) is/are not true
(5) None of these in context of passage ?
Q.2. What do you understand by the phrase - ‘I can wallow (1) The frog lived in a shallow well.
to my heart’s content’ ? (2) The big turtle did not live in the Eastern Ocean.
(1) Rely on my heart’s beating. (3) In ancient times, there were floods nine years
(2) Singing a song. out of ten.
(3) Listen to my heart. (4) All of these
(4) Rest on his assets. (5) None of these
(5) Not mentioned in the passage. PASSAGE - 2
Q.3. Give a synonym for the word “Shallow”. Read the following passage carefully and answer the
(1) Vulnerable (2) Perspicacious questions given below it. Certain words have been printed in
bold to help you locate them while answering some questions.
(3) Deep (4) Depth
For the past 10 years, all exports of pulses have been banned.
(5) None of these This was presumably for food security and price stability.
Q.4. Give an antonym for the word ‘Abashed’. But this export ban has hurt farmers, who couldn’t take ad-
(1) Bewildered (2) Chagrined vantage of high prices. It got worse, because when prices did
(3) Crushed (4) Proud shoot up, the government panicked and started importing at
zero duty. The imports didn’t stop despite a record harvest.
(5) Chased
When prices of food crops go up, the Central government
Q.5. Where did the big turtle come from ? swings into action, clamping down on exports, bringing in
(1) The Northern Ocean (2) The Southern Ocean zero-duty imports, imposing stocking and storage limits, and
(3) The Eastern Ocean (4) The Western Ocean so on. But when the opposite happens, that is when prices
(5) None of these crash, often, there is no corresponding reverse rescue. This is
Q.6. What is the meaning of ‘Crevice’ in the passage? an example of the inherent urban bias in India’s agriculture
policy, which persists to this day. Indeed, among all World
(1) Cleft (2) Vex
100 ENGLISH LANGUAGE
stportal.mahendras.org
Trade Organization members, India is among a handful of (4) imposing stocking and storage limits
countries with negative protection of agriculture. (5) All of the above
Lastly, one more policy lesson is that pulses continue to be Q.5. Choose the most suitable title for the given passage?
in the Agriculture Produce Marketing Committee (APMC)
(1) Enhancing the production
Act. Thus farmers are not free to sell to any buyer they wish
but must go through the APMC. (2) Unshackle Indian agriculture
Pulses are an important source of protein in India. Almost (3) Loan waiver to farmers
one-fifth of total acreage is used to grow them. They are (4) Yet another reaction to policy-induced
also an important source of soil fertility since they provide indebtedness
nitrogen through fixation. Indeed, a pulses plant is called a (5) Advantage of high prices
mini fertilizer factory. Pulses were part of the six technol-
Q.6. Which of the following is not true according to
ogy missions created in the 1980s (they were added to the
the passage?
oilseeds mission), to greatly enhance their production, use
of technology and processing. But despite the mission-mode (1) The nationwide policies of procuring
approach, India is still not self-sufficient in pulses production. wheat and rice have proved harmful to health.
And farmers continue to be at the mercy of nature, markets, (2) The nationwide policies of procuring wheat and
pests and government policies. rice have proved harmful to nutrition.
The story of pulses is repeated across various crops. In cashew, (3) The nationwide policies of procuring wheat and
India’s Kollam used to be the world’s capital, but has lost out rice have proved harmful to environment.
to Vietnam due to its failure to adopt technology and due (4) Coarse cereals, the staple of most rural folk, were
to excessive government control. India pioneered the Green largely ignored by the public distribution system.
Revolution, but scores very poorly in the global hunger index. (5) Coarse cereals are not only now being intro
The monolithic nationwide policies of promoting and procuring duced in the midday meal schemes.
wheat and rice have ultimately proved harmful to health, nutri-
tion and environment. Coarse cereals, the staple of most rural Q.7. Choose the word which is most nearly the OPPOSITE
folk, were largely ignored by the public distribution system, and in meaning as the word printed in bold as used in the
are only now being introduced in the midday meal schemes. passage.
Q.1. Why India’s Kollam which used to be the world’s ENHANCE
capital has lost out to Vietnam in cashew? (1) subtle (2) increase
(A) due to its failure to adopt technology (3) diminish (4) fervid
(B) due to excessive government control (5) flourish
(C) due to excessive usage of pesticides Q.8. Choose the word which is most nearly the OPPOSITE
(1) Both (A) and (B) (2) Only (B) in meaning as the word printed in bold as used in the
passage.
(3) Both (A) and (C) (4) Only (C)
FERTILITY
(5) Only (A)
(1) flexibility (2) sterility
Q.2. Why are pulses an important source of soil fertility?
(3) credibility (4) ability
(1) they provide oxygen through fixation
(5) nudity
(2) they provide halogen through fixation
Q.9. Choose the word which is most nearly the SAME in
(3) they provide sodium through fixation meaning as the word printed in bold as used in the
(4) they provide nitrogen through fixation passage.
(5) Both (1) and (3) MERCY
Q.3. Where according to the passage farmers continue (1) adequacy (2) clemency
to be at? (3) cartography (4) pacifism
(1) the mercy of nature (2) markets (5) sadism
(3) pests (4) government policies Q.10. Choose the word which is most nearly the SAME in
(5) All of the above meaning as the word printed in bold as used in the
Q.4. What happens when prices of food crops go up passage.
according to the passage? ADOPT
(1) the Central government swings into action (1) lend (2) ignore
(2) clamping down on exports (3) accept (4) allure
(3) bringing in zero-duty imports (5) entitle
ENGLISH LANGUAGE 101
stportal.mahendras.org
Q.1. The author’s chief concern seems to be-
PASSAGE - 3
(1) Frustration among students.
Frustration is a global cancer. It has spared no country. In
some countries, frustration exists because these countries (2) Spread of college education.
are populated by ‘have nots’. In other countries which are (3) Standard of education.
populated by ‘haves’. Frustration is among them also because (4) Autonomy to colleges.
they do ‘have’ as far as India is concerned its education
(5) Delinking of degrees.
system is breeding more and more frustration among both
students and teachers. And yet no one seems too clear about Q.2. The author’s attiude is-
what can be done to make it more meaningful. Many reforms (1) Cynical (2) Optimistic
have been discussed length but have come to nothing for one (3) Critical (4) Conservative
reason or another . The authorities are now peddling the idea (5) Constructive
of autonomous colleges as a means of toning up teaching
Q.3. What according to the passage, would be the result
standards. They argue that by allowing certain colleges to
of granting autonomy to some colleges?
introduce their own courses, hold seminars, and above all,
to evolve their own method of assessing students, students (1) The teacher student ratio will come down.
will get a far better deal. Indeed, they make out that such (2) Disparity between city and suburban colleges will
colleges will have free hand in nearly every thing except increase.
granting degrees. In theory all this sounds, attractive enough. (3) Colleges will multiply.
But there is little to show that the managements concerned (4) Some colleges would start selling degrees.
are keen on such reforms. Even today, nothing prevents a
college from inviting guest speakers on specialized subjects (5) All of the above.
or holding courses in English for term vernacular students., Q.4. Which of the following is the most significant feature
But not a single one of those who are now clamouring for of the scheme of autonomous colleges, according to
autonomy has bothered to do so. It is no secret that colleges the passage ?
which may be ‘Freed to an extent from university control (1) They can introduce new courses.
are highest : In Mumbai, for example, there is big cultural (2) They can hold seminars.
gulf between city colleges and Jordon degree of autonomy,
(3) They can evolve their method of assessment.
it will only heightened this disparity.
(4) They can award degrees.
The answer to the vexed problem of declining standards in
(5) They can make admissions without restrictions.
higher education does not lie in encouraging the growth of
model institutions, but in improving over all standards. This Q.5. To which of the following does the author give
is of course easier said than done, since more and more young precedence?
people are seeking degrees. The only solution, however (1) Increasing library facilities.
unpalatable it may sound, is drastically to reduce the number (2) Holding refresher courses for teachers.
of those who are admitted to colleges. Even though there is (3) Abolishing reservation for backward classes.
political pressure on many state governments to build new
(4) Stop opening of new colleges.
colleges and to reserve more seats for backward clases, it will
be sheer folly to expand such facilities recklessly without (5) Instituting entrance examinations.
giving any thought to the quality of education imparted. If Q.6. Where has the author slipped while making suggestions ?
admissions are made far more selective, it will automatically (1) The teacher-student ratio be brought down.
reduce the number of entrants. This should apply particularly
(2) Improvement should not be restricted to select
to new colleges, many of which are little more than degree
institutions.
factories. Only then can the authorities hope to bring down
the teacher-student ratio to manageable proportions. What (3) College teachers need refresher courses.
is more, teachers should be given refresher courses every (4) Political pressures should be withstood.
summer vacation to brush up their knowledge. Besides, (5) None of these
if college managements increase the library budget it will Q.7. Frustration in the world is a result of which of the
help both staff and students a great deal. At the same time, following?
however, it will be unfair to deny college education to
(a) Large number of ‘Haves’
thousands of young men and women unless employers stop
insisting on degrees even for clerical jobs. For a start, why (b) Large number of ‘Have-nots’.
can’t the Government disqualify graduates from securing (c) Cancerous cells.
certain jobs say, Class III and IV posts ? Once the degrees are (1) Only A (2) Only B
delinked from jobs, at least in some important departments,
(3) Only C (4) A and B only
it will make many young people think twice before joining
college. (5) None of these
102 ENGLISH LANGUAGE
stportal.mahendras.org
Q.8. Which of the following is the main difficulty India occupies in American foreign policy
according to the passage, in the way of improving calculus.
standards? (3) The U.S. is India's largest export destination
(1) Paucity of library facilities. and also one of the leading foreign investors
(2) Paucity of Funds. in India.
(3) Ever increasing number of young men wanting (4) Both 2 and 3
degrees. (5) None of these
(4) Grant of autonomy to colleges. Q.2. Which of the following statement(s) is/are NOT
(5) Absence of refresher courses. TRUE in context to the passage for the country 'India'?
Q.9. Give one synonym for the word ‘Vexed’. (1) India is the fastest growing major economy.
(1) Annoyed (2) Recurring (2) India is the largest demoratic country.
(3) Unresolvable (4) Complex (3) India is characterized as a nascent major power
of the U.S.
(5) Dangerous
(4) India is the second fastest growing major
Q.10. Give an antonym for the word ‘folly’.
economy.
(1) Right (2) Exact (3) Mistake
(5) Both 1 and 4
(4) Action (5) Wisdom
Q.3. Which of the following statement(s) is/are TRUE in
PASSAGE - 4 context to the passage?
The President of the USA, Barack Hussain Obama will be (1) The United States plays a significant role in
on his maiden visit to India. That a President of the USA is global arena but not India.
visiting India in his first term, within the first 24 months of his
(2) The President of the U.S.A. is visiting India
tenure, undoubtedly denotes the position India occupies in the
in his first term within the first 24 months of
American foreign policy calculus. It is also an indicator that
his tenure.
the relationship between the world’s two largest democracies
is on a strong footing. India and United states politically and (3) The United States is the World's strongest
economically play a significant role in global arena .While the democracy and India is the largest.
US is the world’s strongest democracy, India is the largest. (4) All 1, 2 and 3
Similarly, while the US is the world’s largest economy, India (5) Both 2 and 3
is the second fastest growing major economy. The US is
Q.4-7. Choose the word which is most nearly the SAME in
India’s largest export destination and also one of the leading
meaning as the word printed in bold as used in the
foreign investors in India. Further, with the Indian economy
passage.
estimated to grow to 90% of the US economy by 2050.
The growing Indo -US relationship will be a decisive force Q.4. Nascent
shaping the contours of the world economy in 21 century. (1) Existing (2) Latest
In a move to further improve economic relations, both (3) Recent (4) New
countries had two rounds of exploratory talks in the first (5) Blossoming
half of the 2008 on the proposed Bilateral Investment
Q.5. Maiden
Treaty, and decided to start formal negotiations soon. The
proposed agreement would enable certification of Indian (1) Latest (2) First
aviation and aeronautic products by the US authorities. (3) Last (4) Recent
Further more India and US will also be soon signing the (5) Foremost
Totalization agreement to promote Indo-US cooperation on
Q.6. Proposed
social security. The agreement will be between the US social
security administration and the Indian Ministry of Overseas (1) Calculated (2) Considered
Indian Affairs. India with more than one billion citizens is (3) Purpose (4) Envisage
often characterized as a nascent major power and ‘Natural (5) Designed
Partner' of the United States one, that many analysts view as Q.7. Footing
a potential counter weight to China’s growing clout.
(1) Angle (2) View
Q.1. Which of the following can be inferred from the
passage? (3) Walk (4) Structure
(1) India and China will also be soon signing the (5) Period
Totalisation Agreement. Q.8-10. Choose the word which is most nearly the
(2) The visit of the president denotes the position OPPOSITE in meaning as the word printed in bold
as used in the passage.
ENGLISH LANGUAGE 103
stportal.mahendras.org
Q.8. Characterized Before 1970s, industrially develop western countries faced
(1) Confused (2) Portray economic crisis. Economic depression lowered down the
profit of big industries. Instead of depending on their own
(3) Felt (4) Wanted
market, the entrepreneurs wanted to expand their business in
(5) Expected other countries of the world. While these western companies
Q.9. Decisive were in search of market in different parts of the world, social-
(1) Particular (2) Little istic states wanted them in the privatisation process of public
sectors. Undeveloped third world countries also realized the
(3) Big (4) Trival
need of foreign capital and technical knowhow for their eco-
(5) Important nomic growth. The process of economic liberalisation began
Q.10. Bilateral as a precondition of globalisation. Thus, capital, technical
(1) Same (2) Unilateral two parties knowledge, labour, profit, managerial skill etc. move from
(3) Lucrative (4) Different one nation to another cutting across the territorial limitation
with the primary goal of expanding trade and business.
(5) Fruitful
Globalisation, as a negative process, stands for the aboli-
PASSAGE - 5 tion of protective economic policies which were previously
Read the following passage carefully and answer the adopted in the national interest. Governments had created
questions given below it. trade barriers by increasing the rate of tariff and through the
Globalisation has emerged as one of the most frequently used imposition of restrictions on import of foreign goods. Glo-
term of which there is no universally acceptable definition. balisation stands for the elimination of such negative barriers
World­wide opinions are divided both in support and opposi- for the free flow of goods and capital. Positively speaking,
tion of it. Globalisation is derived from the term “global” globalisation encourages free world trade, free enterprise
which implies covering the globe. Thus, globalisation means and market forces. It advocates for the process of integrat-
something which covers or relates the whole world instead of ing the domestic economy with the world economy through
being confined to a particular country. With the development of increased collaborations among the entrepreneurs. In this
modern science and technology, whole world has been closely process, they can have access to more capital flows, techni-
united as a global village. What happens in one country has its cal knowhow, cheaper imports and larger export markets.
impact on others. We are using the term internationalism as a India became independent as one of the poorest countries of
political process. This process highlights increasing political, the world. The British colonial rule had destroyed the self-
social, economic and cultural relations among various nations sufficient agrarian economy.
of the world. Now globalisation is advocated as a process aimed Q.1. What do you understand by the term globalisation
at integrating the whole world system. It implies the integration as mentioned in the passage?
of a nation’s economy with the world economic system. (1) It means the policy or practice of restructuring
Globalisation means different things to different people. or reforming the economic and political
Economists consider it as a step towards a fully integrated system.
world market. Some political scientists view it as a march (2) It means the provision of capital for a company,
away from the conventionally defined concept of the state. or the conversion of income or assets into
The sovereignty of the state is challenged with the emergence
capital.
of non-governmental power players in the world order. Glo-
balisation is not an event, rather a process which originated (3) It means a general statement or concept obtained
with the liberalisation and privatisation of the economic by inference from specific cases.
sectors. It aims for the establishment of a borderless world. (4) It means something which covers or relates the
Nations having socialistic economic system and mixed whole world instead of being confined to a
economy failed to achieve targeted economic growth. In particular country.
1980s, public sectors faced loss due to bureaucratic inef- (5) It means the changing of something from private
ficiency, red-tapism, political interference, corruption and to state ownership or control. the
irresponsibility of the managing authority. Where there is organization of a nation of the basis of
‘monopoly’ there is inefficiency due to lack of competition.
Public sectors thus, failed to produce profitable results and communism. 
created massive financial loss. There was price rise, capital Q.2. What does the term internationalism as a political
crunch, lack of technical know-how and slow pace in eco- process highlight?
nomic growth. The situation demanded Soviet Russia to (A) It highlights a process by which an individual
initiate economic reforms through “Perestroika”. More or or group comes to adopt increasingly extreme
less, India faced the same situation. To overcome the situa-
political, social, or religious ideals and
tion, Rao government adopted the new economic policy of
aspirations that reject or undermine the status
liberalisation and privatisement.
104 ENGLISH LANGUAGE
stportal.mahendras.org
quo or undermine contemporary ideas and (A) It stands for lack of technical know-how and
expressions of freedom of choice. slow pace in economic growth.
(B) It highlights increasing political, social, (B) It stands for the situation demanded by Soviet
economic and cultural relations among various Russia to initiate economic reforms through
nations of the world. “Perestroika”.
(C) It highlights the stage of human social (C) It stands for the abolition of protective economic
development and organization which is policies which were previously adopted in the
considered most advanced. national interest.
(1) Only (A) (2) Only (B) (1) Only (A) (2) Only (B)
(3) Only (A) and (B) (4) Only (B) and (C) (3) Only (C) (4) Only (A) and (B)
(5) All (A),(B) and (C) (5) All (A), (B) and (C)
Q.3. Which of the following statements is/are true Q.7. What does the word ‘Globalisation’ encourage as
according to the passage? mentioned in the passage?
(A) The sovereignty of the state is challenged with (A) Globalisation encourages free world trade.
the emergence of non-governmental power (B) Globalisation encourages free enterprise and
players in the world order. market forces.
(B) Globalisation is not an event, rather a process (C) Globalisation encourages the process of
integrating the domestic economy with the
which originated with the liberalisation and
world economy.
privatisation of the economic sectors.
(1) Only (A) (2) Only (B)
(C) Globalisation aims for the establishment of a (3) Only (C) (4) Only (A) and (B)
borderless world. (5) All (A), (B) and (C)
(D) Nations having socialistic economic system and Q.8. Which of the following statements is/are Not true
mixed economy failed to achieve targeted in context of the passage?
economic growth. (1) India became independent as one of the poorest
(1) Only (A) (2) Only (B) countries of the world.
(3) Both (A) and (B) (4) Both (B) and (C) (2) The British colonial rule had destroyed the self-
(5) All (A),(B),(C) and (D) sufficient agrarian economy.
Q.4. Why did public sectors face the loss in 1980s? (3) Entrepreneurs can have access to more capital
(A) Because of bureaucratic inefficiency and red- flows.
tapism. (4) The process of economic liberalisation began
(B) Because of political interference and corruption. as a precondition of globalisation.
(C) Because of irresponsibility of the managing (5) The western companies were in search of
authority. business locations in different parts of the world.
(1) Only (A) (2) Only (B) PASSAGE - 6
(3) Only (A) and (B) (4) Only (B) and (C)
Read the following passage carefully and answer the
(5) All (A),(B) and (C)
questions given below it.
Q.5. What did then government do to overcome the
Q.1-2. The Constitution of India is one of the greatest and
situation, as per the given passage?
unique Constitutions in the world due to its special
(A) The government adopted the new economic features. Framing of the Indian Constitution was
policy of liberalisation and privatisement. inspired by the Constitution of other countries.
(B) The government adopted the economic The Indian Constitution is one of the largest written
liberalisation that began as a precondition of Constitutions in the world. It has about 395 articles and 9
globalisation. schedules. The framers have not only included the basic
(C) The government created the trade barriers. principles of governance, but also made provisions to
(1) Only (A) (2) Only (B) deal with various administrative problems. Although
(3) Only (C) (4) Only (A) and (B) the Indian Constitution is a unique combination of both
rigidity and flexibility, the procedure for amendment
(5) Only (B) and (C)
of the Constitution is neither too rigid nor too flexible.
Q.6. What does Globalisation, as a negative process, The Constitution has adopted a parliamentary system
stand for? of government at center as well as in the state. The

ENGLISH LANGUAGE 105


stportal.mahendras.org
executive powers are exercised by the Council of (2) An integrated nation with heterogeneity on
Ministers handed by the Prime Minister. various aspects.
The secular nature of the state is one of the most (3) The country is divided into three classes: Upper,
important features of the Constitution. India has adopted middle and lower.
the principle of secularism and thus, making difference (4) The Minorities are provided with fundamental
between people based on religion is prohibited. Indian rights.
secularism provides freedom to practice any religion
(5) None of the above.
according to the individual’s choice.
Q.4. As per the perception extracted from the passage, it
Q.1. According to the passage, the detained traits of
is considered to be…..
Indian constitution are…
(1) A criticism of Indian culture
(A) It comprises rudimentary administrative essentials
(2) A Reflective composition of the writer
(B) The strategies to manage the complications of
administration. (3) A passage with analytical tone.
(C) The constitution is infused with dual quality (4) An introspective theory of Indian constitution.
(stiff and pliable) and States’ nonreligious (5) None of the above.
disposition Read the following passage carefully and answer the
(1) Only A (2) Only B questions given below it.
(3) Both A & B (4) All of the above Q5-6. The market system is driven by supply and demand.
(5) Can’t be inferred from the passage. Let’s take bread. People want more bread, meaning
the demand for bread is high. This demand means
Q.2. By understanding the passage, which of the following
you can charge more for bread, so you can make more
can be the intention of author present behind the writing?
money on average by changing wheat into bread than
(1) The author is commiserating with the Indian grinding that same wheat into flour. More people start
constitution for being one of the largest ones. making bread and, after a few production cycles,
(2) As various commendable features are in the there is so much bread in the market that prices
constitution, he is describing it in a laudatory tone. plummet. Meanwhile, the price of flour has been
(3) Biased description of the constitutional attributes. increasing as the supply shrinks, so more producers
buy up wheat for the purpose of making flour – and
(4) Both A and C
on and on.
(5) All of the above.
This extreme and simplified example does encapsulate
Read the following passage carefully and answer the the wonderful balancing act that is supply and
questions given below it. demand. The market is generally much more
Q.3-4. India is a secular nation. Supporters of every religion responsive in real life, and true supply shocks are
have been given the right to follow their religion rare – at least ones caused by the market are rare. On
freely. Religious minorities are given freedom to a basic level, supply and demand helps explain why
safeguard their culture, language and script. India is a last year's hit product is half the price the following
religiously diverse nation because followers of world’s year.
major religions reside in India. India has accepted the Q.5. Which of the following is the author least likely to
principles of secularism and thus, making difference agree with?
between people based on religion is prohibited. To
(1) Demand betokens an immense hike in the need
preserve culture, tradition, language and script, special
of any object.
Fundamental Rights are provided to the minorities.
(2) Once the market is excessively stocked, an
Indian society comprises three main classes. They an_
ascent in the prices of the items takes place.
upper class, middle class and lower class. One finds sub-
divisions among each class. These sub-divisions include (3) People are obligated to pay more for a particular
upper middle class and lower middle class. Some of item in case of demand goes up.
the other forms of classes include businesspersons and (4) Both A & B
workers, landowners and landless labourers. One can (5) Both A & C
also observe classes of literate as well as illiterate. These
Q.6. The word “encapsulate” in the passage is farthest in
classes show the diversity in class system.
meaning to:
Q.3. According to the passage, what may be the central
(1) Summarize (2) Put in a nutshell
idea of the passage?
(3) Abridge (4) Expand
(1) India is infused with concept of exclusion of
religion and religious considerations. (5) Control

106 ENGLISH LANGUAGE


stportal.mahendras.org

eXPLANATION
PASSAGE 1 PASSAGE 5
Q.1.(4) Q.2.(4) Q.3.(5) Q.4.(4) Q.5.(3) Q.1.(4) Answer lies in the beginning of the first paragraph.
Q.6.(1) Q.7.(2) Q.8.(4) Q.9.(4) Q.10.(2) Q.2.(2) Answer lies at last of the first paragraph.
Q.3.(5) Answer lies in the beginning of the second
PASSAGE 2
paragraph.
Q.1.(1) Hint: Answer lies in the beginning of the fourth Q.4.(5) Answer lies in the beginning of the third paragraph.
paragraph.
Q.5.(1) Answer lies at last of the third paragraph.
Q.2.(4) Hint: Answer lies in the beginning of the third Q.6.(3) Answer lies in the beginning of the fourth
paragraph. paragraph.
Q.3.(5) Hint: Answer lies at last of the third paragraph. Q.7.(4) Answer lies in the mid of the fourth paragraph.
Q.4.(5) Hint: Answer lies in the first paragraph. Q.8.(5)

Q.5.(2) Q.6.(5) PASSAGE 6


Q.1.(4) The answer can be inferred by understanding the
Q.7.(3) For other options
second paragraph.
Fervid(Adj)- passionate Q.2.(2) By understanding the whole passage, the answer
Q.8.(2) Q.9.(2) can be inferred.

Q.10.(3) For other options Q.3.(2) By reading and understanding the complete
passage, it can be clearly understood that the
Allure(V)-attract whole passage is describing the various cultures,
PASSAGE 3 religions and ideologies present in India.
Q.1.(3) Q.2.(5) Q.3.(2) Q.4.(3) Q.5.(4) Q.4.(3) In this passages, the author tries to justify to
support the Indian constitution, present the factors
Q.6.(4) Q.7.(4) Q.8.(3) Q.9.(4) Q.10.(5) and tell you why he feels something is better.
PASSAGE 4 Q.5.(5) By understanding the first paragraph, the answer
Q.1.(4) Q.2.(1) Q.3.(5) Q.4.(5) Q.5.(2) can be inferred.
Q.6.(2) Q.7.(1) Q.8.(1) Q.9.(4) Q.10.(2) Q.6.(4)

ENGLISH LANGUAGE 107


stportal.mahendras.org

FILL IN
CHAPTER

16 THE BLANKS
Scan the QR code to get video of this chapter.

Fill in the blanks is a type of question or phrase with one or more The article "an" will also give you an additional clue
types of words replaced with a blank line , giving the reader a because "an" indicates that the missing word starts with
chance to add the missing word (s) :- a vowel sound.
1. First read the sentence to yourself so you can hear what If a blank is preceded by a noun or a pronoun, it often
is being asked . Listen carefully to the way the sentence indicates a verb. Adverbs like "usually," "quickly," and
sounds each time and choose the option that sounds best "immediately" that appear before or after a blank also
in the sentence. indicate that the missing word is a verb. Focus on the
2. Try to quickly skim the text. words/terms given in the options.

3. Look for the clues in the sentence :- It's not just the words 6. Look at the context around the gap and try to predict the
directly before and after a blank that can provide clues. word or type of word that could logically fill each gap
Reading the entire sentence should help you figure out before you look at the word options.
the general topic or subject area. 7. Reject options that you know are not appropriate in terms
4. Make sure your choice fits logically and grammatically. of meaning or the grammatical context.

5. Try to figure out the part of speech: 8. Read each sentence to yourself several times, adding in
each of the remaining possible options one-by-one. This
The answers to the majority of fill in the blank questions will help you activate your knowledge of collocation.
will be verbs or nouns. An effective strategy for fill in
the blank questions is to try to determine which part of 9. Check the words on either side of each blank.
speech the missing word is. 10. If the sentence is of two blanks, then check the part of
If an article ("a," "an," or "the") appears directly before speech in the first option, if it doesn’t match ,that means
the blank, the missing word is almost certainly a noun. that option should be rejected and vice-versa.

108 ENGLISH LANGUAGE


stportal.mahendras.org

exercise
(C) do not need to be segregated
EXERCISE- 1
(1) Only (B) (2) Only (C)
Choose the most appropriate option among (A), (B)
(3) Both (A) and (B) (4) Both (B) and (C)
and (C) which completes the sentence and makes it
(5) None of them
meaningfully correct.
Q.6. Higher education attainment _______________, to
Q.1. Last night critics said that by raising the question
our economy and to individuals.
Facebook ran the risk of appearing to ____________
is a criminal offence. (A) is more importance than ever
(A) condoned such activity, which (B) is more much important then ever
(B) condone such activity, who (C) is much more important than ever
(C) condone such activity, that (1) Only A (2) Both A and C
(3) Only C (4) Both A and B
(1) Only (B) (2) Only (C)
(5) None of them
(3) Both (A) and (B) (4) Both (B) and (C)
(5) None of them Q.7. Many of the institutions targeted ___________
need-based aid, and have committed to do even
Q.2. GAIL owns a petrochemical plant in Uttar Pradesh,
more through programs like the American Talent
which ______________ the marketing unit to either
Initiative. 
IOC or BPCL.
(A) already spent significance amounts at
(A) could be sell along with
(B) already spend significant amounts on
(B) could be sold along with
(C) spend significant amounts earlier on
(C) could being sold along with
(1) Only A (2) Both A and C
(1) Only (B) (2) Only (C)
(3) Both (A) and (B) (4) Both (B) and (C) (3) Only C (4) Both B and C
(5) None of them (5) None of them
Q.3. Italy faces a hung parliament after populist parties Q.8. Just consider the effect of the legislation on India’s
surged in a general election characterized by anger long-term _________________ .
over _______________ .
(A) economic growth and competitiveness
(A) migrating and a stuttering economy
(B) economic surge and competitiveness
(B) migration and stutter economy
(C) economical growth and competitiveness
(C) migration and a stuttering economies
(1) Only A (2) Both A and C
(1) Only (B) (2) Only (C)
(3) Both (A) and (B) (4) Both (B) and (C) (3) Only C (4) Both A and B
(5) None of them (5) None of them
Q.4. The two populist parties showed that the wave of Q.9. What’s the common factor _____________
anti-system voting in Europe, _________________ avoidable tragedies? 
sparked Brexit, has triumphed in Italy. (A) in nearly every one of these
(A) who many observers believe (B) in near every one of these
(B) which much observers believe (C) in nearly every one of they
(C) which many observers believe (1) Only A (2) Both A and C
(1) Only (B) (2) Only (C) (3) Only C (4) Both A and B
(3) Both (A) and (B) (4) Both (B) and (C)
(5) None of them (5) None of them

Q.5. The company had earlier resisted the split on the Q.10. The National Rifle Association _____________ of
grounds that its gas marketing and transmission these free-for-all markets, and Congress isn’t willing
businesses operate at arm's length and, hence, to address the issue. 
_____________ . (A) continues to oppose any federal regulation
(A) do not needed to be separated (B) maintains to disapprove any federal regulation
(B) do not need to be separated (C) continues to opposite any federal regulation

ENGLISH LANGUAGE 109


stportal.mahendras.org
(1) Only A (2) Both A and C (3) impose (4) impel
(3) Only C (4) Both A and B (5) improve
(5) None of them
Q.10. The Managing Director treated the employees to a
EXERCISE- 2 _______ lunch at an expensive hotel.
Fill in the blanks in the following sentences with the most (1) Precious (2) Thriving
appropriate word from among those given words in options.
(3) Studpendous (4) Magnanimous
Q.1. Samples were collected and sent to _______ for (5) Sumptuous
testing.
(1) schools (2) jewellery EXERCISE- 3
Fill in the blanks in the following sentences with the most
(3) library (4) laboratories
appropriate word from among those given words in options.
(5) research
Q.1. ______ I not informed you about this, you would
Q.2. There are a large number of reasons which _______ have missed the examination.
people to buy gold coins during Dhanteras.
(1) If (2) Provided
(1) hinder (2) discourages
(3) Unless (4) Had
(3) foster (4) priority
(5) motivate (5) Have
Q.3. The forest department grants permission for _______ Q.2. The most encouraging fact is that the technology is
monkeys. spreading ______ the country and reaching villages
and small towns.
(1) capturing (2) diverting
(1) from (2) far
(3) guide (4) caught
(3) among (4) over
(5) attract
(5) across
Q.4. The professors of the college consider students their
______ and respect their views. Q.3. My teacher is one of the best teachers ______.
(1) equals (2) enemies (1) Who I know (2) Which I know
(3) rivalry (4) friendship (3) I had know (4) I know
(5) collegue (5) that I know
Q.5. The festival season brings ______ good cheer in the Q.4. His _______ contribution to the relief fund was
market. appreciated by his family.

(1) over (2) in (1) negligible (2) noteworthy

(3) at (4) from (3) spontaneous (4) generous

(5) for (5) meagre

Q.6. He likes spending money on _____property. Q.5. If only he ________ told us the truth in the first
place, things wouldn't have gone wrong .
(1) sold (2) speculate
(1) has (2) would have
(3) worth (4) buying
(3) had (4) should have
(5) manipulate
(5) could have
Q.7. People prefer performing puja in _____ style.
Q.6. The neighbour grabbed the girl, and rolled her on
(1) Artificial (2) traditional the road and ______ the flames.
(3) orthodox (4) show (5) uniqueness (1) smothered (2) extinguished
Q.8. He _____ his car outside the shop. (3) burn out (4) fizz
(1) Parked (2) throwing (5) move in
(3) burn (4) stoppage Q.7. Sometimes the greatest inventions ________ an idea
(5) vandalizing of starting simplicity.
Q.9. If you drink too much, it will _______ your (1) stumble (2) hinge upon
judgement. (3) starves without (4) lacks
(1) impede (2) impair (5) moved on
110 ENGLISH LANGUAGE
stportal.mahendras.org
Q.8. On dark night, Darvesh _____ passing by a day well. Q.7. An employment advertisement should ............the
(1) wasn't (2) happened to be number of vacancies.

(3) discovered in (4) found to (1) provide (2) declare

(5) would not (3) contain (4) specify

Q.9. Sam asked me to keep his secret ________. (5) focus

(1) secret (2) in myself Q.8. The family gave father a gold watch on the ............
of his fifteenth birthday.
(3) amongst us (4) between us
(1) time (2) event
(5) among them
(3) occasion (4) celebration
Q.10. It was ________ weather that she had ever
experienced. (5) rational

(1) cold (2) the worst Q.9. The passengers were afraid but the captain ............
them that there was no danger.
(3) good (4) bad
(1) promised (2) advised
(5) better
(3) assured (4) counselled
EXERCISE- 4 (5) provided
Fill in the blanks in the following sentences with the most
Q.10. It’s very kind of you to ............ to speak at the
appropriate word from among those given words in options.
meeting.
Q.1. The government is confident that the standard of
(1) comply (2) agree
living will begin to ........ again soon.
3) accept (4) concur
(1) rise (2) lift
(5) delivered
(3) flourish (4) revive
(5) specific EXERCISE- 5
Q.2. On second reading his poems strike us as Fill in the blanks in the following sentences with the most
singularly .......... of sublime emotions. appropriate word from among those given words in options.

(1) attributive (2) significative Q.1. The primary purpose of modern weapons is ............
to prevent a particular course of action by a specific
(3) symptomatic (4) evocative threat.
(5) movement (1) deterrent (2) prognostic
Q.3. Health is too important to be .......... (3) minatory (4) hegemony
(1) neglected (2) discarded (5) unbelieveably
(3) despised (4) detested Q.2. As for the free world, trade with Cuba as been taking
(5) tell place on a modest scale despite the .............
Q.4. Even a ......... glance will reveal the mystery. opposition of the United States.

(1) crude (2) cursory (1) casual (2) independent

(3) critical (4) curious (3) clandestine (4) overt

(5) prefer (5) urge

Q.5. Like any other country India has its ............ share Q.3. The basic structure of the living cell is a problem
of superstitions. whose ................ can be judged by reference to the
difficult exploration of the structure of the atom.
(1) abundant (2) fair
(1) importance (2) universality
(3) proper (4) peculiar
(3) complexity (4) antiquity
(5) proportion
(5) clarity
Q.6. Hindus believe that ....... from the cycle of birth and
rebirth can be attained only by good deeds. Q.4. The endless battle to modernise the .............
structure of work rules on the nation's railroads
(1) bondage (2) deliverance appears destined to reach the showdown stage
(3) delivery (4) retirement with a strike at one minute after midnight
tonight.
(5) renewal
ENGLISH LANGUAGE 111
stportal.mahendras.org
(1) anemic (2) impracticable (1) capacity (2) ongoing
(3) archaic (4) streamlined (3) tendency (4) potential
(5) serious
(5) markedly
Q.2. I. India’s economic rise since 1991 has ____ in
Q.5. Cyprus is still not economically viable, a sharp rise in resource needs, from petroleum
though............... important, it is militarily weak in products to power and infrastructure.
its own right.
II. Regular trips back to her motherland have _____
(1) necessarily (2) strategically in her first book My Sweet Dreamland.
(3) scarcely (4) independently (1) transformed (2) resulted
(5) cruely (3) transcribed (4) translated
Q.6. He's gone through a ................ He is not at all the (5) brought
man he was when he was a combat officer.
Q.3. I. Nehru may have had ____ for Akbar and the great
(1) metamorphosis (2) crisis Mughals’faith in religious tolerance, but Nehru’s
(3) frustration (4) surveillance single-minded emulation of Asoka left no room
(5) Examination for Akbar’s own heroic attempt to foster religious
coexistence.
Q.7. It was Jacob Grimm who transformed philology
from an .............. study in to an exact science. II. I have always had the greatest _____ for him.

(1) abstruse (2) alleged (1) admiration (2) contempt

(3) esoteric (4) errant (3) good will (4) love

(5) crucial (5) perception

Q.8. As the waves rose and the ship tossed, many of the Q.4. I. The Empowered Group of Ministers on telecom
passengers felt............ is _____ to meet on Friday.

(1) lethargic (2) subdued II. No new talks are _____.

(3) tremulous (4) queasy (1) fixed (2) going

(5) uneasily (3) scheduled (4) discussing

Q.9. Although advertising men often complain that their (5) sure
industry is hemmed in by government regulations, Q.5. I. Buyers can look for deals _____ a variety of
the fact remains that a/an ............... attitude toward product ranges.
Madison Avenue continues to exist in this country. II. The wind pushed her hair _____ her face.
(1) laissez faire (2) savoir faire (1) in (2) among
(3) bete noire (4) idee fixe (3) on (4) about
(5) allusion (5) across
Q.10. The knockout wallop travelled only seven or eight Q.6. The Prime Minister ____ the jump in India’s ranking
inches and, admittedly, did not look like much. But as historic and said it was a ____ of all-round and
boxing experts, and ............. scientists, will multi-sectoral reform push.
attest that punches that travel more than a foot lose
much of their initial force, (1) hailed, result

(1) nuclear (2) biological (2) accepted, representation


(3) electronic (4) kinetic (3) acclaimed, cause
(5) successful (4) rejected, evolution
EXERCISE- 6 (5) declined, consequence
Fill in the blanks in the following sentences with the most Q.7. The panel has decided to have a thorough ____ at
appropriate option. electoral reforms, delay in appointment of judges
Q.1. I. For a country with one of the largest reserves of and the problems in the public grievance _____
natural resources in the world, the transformational mechanism.
_____ of India’s resources sector is immense. (1) attention, satisfaction
II. We are aware of the _____ problem and have (2) examination, elimination
taken every precaution.
112 ENGLISH LANGUAGE
stportal.mahendras.org
(3) study, reforms Q.4. The...........of democratic freedom is dialogue and
(4) focus, solution the ............. interchange of diverse ideas.

(5) look, redressal (1) deterioration, untrammeled

Q.8. Avigorous Nort-East monsoon _____ to intensity (2) height, restrained


over Tamil Nadu and Puducherry, _____ heavy to (3) essence, unhampered
very heavy rain at a few places and extremely heavy (4) alienation, compulsory
rain in isolated places.
(5) epitome, discriminating
(1) rose, giving
Q.5. Although there were............circumstances in this
(2) increased, dropping particular violation of the law, the judge ruled that
(3) went down, showering there had to be strict............or there would be no law
(4) scaled up, delivering at all.

(5) declined, pouring (1) extraordinary, complaisance

Q.9. A mechanical engineer _____ training, he joined Tata (2) specific, obedience
Steel in 1988 after _____ his management education (3) tantalising, adherence
from IIM Calcutta. (4) extenuating, compliance
(1) in competing (2) by, completing (5) questionable, observation
(3) of, doing (4) for, joining Q.6. In spite of all............,when cheaters were caught,
(5) with, taking there is evidence of the ..... of the rigid rules of
Q.10. ......of crops was due to continuous............... external help on civil service tests.

(1) Loss, draught (2) Ruin, draft (1) threats, encompassment


(3) Failure, drought (4) Depreciation, drift (2) surveillance, vulnerability
(5) None of these (3) temptation, flouting
EXERCISE- 7 (4) precautions, circumvention
In each of the following sentences there are two blank (5) discouragement, acceptance
spaces. Below each sentence there are four or five pair of
Q.7. Hungarians may grumble about the difficulty
words denoted with numbers (1), (2), (3), (4) and (5). Find
of acquiring cars, but they point quickly to a
out which pair of words can be filled up in the blanks in
compensation: the...............look of their tree-
the sentence in the same sequence to make the sentence
lined avenues and the absence of ............. exhaust
meaningfully complete.
fumes.
Q.1. It is a well-known............that the lover of the sea
(1) calm, superfluous (2) otiose, poisonous
craves for dry land, the age-old........to be where we
are not. (3) tranquil, copious (4) anomalous, fetid
(1) belief, antipathy (2) anomaly, delirious (5) uncluttered, noxious
(3) credo, inspiration (4) contention, duplicity Q.8. In architecture, much more than in any of the other
fine arts, there is a marked time lag between the
(5) paradox, yearning
.................. of ideas and their ............... in the shape
Q.2. Why should a university...............the values of completed building.
which are supposedly basic to its functioning and
(1) settlement, fruition
give an honorary doctorate to one who has not
distinguished himself in a/an ............. manner? (2) creation, welcoming
(1) deflate, scholarly (2) foretell, worldly (3) tradition, modernization
(3) vilify, collegiate (4) abuse, doctrinaire (4) dawn, practicability
(5) debase, academic (5) emergence, application
Q.3. Such stalling tactics are...........to all fans and cannot Q.9. In analyzing the.............teenage population,
be........... . Madison Avenue has decided that it is eminently
receptive to the ............... of advertising.
(1) repugnant, condoned (2) anathema, ascertained
(1) proliferating, become
(3) injurious, explained (4) unfair, superseded
(2) ubiquitous, mystique
(5) understandable, countenanced
ENGLISH LANGUAGE 113
stportal.mahendras.org
(3) diminishing, fantasy (1) rare, escape (2) outdated, know
(4) burgeoning, blandishments (3) volatile, notice (4) powerful, protect
(5) extinct, enjoy
(5) viable, syndrome
Q.7. We cannot....such a/an.....act of violence.
Q.10. No matter how ......... the Russians are of wish to
appear, they, as well as we, know that to survive it is (1) tolerate, insipid (2) consider, important
necessary to reach agreements which may .............. (3) commit, magnificent (4) pardon, egregious
mutual sacrifices. (5) neglect, insignificant

(1) fatuous, necessitate Q.8. The secretary.........the society’s funds,.....he was


dismissed.
(2) bellicose, evade
(1) misplaced, soon (2) rolled, thus
(3) refractory, obliterate (3) pirated, therefore (4) misappropriated, so
(4) indifferent, subsume (5) Continued, for
(5) intransigent, entail Q.9. The.......words of the mother comforted the
........child.
EXERCISE- 8
(1) harsh, naughty (2) sweet, happy
In each of the following sentences there are two blank spaces. (3) soft, energetic (4) melodious, playful
Below each sentence there are four or five pair of words (5) salty, sad
denoted with numbers (1), (2), (3), (4) and (5). Find out which
word can be filled up in the blanks in the sentence in the Q.10. Santosh looked very happy and.........when he heard
same sequence to make the sentence meaningfully complete. that his proposed scheme was.......by the committee.
Q.1. They wanted to....all these books, but they could not (1) energetic, rejected
find.....time to do so. (2) elated, accepted
(1) cover, almost (2) pursue, necessary (3) satisfied, stalled
(3) dispose, some (4) read, sufficient (4) disconsolate, approved
(5) buy, some
(5) overwhelming, received
Q.2. Though he is reputed for his technical...., his books
were sadly........of the works of others as he lacked EXERCISE- 9
originality. In each of the following sentences there are two blank
(1) advice, unconscious (2) skill, independent spaces. Below each sentence there are four or five pair of
words denoted with numbers (1), (2), (3), (4) and (5). Find
(3) knowledge, ignorant (4) expertise, derivative out which pair of words can be filled up in the blanks in
(5) dependence, indicative the sentence in the same sequence to make the sentence
Q.3. He had managed to........several times, but was meaningfully complete.
finally........by the police. Q.1. A ..............is a.................... .
(1) deceive, cheated (2) defend, acquitted (1) norm, standard (2) knowledge, mistake
(3) escape, arrested (4) cheat, robbed (3) student, school (4) doctrine, follower
(5) thesis, superstition
(5) abscond, kidnapped
Q.2. A ...........shade of distinction is a................. .
Q.4. The candidate’s .......at the polls was.....as he won
with a striking margin. (1) beautiful, vindication (2) complete, profanity
(3) subtle, nuance (4) thorough, prejudice
(1) claim, unrealistic (2) victory, overwhelming (5) entire, paradox
(3) image, real (4) strategy, unsuccessful Q.3. .............flattery is known as......................
(5) candidature, inappropriate (1) Regular, maturity
Q.5. In......of international matters, there is always an (2) Indiscriminate, encomium
element of risk in.....one might do.
(3) Servile, adulation
(1) view, whichever (2) many, doing
(3) defence, wrong (4) case, whatever (4) Unasked for, gratitude
(5) spite, whatever (5) Cowardly, temerity
Q.6. We must prevent endangered wild animal Q.4. At some private schools, pupils are under the
species from becoming.....in order that our future ............. of a ........... .
generation may.....the great diversity of animal life.
114 ENGLISH LANGUAGE
stportal.mahendras.org
(1) guidance, palladium (2) tutelage, teacher (4) incontrovertibly, viable
(3) coaching, verity (4) assiduity, palladium (5) unequivocally, mobile
(5) consensus, mentor Q.2. Human memory is not.............especially on .............
Q.5. A large..............centre is a/an............. . happenings.

(1) district, affliction (2) transport, automobile (1) infallible, unexpected (2) dependable, simple

(3) civic, utarchy (4) shopping, emporium (3) confidant, fanciful (4) reliable, inventive

(5) educational, indignity (5) noteworthy, fanciful

Q.6. ...................persons are inclined to.............. . Q.3. Athletes have so perfected their techniques in track
and field events that the ........... becomes ............
(1) Obese, corpulence before record books can be published.
(2) Generous, leanness (1) announcement, public
(3) Domineering, temperance (2) meet, official
(4) Vacillating, determination (3) time, authentic
(5) Cowering, effrontery (4) fantastic, common place
Q.7. A ..............is a temporary................. . (5) result, universal
(1) deviation, rest (2) shambles, journey Q.4. Like the .......... part of an iceberg, much of what is
(3) respite, relief (4) paradox, enchantment really interesting in the capital is not......
(5) feint, spell (1) inner, known (2) submerged, visible
Q.8. .............language may also be termed............... (3) greater, dangerous (4) upper, viable
(1) Eloquent, exiguous (5) lower, penetrable
(2) Frenzied, placid Q.5. Hence the word sophistry has an unfavourable
(3) Abusive, scurrilous ......... and means arguing deceitfully, attempting to
turn a poor case into a good one by means of clever
(4) Contumacious, flattering but................ reasoning.
(5) Denunciatory, peripatetic (1) denotation, ingenuous
Q.9. A..............of small stones is called a................ (2) meaning, ingenious
(1) design, numismatist (2) collar, tiara (3) connotation, specious
(3) seller, connoisseur (4) mound, lithograph (4) significance, vague
(5) pattern, mosaic (5) impact, cogent
Q.10. Suzerainty is..............control over a ............... state. Q.6. He warned the workers against the anti-social
(1) full, democratic (2) domestic, backward policies, which he declared would ............rather
(3) central, unified (4) political, dependent than.............the plight of the common people.
(5) economic, federate (1) rescue, destroy
EXERCISE- 10 (2) encourage, defy
In each of the following sentences there are two blank (3) aggravate, alleviate
spaces. Below each sentence there are four or five pair of
(4) empower, improve
words denoted with numbers (1), (2), (3), (4) and (5). Find
out which pair of words can be filled up in the blanks in (5) protract, inhibit
the sentence in the same sequence to make the sentence EXERCISE- 11
meaningfully complete.
Q.1-5. In the following sentences there are two blank spaces
Q.1. To encourage colonial peoples in their aspirations and five pair of words have been denoted in the
to attain independence before it is ........proved that table match the suitable pair. Find out which pair of
a..............state will evolve instead of anarchy is words can be filled in the blanks in order to make a
unforgivable. meaningful sentence.
(1) unchanged, formidable Q.1. The students were asked to __________ and then
(2) certainty, sympathetic exhale forcefully into a testing device to check their
(3) succinctly, redoubtable lung__________.

ENGLISH LANGUAGE 115


stportal.mahendras.org

Column 1 Column 2 Column 1 Column 2


A. join D. power A. thought D. purify
B. planned E. immaculate
B. focus E. capacity
C. suggested F. clean
C. inhale F. part (1) A-E (2) B-E
(1) A-E (2) B-E (3) C-F (4) A-F
(3) C-D (4) A-F (5) C-E
(5) C-E EXERCISE- 12
Q.2. Japan was among the first countries to __________ its Q.1-5. In the following questions, two sentences are given
vehicle manufacturers to do end-of-life __________ with two blanks and against them two pair of
of their merchandise. same words (A) & (B) but in different order are
Column 1 Column 2 given in each option, Choose the correct option
A. mandate D. recycling of which both the words of each pair can be filled
in the blanks respectively and complete both the
B. force E. convert sentences meaningfully.
C. panache F. flamboyant
Q.1. (A) The Gita tries to ________ the individual’s
(1) A-E (2) B-E attention towards a sincere examination of the
(3) C-D (4) A-D worth of one’s goals and desires by using the
most valuable gift ‘the _______ ’ to analyze and
(5) C-E see things in proper perspective.
Q.3. High-altitude areas in Jammu ___________ the (B) It is foolish to __________ into life seeking
season's first heavy snowfall which led to closure goals that only leave one stressful and
of the Mughal Road, the alternate link _________ dissatisfied, for desires that remain unfulfilled
Kashmir and rest of the country, for vehicular are the main _________ of sorrow and
traffic. problems in life.
Column 1 Column 2 (1) A- plunge, intellect B- plunge, redirect,
A. witnessed D. during (2) A- redirect , intellect B- plunge , cause
B. observed E. nearly (3) A- plunge, redirect B- cause, intellect
C. spotted F. between
(4) A- redirect, cause B- plunge, intellect,
(1) A-E (2) B-E (5) No pair is correct
(3) C-D (4) A-F Q.2. (A) Current history, which looks episodic, is the
(5) C-E _______ stages of a very long-term policy
Q.4. The NIA has __________ a case in connection with moving internationally, but ______ only in the
the killing of RSS leader nearly a month after the long perspective of time.
government decided to hand over the __________ (B) We need not confine the basic elements of
to the central probe agency. ________ thoughts to modern times, especially
when we ________ the roles and status of
Column 1 Column 2 women in our own modern societies.
A. lodged D. analysis
(1) A- primitive, contemplate
B. reported E. investigation
B- culminating, visible
C. registered F. inspection
(2) A- culminating, visible
(1) A-E (2) B-E
B- primitive, contemplate
(3) C-D (4) A-F
(3) A- contemplate, visible
(5) C-E
B- primitive, culminating
Q.5. A survey conducted by Heal foundation __________
that a majority of people think it is the whole and (4) A- culminating, contemplate
sole responsibility of the government to ________ B- primitive, visible
the air. (5) No pair is correct

116 ENGLISH LANGUAGE


stportal.mahendras.org
Q.3. (A) If one attains mystical insight by divine
EXERCISE- 13
intervention, it is likely to ______ in a closed
system of thought, meaning a system that claims Q.1-5. In the following questions, two sentences with
to _____ all the necessary knowledge for proper blanks have been given. These sentences are
conduct of life. followed by five alternatives. Choose the one
which fits into both the sentences-:
(B) Happiness has _________ something that
can be monitored and measured, including Q.1. I. Meteorological conditions have slightly improved
by our behavior, use of social media and causing ______ of pollutants but the air quality
bodily ________ such as pulse rate and facial might deteriorate with dense fog engulfing the
expressions. capital which may degrade the air quality.
(1) A- become, possess B- ensue, indicators II. The company broke the law when it decided for
__________ of hazardous waste into the ocean.
(2) A- ensue, indicators B- become possess
(1) Spread (2) Accumulation
(3) A- ensue, possess B- become, indicators
(3) Dissemination (4) Divergence
(4) A- possess, ensue B- indicators, become
(5) Dispersion
(5) No pair is correct
Q.2. I. There are movies which we should watch with
Q.4. (A) Our huge problem with plastic is the result
kids to ____________and inspire in them
of a permissive legal framework that has
virtues and humanitarian ethics.
__________ the uncontrolled rise of plastic
pollution, despite clear ________ of the harm II. With his efforts, he has brought political and
it causes to local communities and the world’s judiciary action that ___________ media in
oceans. favor of the most deprived section of the society.
(B) Economists have _________ most of the (1) Refine (2) Sensitize
20th century ignoring psychology, positive (3) Petrify (4) Pacify
or otherwise but today there is a great deal of
(5) Benumbed
_________ on how happiness can shape global
economies on a smaller scale. Q.3. I. There will be a ‘boomerang effect’ on regulation
with more __________ policy to compensate
(1) A- evidence, emphasis
for the current weak regulation.
B- legitimatized, depleted
II. The Government needs to be more ______ for the
(2) A- legitimatized, emphasis security measures concerned so far in the country.
B- depleted, evidence (1) Stringent (2) Mild
(3) A- depleted, evidence (3) Pliant (4) Liberal
B- emphasis, legitimatized (5) Sensitive
(4) A- legitimatized, evidence Q.4. I. Rural distress, farm discontent, and “arrogance
B- depleted, emphasis of power” are being cited as reasons for the ruling
party’s impending _______ in Chhattisgarh.
(5) No pair is correct
II. The views expressed by the orator, it was called
Q.5. (A) Encouraging individuals to __________ more
a ___________by the critics.
will never solve the problem of a massive
production of single-use plastic that should (1) Tumultuous (2) Debacle
have been __________ in the first place. (3) Debonair (4) Ignominy
(B) Scientists have long recognized that plastics (5) Ubiquity
_______ slowly, if at all, and pose multiple
Q.5. I. We should go into the details on why there was
__________ to wildlife through entanglement
a/an___________ which forced him to take this
and consumption.
ultimate decision of resignation.
(1) A- biodegrade, avoided B- recycle, threats
II. The political parties have reached a/an
(2) A- recycle, avoided B- threats ,biodegrade ________ in the present situation.
(3) A- recycle, avoided B- biodegrade, threats (1) Implausible (2) Impasse
(4) A- threats, avoided B- recycle, biodegrade (3) Breakthrough (4) Plight
(5) No pair is correct. (5) Standstill

ENGLISH LANGUAGE 117


stportal.mahendras.org

EXPLANATION
Impel - if an idea or feeling impels you to do
EXERCISE 1
something.
Q.1.(2) Option (2) condone such activity, that
eg. He felt impelled to investigate further.
Q.2.(1) Option (1) could be + v3
Q.10.(5) sumptuous - very expensive and looking very
Q.3.(5) Option (5) migration and a stuttering economy impressive.
Q.4.(2) Option (2) which many observers believe, For other options :
observers are countable noun, so plural verb is
thriving- becoming successful, strong, healthy.
used.
stupendous - extremely large or impressive.
Q.5.(4) Both (B) and (C), because both follow the correct
grammatical structure. magnanimous - Kind, generous and forgiving.
Q.6.(3) EXERCISE 3
Q.7.(4) Q.1.(4) Q.2.(5) Q.3.(5)
Q.8.(4) Q.4.(4) For other options :
Q.9.(1) Spontaneous - not planned but done because you
suddenly want to do it.
Q.10.(4)
meagre - small in quantity and poor in quality.
EXERCISE 2
Q.5.(3)
Q.1.(4) laboratories - a room or building used for scientific
research. Q.6.(2) extinguish - to make a fire stop burning or a light
stop shining.
Q.2.(5) For other options:
For other options :
hinder - to make it difficult for somebody to do
something. smothered - to kill somebody by covering their
face so that they cannot breathe.
foster - to encourage something to develop.
Eg. He smothered the baby with a pillow.
Q.3.(1) capture - to catch a person or an animal as a
prisoner. fizz- the small bubbles of gas in a liquid.
For other options: Q.7.(2) hinge upon - to depend on something completely.
diverting - entertaining and amusing. For other options :
Q.4.(1) For other options: stumble - to hit your foot against something while
you are walking or running.
rivalry - a state in which two people are competing
for the same thing. Q.8.(2) Q.9.(4) Q.10.(2)
Q.5.(2) EXERCISE 4
Q.6.(4) For other options: Q.1.(1) For other options :
speculate - to buy property or goods hoping to flourish - to develop quickly and be successful or
make a profit when you see them but at a risk. common.
Q.7.(2) traditional - being part of the beliefs, customs. revive- to become, or to make somebody/
something become strong again.
For other options:
Q.2.(1) attributive - (attribute :- feature, character) used
orthodox - following closely the traditional beliefs
before a noun to describe it.
and practices of a religion.
For other options :
Q.8.(1) For other options:
significative - implied, ellusive.
vandalizing - to damage something, especially
public property, deliberately and for no good symptomatic - being a sign of an illness or a
reason. problem.
Q.9.(2) Impair - to damage something or make something evocative - making you think of or remember a
worse. strong image or feeling.
For other options: Q.3.(1) For other options :
impede - to delay or stop the progress of discarded - to get rid of something that you no
something. longer want or need.
Impose - to force somebody to have to deal with despised - disliked and have no respect.
something difficult.
118 ENGLISH LANGUAGE
stportal.mahendras.org
Q.4.(2) cursory - done quickly and without giving enough Eg. I started to feel queasy as soon as the boat left
attention to details. the harbour.
Q.5.(2) For other options : For other options:
peculiar- strange or unusual lethargic-lacking in energy.
Q.6.(2) deliverance - the state of being rescued from subdued - unusually quiet or unhappy.
danger, evil or pain. tremulous - Eg. He watched her tremulous hand
For other options : reach for the teacup.
bondage - the state of being a slave or prisoner. Q.9.(1) laissez faire - if a government is laissez faire, it
Q.7.(4) does not have many laws and rules which control
economy.
Q.8.(3) For other options :
For other options:
rational - based on reason rather than emotions.
savoir faire - The ability to do and say the right
Q.9.(3) Q.10.(2)
thing in any social situation.
EXERCISE 5 bete noire - a person or thing that you particularly
Q.1.(1) deterrent dislike or that annoys you.
Eg. hopefully his punishment will act as a deterrent Q.10.(4) Kinetic-involving or producing movement.
to.
EXERCISE 6
For other options :
Q.1.(4) Q.2.(2) Q.3.(1)
prognostic - (prognosis) an opinion, based on
Q.4.(3) Q.5.(5) Q.6.(1)
medical experience.
Q.7.(5) Q.8.(4) Q.9.(2)
minatory- threatening
Q.10.(3)
hegemony - control by one country, organisation
etc. over other countries etc. within a particular EXERCISE 7
group. Q.1.(5) paradox - A situation or statement which seems
Q.2.(4) overt-done in an open way and not secretly. impossible or is difficult to understand.
For other options : yearning - desiring very strongly, especially
clandestine - done secretly or kept secret. something that you cannot have.
Q.3.(3) For other options: For other options:
antiquity - The ancient past. antipathy - strong dislike, opposition or anger.
Eg. The statue was brought to Rome in antiquity. anomaly - a person or thing that is different from
what is usual.
Q.4.(3) archaic - very old - fashioned.
delirious - unable to think or speak clearly because
For other options: of fever, excitement or mental confusion.
streamlined - To give something a smooth even Credo - a set of beliefs
shape so that it can move quickly and easily
through air or water. contention - angry disagreement between people.
Q.5.(2) strategically - (adv.) (Strategic - done as a part of a Q.2.(5) debase - to make something or somebody less
plan that is meant to achieve a particular purpose.) valuable or respected.
Q.6.(1) metamorphosis - a process in which somebody For other options:
or something changes completely into something foretell - to know or say what will happen in the
different. future.
For other options: vilify - to say or write unpleasant things about
surveillance - The act of carefully watching a something or somebody so that other people will
person suspected of a crime. have a low opinion of them.
Q.7.(1) esoteric - very unusual; understood or liked by doctrinaire - strictly following a theory in all
only a small number of people. circumstances.
For other options: Q.3.(1) repugnant - making you feel strong dislike or
disgust.
abstruse - difficult to undestand.
condoned - to accept behaviour that is morally
Q.8.(4) queasy - likely to vomit wrong.

ENGLISH LANGUAGE 119


stportal.mahendras.org
For other options: Superflous - more than you need or want.
anathema - a thing or an idea which you hate. Otiose - having no useful purpose.
because it is the opposite of what you believe. Tranquil - quiet and peaceful.
superseded - to take the place of something that anomalous - different from what is normal or
is considered to be old fashioned expected.
countenanced - to support something or agree to fetid - smelling very unpleasant.
something happening.
Q.8.(5) emergence : to start to exist.
Q.4.(3) essence - The most important quality or feature of
application - practical use of something.
something.
dawn - The time of day when light first appears.
unhampered - it is opposite of hamper.
For other options :
hamper means to prevent somebody from.
Fruition - The successful result of a plan.
doing and achieving something.
Q.9.(4) Burgeoning - to begin to grow or develop rapidly,
For other options:
Blandishments - pleasant things that you say to
untrammeled - not restricted or limited.
somebody.
Deterioration - to become worse.
For other options:
restrained - restricted
Proliferating - Increasing rapidly
alienation - make somebody feel that they do not
Ubiquitious - seeming to be everywhere.
belong in a particular group.
Mystique - The quality of being mysterious or
epitome - a perfect example of something.
secret that makes somebody seem interesting.
Q.5.(4) Extenuating - showing reasons why a wrong or
diminishing-To become smaller, weaker etc.
illegal act, should be judged less seriously or
excused. fantasy - a pleasant situation that you imagine but
that is unlikely to happen.
compliance - The practice of obeying rules or
requests made by people in authority. viable - that can be done.
For other options: syndrome - a set of physical conditions that show
you have a particular disease.
complaisance - ready to accept other people’s
actions and opinions. Q.10.(4) indifferent - having or showing no interest in
somebody/something.
tantalising - to make a person want something that
they can not have. subsume - to include something in a particular
group.
Adherence - The act of behaving according to a
particular rule. For other options:
Q.6.(4) Precautions - something that is done in advance intransigent - unwilling to change their opinions
in order to prevent problems or to avoid danger. or behavious in a way that would be helpful to
others.
circumvention - to find way of avoiding a
difficulty. bellicose-having or showing a desire to argue.
For other options: evade - to escape from somebody/something.
encompassment - to include a large number refractory - difficult to work
surveillance - The act of carefully watching a obliterate - To wipe out
person suspected of a crime. fatuous - Foolish or silly.
vulnerability - weak and easily hurt physicially or EXERCISE 8
emotionally.
Q.1.(4) For other options:
Temptation - The desire to do or have something
Pursue - to do something or try to achieve
that you know is bad or wrong.
something over a period of time.
Flouting - to show that you have no respect for a
Q.2.(4) expertise - Expert knowledge or skill in a particular
law etc.
subject, activity or job.
Q.7.(5) Uncluttered - not containing too many objects,
Q.3.(3) For other options:
details or unnecessary items.
acquitted-To decide and state officially in court
Noxious - poisonous or harmful.
that somebody is not guilty of a crime.
For other options:
120 ENGLISH LANGUAGE
stportal.mahendras.org
For other options: assiduity (noun)-working very hard and taking
abscond - to escape from a place that you are not great care that everything is done as well as it can
allowed to leave without permission. be.

Q.4.(2) Q.5.(4)

Q.5.(4) For other options: Q.6.(1) corpulence (noun)-people say ‘corpulent’ to avoid
saying ‘fat’.
Spite - a feeling of wanting to hurt or upset
somebody. Q.7.(3) respite-a short delay allowed before something
difficult or unpleasant must be done.
Q.6.(5) For other options:
feint - to confuse your opponent by making them
Volatile - changing easily from one mood to think you are going to do one thing when you are
another. really going to do something else.
Q.7.(4) egregious - extremely bad. Q.8.(3) scurrilous - very rude and insulting.
For other options: For other options:
Insipid - to demand that something happens or contumacious-lacking respect for authority.
that somebody agrees to do something.
peripatetic - going from place to place.
Q.8.(4) For other options:
Q.9.(5) For other options:
pirated - copied and used or sold somebody’s
numismatist - a person who collects or studies
work or a product without permission and without
coins or medals.
having the right to do so.
connoisseur - an expect on matters involving the
Q.9.(4)
judgement of beauty, quality or skill.
Q.10.(2) elated - very happy and excited because of
lithograph- a picture printed by lithography.
something good that has happened or will happen.
Q.10.(4) For other option
For other options:
federate- to unit under a central government or
stalled - to stop suddenly because of a lack of
organisation while keeping some local control.
power or speed.
Eg. Discussions have once again stalled. EXERCISE 10
disconsolate - very unhappy and disappointed. Q.1.(4) For other option
Unequivocally (adj)- expressing your opinion or
EXERCISE 9 intention very clearly and firmly.
Q.1.(1)
Q.2.(1) Infallible- never wrong, never making mistake.
Q.2.(3) subtle - not very noticeable or obvious.
For other option :
For other options:
Confidant- a person that you trust and who you
profanity - behaviour that shows a lack of respect talk to about private or secret things.
for God or holy things. Q.3.(5) Q.4.(2)
Q.3.(3) Servile - wanting too much to please somebody Q.5.(3) Connotation- an idea suggested by a word in
and obey them. addition to its main meaning.
adulation - admiration and praise. Specious-seeming right or true but actually wrong
For other options: or false.
encomium - a speech or piece of writing that praises For other options :
somebody or something highly. t e m e r i t y - denotation- the act of naming something with a
extremely confident behaviour that people are word.
likely to consider rude.
ingenuous- honest, innocent and willing to trust
Q.4.(2) tutelage - the state of being protected or controlled people.
by another person, organization or country.
ingenious- having a lot of clever new ideas and
For other options: good at inventing things.
palladium - a chemical element. Cogent- strongly and clearly expressed in a way
verity - a belief or principle about life that is that influences what people believe.
accepted as tree.

ENGLISH LANGUAGE 121


stportal.mahendras.org
Q.6.(3) Q.3.(3) For other options:
EXERCISE 11 Ensue (v): happen or occur as a result.
Q.1.(5) For other options: Q.4.(4) For other options:
Inhale (v)- breathe in (air, gas, smoke, etc.) Deplete (v): consume, spend
Q.2.(4) For other options: Legitimatize (v): allow, grant
Panache (n)- flamboyance, confidence Q.5.(3)
Flamboyant (adj.)- exuberant, confident EXERCISE 13
Q.3.(4) For other options: Q.1.(5) For other options:
Spot (v)- allow (an advantage) to (someone) in a Dispersion (n): throwing or relieving something
game or sport. into large or wide area.
Q.4.(5) For other options: Q.2.(2) For other options:
Lodged (Adj.)- (of a crop) flattened by wind or Petrify (v)- terrify
rain. Pacify (v)- calm, appease
e.g. - in lodged crops there is rapid leaf decay Benumb (v)- to make inactive especially by cold.
Q.5.(3) For other options: Q.3.(1) For other options:
Immaculate (adj.)- clean, spotless Pliant (adj.)- easily influenced or directed
EXERCISE 12 Q.4.(2) Debacle (n)- a complete failure, fiasco
Q.1.(2) For other options: For other options:
Intellect (n): a person's mental powers. Tumultuous(adj.)- excited, confused
Plunge (v): fall suddenly and uncontrollably. Debonair(adj.)- an attractive, confident man
Q.2.(2) For other options: Ubiquity(n) – being very common
Primitive (adj.): having been in existence for a Ignominy(n)- humiliation
very long time.
Q.5.(2) Impasse(n)- deadlock, no progress possible
Culminate (v): reach a climax or point of highest
Plight(n) – a dangerous, difficult situation.
development.
Contemplate (v): look thoughtfully

122 ENGLISH LANGUAGE


stportal.mahendras.org

CHAPTER
PHRASE
17 REPLACEMENT
Scan the QR code to get video of this chapter.

Sentence correction questions are designed to test a questions. It can keep you away from making careless
candidate’s ability to identify written English that is mistakes that would harm your score.
grammatically-correct. Each question will begin with (7) Don’t forget about the logic of the sentence.
sentences, parts of which have been underlined. You will
then be presented with 5 different answer choices presenting When down to those last two options, plug each one
alternative ways of stating the underlined portion of the text. back into the sentence and see which one makes more
sense. Check for clear and logical modification.
The correct answer will have all of the following 3
characteristics: Grammar Tested on Sentence Correction
1. No grammatical mistakes There are several grammatical areas that come up
frequently on Sentence Correction items:
2. Correct sentence structure
##verb/tenses
3. The meaning of the sentence should not change.
##subject/verb agreement
The strategies that can be used to attempt these
questions are :- ##modifiers
(1) Look out for multiple errors:- (Note:-Modifiers are words, phrase, or clauses that
provide description in sentences. Modifiers, breathe life
A sentence might contain more than one error. A into sentences and can be adjective, adjective clauses,
common fallacy that test-takers fall into is to find one adverbs, adverb clauses, infinitive phrases, participle
error and then quickly choose the answer that corrects phrases and prepositional phrases .)
that mistake, without considering whether there are other
errors in the sentence that an alternative answer choice ##comparisons .
might also address. ##pronoun usage
(2) Look out the error in the underlined text only and ##idioms
always start with the Subject/Verb.
##clarity
Do begin by identifying the main subject and the
The last two aren't really rules, and they can be the most
predicate verb. The subject is the noun that is doing the
complicated. For that reason, they should be among your
action of the sentence. It may not always be the first
last priorities.
noun you see in the sentence. The predicate verb is the
main action of the sentence that is being done by the Idioms
subject. There could be many verbs in the sentence, so Especially if English is not your first language, idioms
don’t be fooled! can be the most difficult part of Sentence Correction. As
(3) Employ process of elimination wherever possible. noted, you won't learn them all. It isn't even worth your
The easiest way to use process of elimination in sentence time to try.
correction questions is simply to eliminate any answer However, that doesn't mean you shouldn't learn any. As you
choice that is itself grammatically incorrect. A candidate go through practice questions, note those idioms that arise.
can also eliminate any choice that alters the intended There will be some idioms that are particularly common. If
meaning of the sentence. you learn those, you'll be able to handle most of the idioms
(4) Do not worry about spelling or capitalization errors. you'll see in exam paper.

The test writers do not test for these two errors. For Example

(5) Before you select your answer, re-read the whole A common error found in exam on Sentence Correction
sentence. Check to make sure the meaning is questions is the misuse of verb and tense. Verbs/tenses exist
unambiguous and that it’s clear, despite its length. in order to allow us to specify at what point in time some
event occurred – did it happen at one point in the past, or is it
(6) Read the entire sentence the second time, inserting still happening? Is it happening now, or will it happen in the
your selected answer choice. We have found this to be future? etc. Because so many different tenses exist, that are
a very powerful tip for dealing with sentence correction often extremely complicated, using several different tenses
ENGLISH LANGUAGE 123
stportal.mahendras.org
in a single sentence. The correct tense (or tenses) makes the Incorrect: If the cyclist wins the race, it will be representing
sequence of actions clear. an extraordinary comeback from his earlier cancer.
Here's an example of a relatively simple verb/tense error, Correct: If the cyclist wins the race, it will represent an
and its correction: extraordinary comeback from his earlier cancer.
Incorrect: After he had finished his performance, he would Why is the second sentence correct? Because a positive
go to the party. outcome of the race, which is as yet undetermined, is only
Correct: After he had finished his performance, he went to going to "represent his comeback" once – as soon as it
the party. happens. The first sentence implies that the cyclist's victory is
going to keep representing a comeback for the duration of his
Why is the second sentence correct? Because the order of victory – which is confusing, and doesn't make much sense.
events is well clarified. Both events- the performance and the
party - happened in the past, but the performance happened English verb/tense is - clearly - an extraordinarily
first, and the party second. Thus both verbs should be in complex subject. To make your efforts a bit simpler, keep
the past tense: "had finished" in past perfect to indicate that a few general rules in mind: first, to help determine whether
this happened first, and then "went" in past indefinite. The the verbs in a sentence are in the proper tenses, pick one
incorrect sentence implies that the performance happened event as a "base" action, and then try to figure out when
once in the past, but that his after-performance party other events occurred in relation to that event. Try to decide
attendance was ongoing - which doesn't make any sense. whether the events occurred prior to the base action, or after

Here's another example. the base action; or at the same time as the base event took
If the cyclist wins the race, it will be representing an extraordinary place. Keep in mind that actions that start before the base
comeback from his earlier cancer. event may continue after it.
To determine whether this sentence is correct, let's break it Ask yourself: "What happened first, second? What makes
down into its constituent parts: sense logically?". This is only half of the process however:
after you determine when the events took place, you still need
The "if clause" at the beginning of the sentence indicates
to know what verb form corresponds to the time sequence
a hypothetical: a sentence written in if...then... form. This
you've identified. This requires a working knowledge of verb/
kind of sentence requires that the dependent event be in the
tense, as well as mood and voice - it's very important to study
simple future tense: meaning that the event, if it happens,
them. A verb/tense, mood, and voice guide is included in the
will happen once, at some time in the future, following the
extras section; it is recommended that you take a look at it,
first event's ocurrence. It will not keep happening. Here,
even if you already feel comfortable working with verbs.
however, the dependent event is in the future continuous,
not the simple future. Tips for recognizing verb/tense errors:

124 ENGLISH LANGUAGE


stportal.mahendras.org

exercise
A phrase is bold in the sentence and you have to choose (B) serve as a forum for reflection rather then
the option among (A), (B) and (C) whichever is most (C) serve as a forum for reflection rather than
appropriate for the sentence given below.
(1) Only (A) (2) Only (B)
EXERCISE- 1
(3) Only (C) (4) None of the above
Q.1. Having completed the physical examination, the (5) No correction required
tonsils were found to been diseased.
Q.6. If your non-verbal behaviour is timid and weak, even
(A) the tonsils were found to be diseased the strongest performance review may not translate
(B) a tonsils, were finding to disease be into a promotion.
(C) the tonsils were finding to being diseased (A) may not translating into
(1) Only A (2) Both A and C (B) may not translate into
(3) Only B (4) Both A and B (C) may not be translate into
(5) None of them
(1) Only (A) (2) Only (B)
Q.2. All rounders in any cricket team, in actuality, these (3) Only (C) (4) None of the above
are normally difficult to discover. (5) No correction required
(A) them are normally (B) these are normal Q.7. Since there is a lot of awkwardness around
(C) they are normally disability, people from the mainstream community
have developed terminology to make it more
(1) Only A (2) Both A and C
comfortable for them.
(3) Only C (4) Both A and B
(5) None of them (A) to made them more comfortable for them
Q.3. In the attempt to destroy them with (B) to make it more comfort for them
completeness, the Indian team has launched the (C) to make it more comfortable for it
coordinated attack on the opposing team.
(1) Only (A) (2) Only (B)
(A) In the attempt to destroy them with
(3) Only (C) (4) None of the above
completeness
(5) No correction required
(B) In attempting to destroying them completely
Q.8. The media hype around companies that publicize
(C) In an attempt to destroy them completely its initiatives makes it seem like there is a lot
(1) Only A (2) Both A and C happening. But we have barely made a dent.
(3) Only C (4) Both A and B (A) publicizes their initiative makes
(5) None of them
(B) publicize their initiatives makes
Q.4. The response to the challenge is to make the gains
(C) publicize their initiatives make
of globalisation more visible and its transient
downsides politically less painful. Trade leaders (1) Only (A) (2) Only (B)
gathered in Buenos Aires can ill-afford to lose sight (3) Only (C) (4) None of the above
of this imperative. (5) No correction required
(A) their transient downsides politically less Q.9. Product teams come together at the beginning and
(B) their transient downsides politically less end of every week, to jointly outline and reflect on
the week’s goals and challenges.
(C) its transient downsides political less
(A) come together on the beginning and
(1) Only (A) (2) Only (B)
(B) come together at the begin and
(3) Only (C) (4) None of the above
(5) No correction required (C) come togetherness at the beginning and
Q.5. Lending credence to such scepticism is the lukewarm (1) Only (A) (2) Only (B)
stance the U.S. has adopted towards the WTO over (3) Only (C) (4) None of the above
the past year, suggesting that the Ministerial meet (5) No correction required
should serving as a forum for reflection rather Q.10. It’s important to have the body language of a
than to shape substantive agreements. winner—and you can practise it, though it tend to
(A) serve as a forum for reflect rather than be innate rather than a learned gesture.

ENGLISH LANGUAGE 125


stportal.mahendras.org
(A) it tends to be innate rather (2) at places convenient
(B) it tends to being innate rather (3) from a place of convenience
(C) it tend to be innate whether (4) to a place convenient
(1) Only (A) (2) Only (B) (5) No correction required
(3) Only (C) (4) None of the above Q.3. As compared to last year there has been a sharp rise
(5) No correction required in interest rates.
Q.11. The study found that when blind athletes won, they (1) sharper rise of (2) as sharp as rise
would raise their hands to the sky in the classic
expression of pride. (3) sharp rises in (4) sharply rising
(A) they would raising their hands (5) No correction required.
(B) they would raise its hands Q.4. A twenty first century economy cannot be helding
hostage by power cuts nor travel on nineteenth
(C) they would be raise their hands century roads.
(1) Only (A) (2) Only (B) (1) could not be held (2) cannot be held
(3) Only (C) (4) None of the above
(5) No correction required (3) would not held (4) can neither be held
Q.12. Notable among the proposals trade ministers will (5) No correction required
be consider are those relating to new rules on Q.5. When you returned to branch office you must enter
farm subsidies, the elimination of support for all the relevant details in the ledger stored in this
unsustainable fisheries, and the regulation of cupboard
e-commerce.
(1) return (2) returning
(A) will consider are those relating to new (3) on returning (4) while returning
(B) will considering are those relating to new (5) No correction required
(C) will consider are them relates to new Q.6. He listened to my objections patiently and then
explained which he had taken such step.
(1) Only (A) (2) Only (B)
(3) Only (C) (4) None of the above (1) how he had took (2) why he had taken
(5) No correction required (3) what he had taken (4) why he has taken
Q.13. The two countries see the news as a prerequisite to (5) No correction required
address the prevailing imbalance in the Agreement
on Agriculture, which unfairly benefits developed EXERCISE- 3
countries. Choose the correct phrase from the options below. If ‘No
(A) which unfair benefits developed countries correction’ is required choose (5) as the answer.

(B) which unfairly benefit developed countries Q.1. He is in judicial custody in connection to a multi-
crore scam.
(C) which unfairly benefits developed country
(1) on connection at
(1) Only (A) (2) Only (B)
(2) In connection with
(3) Only (C) (4) None of the above
(5) No correction required (3) into connection around

EXERCISE- 2 (4) around connection for


(5) No correction required
Choose the correct phrase from the options below. If ‘No
correction’ is required choose (5) as the answer. Q.2. It prompted the bench of judges to sought for there
classification from CBI.
Q.1. Among his many good qualities, that I remember
is his honesty (1) Seek further (2) Sought further
(1) All of his (2) Some of the (3) Seeks far (4) Seek farther
(3) only of his (4) one of his (5) No correction required
(5) No correction required Q.3. India needs to plan to safeguarded the environment.
Q.2. The organization has agreed to shift the conference (1) Safeguard the environmental
at some place convenience to all participants
(2) Safeguard's the environment
(1) any of the place convenient
(3) Safeguard the environment
126 ENGLISH LANGUAGE
stportal.mahendras.org
(4) Safeguarding the environment (1) joined hands to (2) join hands
(5) No correction required (3) join hands with (4) joining hands along
(5) No correction required
Q.4. He has been given the honorary title lastly held by
his father. Q.2. Nothing was charged for those who took part in the
event.
(1) recent holding (2) one held
(3) recently hold (4) last held (1) take part of (2) taken parts with
(5) No correction required (3) takes parted with (4) took part into
Q.5. All eight-men were sentenced to rigorous (5) No correction required
imprisonment. Q.3. More people will become keen too opt for a home
(1) For rigorously imprision loan.
(2) At rigorous imprisonment (1) keen to opt (2) keen of opt
(3) About rigorous imprision (3) keen at opting (4) keen by opted
(4) With rigorously imprisonment (5) No correction required

(5) No correction required Q.4. The story revolves round two small town girls.

Q.6. There was a total failure of the administration to (1) revolve circling (2) revolved about
acted on time. (3) revolve under (4) revolves around
(5) No correction required
(1) Act in time (2) Acts on timely
Q.5. The player was find guilty of match fixing.
(3) Act on time (4) Acted within time
(5) No correction required (1) finds guilty (2) find guilt

Q.7. He has a collection of photographs captures the (3) found guilty (4) found guilt
essence of Bhutan. (5) No correction required

(1) Essential capture (2) Capturing the essence Q.6. If the present trend continues the cost of a good
personal computer system even can be lower from
(3) Captures essence (4) Captured the essence Rs. 10,000 soon.
(5) No correction required (1) Can be lowest up to Rs. 10,000 soon.
Q.8. His IQ was assessed to become the highest among (2) Can be as low as Rs. 10,000 soon
his fellow students.
(3) Could be lower to Rs. 10,000 immediately
(1) assessed to becomes
(4) Will be as lower as Rs. 10,000 soon
(2) assessed to becoming
(5) No correction required
(3) assessed to became
Q.7. Other countries eradicated this disease ten years
(4) assessed upto be ago.
(5) No correction required (1) have eradicated (2) will eradicate
Q.9. It was fascinating seeing that how classes could (3) had eradicated (4) will be eradicated
form without anyone organizing anything. (5) No correction required
(1) seeing how that (2) for seeing when Q.8. Resolutions must be introducing quickly to repeal
(3) on see that (4) to see that the outdated laws.
(5) No correction required. (1) Be introducing to quick repeal
Q.10. The RBI is doing the correct thing by asking the (2) have to be introduced to quick repealing.
banks to fund the self-help groups directly.
(3) Be quickly introduced to repeal.
(1) was doing (2) have doing
(3) is done (4) has done (4) Be quick introducing to repeal
(5) No correction required. (5) No correction required
EXERCISE- 4 Q.9. Though his actions were severe criticism, he didn't
lose his temper.
Choose the correct phrase from the options below. If ‘No
correction’ is required choose (5) as the answer. (1) were severely criticised
Q.1. They can joint hands with builders who are (2) were at severely criticising
developing residential properties. (3) Had severely criticised
ENGLISH LANGUAGE 127
stportal.mahendras.org
(4) Had severe criticised Q.1. The members of the department has a visual
(5) No correction required analog scale when they ask the patients and the
caregivers to rate its behaviour, tasks they can
Q.10. She always behaves as if she has not care at all do and their state of mind.
about my feelings.
(1) have a visual analog scale when they ask
(1) As though she will not (2) As if she did not the patients and the caregivers to rate their
(3) As far as she doesn't (4) Like if she does not behaviour, tasks they can
(5) No correction required (2) has a visual analog scale where it ask the patients
and the caregivers to rate its behaviour, tasks
EXERCISE- 5 they can
Choose the correct phrase from the options below. If ‘No (3) have a visual analog scale where they ask
correction’ is required choose (5) as the answer. the patients and the caregivers to rate their
Q.1. Solar energy is been used to provide light to behaviour, tasks they can
people. (4) has a visual analog scale which they ask
(1) have being used to (2) be used to the patients and the caregivers to rate their
(3) being used to (4) be using to behaviour, tasks it can
(5) No correction required (5) No correction required
Q.2. Two more projects will be taking up by the Q.2. The international team behind the unprecedented
government soon. space survey said that its discovery literally
sheds new light on some of the Universe’s deepest
(1) took up (2) takes up
secrets, including the physics of black holes and
(3) take up (4) taken up
how clusters of galaxies evolve.
(5) No correction required
(1) the unprecedented space survey said that their
Q.3. CBDT had expanded its investigative branch too discovery literally shed new light on some
deal with the flood of information coming form of the Universe’s deep secrets, including the
foreign countries. physics of black holes
(1) to deal with (2) to dealt with (2) the unprecedented space survey said that its
(3) had deals with (4) have deal with discovery literally shed new light on some of
(5) No correction required the Universe’s deepest secret, including the
physics of black holes
Q.4. India shot down a petrol plane belong to Pakistan
Navy. (3) the unprecedented space survey said that their
discovery literally shed new light on some of
(1) petrol plane belong the Universe’s deepest secrets, including the
(2) patrol plane belonging physics of black holes
(3) patrol planes belonging (4) the unprecedented space survey said their
discovery literally shed new light on some of
(4) petrol plane belonged
the Universe’s deepest secrets, including the
(5) No correction required physics of black holes
Q.5. More policemen will be deployed for the security (5) No correction required
of the foreign minister. Q.3. More than two hundred astronomers from various
(1) will have deploy countries were involved in the study, which used
(2) have deployment radio astronomy to look at a segment of sky over
the northern hemisphere, and found different
(3) will be deploy previously unseen light sources thought to be
(4) will have deployed distant galaxies.
(5) No correction required (1) were involving in the study, which used radio
astronomy to look at a segment of sky over
EXERCISE- 6 the northern hemisphere, and found different
Q1-5. In questions given below, a part of the sentence previous unseen light
is highlighted. Below are given alternatives to (2) were involved in the study, which were used
the highlighted part which may improve the radio astronomy to look at a segment of sky over
sentence. Choose the correct alternative. In case no the northern hemisphere, and found different
improvement is needed, option 'E' is the answer. previously unseen lights

128 ENGLISH LANGUAGE


stportal.mahendras.org
(3) were involved in the study, which used radio galaxies in the Universe, although many are too
astronomy to look into a segment of sky over old and distant to be observed using
the northern hemisphere, and founded different
(5) No correction required
previously unseen light
(4) were involving in the study, which use radio EXERCISE- 7
astronomy to look at a segment of sky over Q.1.5. In the question, a sentence is given with a phrasal
the northern hemisphere, and found different verb written in bold, Read the sentence carefully
previous unseen light and decide if the phrasal verb is correctly
mentioned as per the context of the sentence, If it
(5) No correction required
is not used correctly, choose the correct one from
Q.4. The discovery of the new light sources may also the alternatives suggested below. In case of being
helped scientists better understand the behaviour correctly used, choose ‘No correction required’
of one of space most enigmatic phenomenon. as your answer.
(1) The discovery of the new light sources may also Q.1. In spite of the firing 300 demonstrators succeeded
help scientists better understand the behave of in reaching the border at Chakoti, where Mr. Khan
one of space’s more enigmatic phenomena. eventually agreed to call off the march.
(2) The discovery of the new light sources may also (1) Call at (2) Call away
help scientists better understand the behaviour (3) Call for (4) Call out
of one of space’s most enigmatic phenomena.
(5) No correction required.
(3) The discovery of the new light sources may also
help scientists better understand the behaviour Q.2. No country has a monopoly on bravery; great deeds
of one of space’s most enigmatic phenomenons. of heroism are liable to break down in the most
unexpected places.
(4) The discovery of the new light sources may also
help scientists better to understand the behaviour (1) break away (2) break out
of one of space’s most enigmatic phenomenons. (3) break into (4) break up
(5) No correction required (5) no correction required.
Q.5. The Hubble telescope has producing images that Q.3. The Government buildings along the Embankment
leads scientists to believe there are more than drop away behind us, and here I am in another world,
hundred billion galaxies in the Universe, although another life.
many are too old and distant to be observed using (1) Drop around (2) Drop in
traditional detection techniques.
(3) Drop back (4) Drop off
(1) has produced images that lead scientists to belief
there are more than hundred billion galaxies in (5) No correction required.
the Universe, although many are too old and Q.4. The transition back to work is easier, however, if
distance to be observed using individuals have worked previously and have skills
to fall back .
(2) has been produced images that lead scientists
to believe there is more then hundred billion (1) FALL BACK (2) FALL BACK ON
galaxies in the Universe, although many are too (3) FALL BEHIND (4) FALL DOWN
old and distant to be observed using
(5) No correction required.
(3) has produced images that leads scientists to
believe there are more then hundred billion Q.5. Although tobacco ads are prohibited companies get
around the ban by sponsoring music shows.
galaxies in the Universe, although many are too
old and distant to observe using (1) Get across (2) Get along
(4) has produced images that lead scientists to (3) Get at (4) Get away
believe there are more than hundred billion (5) No correction required.

ENGLISH LANGUAGE 129


stportal.mahendras.org

EXPLANATION
Q.3.(1) keen + to +verb1st form
EXERCISE 1
Q.4.(4) It should be “revolves around”
Q.1.(1)
Q.5.(3) found - It should be the past form because the latter
Q.2.(3) All rounders take pronoun ‘they’.
part of the sentence is in the past tense.
Q.3.(3) Attempt should be preceded by a vowel.
Q.6.(2) Can be lowered from Rs. 10,000 soon. Should be
Q.4.(5) replaced with can be as low as Rs. 10,000 soon.
Q.5.(3) Q.7.(5) No correction required.
Q.6.(2) Q.8.(3) be+adverb+3rd form of verb + infinitive should be
Q.7.(5) used.
Q.8.(2) Q.9.(1) were + adverb+verb 3rd form should be used.
Q.9.(5) Q.10.(2)
Q.10.(1) EXERCISE 5
Q.11.(5) Q.1.(3) is should be replaced with is being used to.
Q.12.(1) Q.2.(4) will always accepts future tense so it is ‘be taken
Q.13.(5) up’
Q.3.(1)
EXERCISE 2
Q.4.(2) it should be patrol plane belonging.
Q.1.(4) one of his because in this sentence we are talking
about only one quality from many. Q.5.(5) No correction required.
Q.2.(4) to a place convenient EXERCISE 6
Q.3.(5) No correction required Q.1.(3) The members of the department have (according
Q.4.(4) can neither be held as neither is followed by nor. to ‘members’- plural verb is used) a visual analog
It’s a case of conjunction scale where (as per expression, ‘where’ is correct
) they (according to ‘members’) ask the patients
Q.5.(3) on returning and the caregivers to rate their (according to
Q.6.(2) why he had taken ‘members’) behavior, tasks they (according to
‘members’) can do and their state of mind.
EXERCISE 3
Q.2.(3) The international team behind the unprecedented
Q.1.(2) In connection with is the suitable phrasal space survey said that (conjunction is required)
preposition. their (as per the expression of ‘members’ which
Q.2.(1) Sought for there should be replaced with seek ‘team’ gives here) discovery literally shed (past
further that means to look further. form is required) new light on some of the
Q.3.(3) Safeguard the environment as ‘to’ always accepts Universe’s deepest (according to the structure of
the first form of verb so 3rd option. ‘one of’ superlative degree is required) secrets
(according to the structure of ‘one of’ superlative
Q.4.(4) Lastly held should be replaced with last held. degree is required), including the physics of black
Q.5.(5) To rigourous imprisonment- No correction holes and how clusters of galaxies evolve.
required. Q.3.(5) More than two hundred astronomers from various
Q.6.(3) countries were involved (passive expression is
required) in the study, which used (past active
Q.7.(2) Capturing the essence should be the correct.
expression is required- the study used) radio
Q.8.(5) No correction required astronomy to look at (to simply direct the eyes
Q.9.(4) Seeing that should be replaced by to see that. towards someone or something so that it can
be seen).a segment of sky over the northern
Q.10.(4) Use 'has done'.
hemisphere, and found (past expression of Find
EXERCISE 4 means ‘to get’) different previously (adverb
is required as per verb ‘unseen’) unseen light
Q.1.(3) Join hands with should be used.
(uncountable noun, can’t be written in plural)
Q.2.(5) No correction required sources thought to be distant galaxies.

130 ENGLISH LANGUAGE


stportal.mahendras.org
Q.4.(2) The discovery of the new light sources may also break away: To separate from a crowd
help (with modals, V1 is used) scientists better break down: To go out of order, cease to function
understand (after ‘help’ bare infinitive is used,
so ‘to’ is not to be used) the behavior (noun break into: To enter by force
is required) of one of space’s (expression of break up: To come to an end (marriage,
possession is required) most enigmatic phenomena relationship)
(plural is required, which is Phenomena).
Q.3.(5) Drop away: Become smaller, get worse.
Q.5.(5) The Hubble telescope has produced (V3 is after
Drop around: Visit someone, often without making
‘has’) images that lead (according to ‘images’
an arrangement
plural verb is required) scientists to believe
(verb is required) there are more than (‘than’ is Drop back: Move towards the back of a group
used after comparative degree) hundred billion Drop off: Take something or someone to a place
galaxies in the Universe, although many are and leave it or them there.
too old and distant (adjective is needed) to be
Q.4.(2) Fall Back On = have recourse to when in difficulty.
observed (passive expression is needed here) using
traditional detection techniques. Fall Back = retreat

EXERCISE 7 Fall Behind = to not meet obligations on time

Q.1.(5) Call off: To cancel Fall Down = fall to the ground or floor

Other options: Fall In = collapse

Call at: Stop somewhere briefly (ship, train, etc.) Q.5.(5) Get around= to become known.

Call away: Ask someone to leave a place. Get across = to communicate.

Call for: Go somewhere to get someone. Get along = to have a good / friendly relationship
with someone.
Call out : Shout something
Get at = to reach, to access to something.
Q.2.(2) break out: To start suddenly
Get away = to go away from someone or something

ENGLISH LANGUAGE 131


stportal.mahendras.org

CHAPTER
Mis-spelt and
18 Inappropriate
Words
Scan the QR code to get video of this chapter.
Correct spelling Incorrect spelling Correct spelling Incorrect spelling
accommodate, accommo- foreseeable forseeable
accomodate, accomodation forty fourty
dation
achieve acheive forward foward
across accross friend freind
aggressive, aggression agressive, agression further futher
apparently apparantly gist jist
appearance appearence glamorous glamourous
argument arguement government goverment
assassination assasination guard gaurd
basically basicly happened happend
beginning begining harass, harassment harrass, harrassment
believe beleive, belive honorary honourary
bizarre bizzare humorous humourous
business buisness idiosyncrasy idiosyncracy
calendar calender immediately immediatly
Caribbean Carribean incidentally incidently
cemetery cemetary independent independant
chauffeur chauffer interrupt interupt
colleague collegue irresistible irresistable
coming comming knowledge knowlege
committee commitee liaise, liaison liase, liason
completely completly lollipop lollypop
conscious concious millennium, millennia millenium, millenia
curiosity curiousity Neanderthal Neandertal
definitely definately necessary neccessary
dilemma dilemna noticeable noticable
disappear dissapear occasion ocassion, occassion
disappoint dissapoint occurred, occurring occured, occuring
ecstasy ecstacy occurrence occurance, occurence
embarrass embarass pavilion pavillion
environment enviroment persistent persistant
existence existance pharaoh pharoah
Fahrenheit Farenheit piece peice
familiar familar politician politican
finally finaly Portuguese Portugese
fluorescent florescent possession posession
foreign foriegn preferred, preferring prefered, prefering
propaganda propoganda

132 ENGLISH LANGUAGE


stportal.mahendras.org
publicly publically siege seige
really realy
receive recieve successful succesful
referred, referring refered, refering supersede supercede
religious religous surprise suprise
remember rember, remeber tattoo tatoo
resistance resistence tendency tendancy
sense sence therefore therefor
separate seperate threshold threshhold

Spell Error
Accept : (verb) - to receive willingly, to approve, to aggree.
Except : (preposition or verb) - exclusion or leave out.
Ad : An advertisement.
Add : to combine, join, unite or to find a sum.
Advice : (noun) - suggestion or recommendation.
Advise : (verb) - to suggest.
Affect : (verb) - to change.
Effect : (noun) - result.
Ate : Past tense of verb [to eat].
Eight : number 8.
Buy : (verb) - to purchase.
By : next to something, by way of something.
Bye : Used to express farewell. Short for [goodbye].
Choose : (verb) to make a choice or selection.
Chose : past tense of the verb [to choose].
Choice : (noun) choosing; selection.
Cite : to mention something or to quote somebody as an example or proof.
Site : the location of an event or object. A website.
Sight : ability to see, a thing that can be seen.
Decent : kind, tolerant, respectable, modest.
Descent : family origins or ancestry. Also the process of coming or going down.
Dissent : (verb or noun) disagreement with a prevailing or official view.
Desert : (verb) to leave or abandon, (noun) waterless land with no vegetation and covered
with sand.
Dessert : sweet food served after the meal.
Eat : to put food into the mouth, chew it and swallow it.
It : the thing, animal or situation which has already been mentioned.
Four : number 4
For : to indicate the object, aim, or purpose of an action or activity.
Here : in, at, or to this place.
Hear : to perceive (sound) by the ear.
Knew : past simple of the verb [to know].
New : recently created.
Know : (verb) to be familiar with someone or something.
Now : at the present time or moment.
No : negative reply, refusal or disagreement. ise and admiration. (verb) desire to know. ities.
ENGLISH LANGUAGE 133
stportal.mahendras.org

exercise
In each question below a sentence with four words printed in bold type is given. These are numbered (1),(2),(3)
and (4) . One of these four words printed in bold may be either wrongly spelt or inappropriate in context of the
sentence. Find out the word which is wrongly spelt or inappropriate if any. The number of that word is your
answer. If all the words printed in bold are correctly spelt & also appropriate in the context of the sentence, mark
(5) ”All correct” as your answer.
EXERCISE- 1
Q.1. According to a sleep expert(1) not everyone kneed(2) eight hours’ sleep a night(3) you just need to listen(4) to
your body. All correct (5)
Q.2. Some people are very sensitive(1) to the effects(2) of caffeine and for these people it’s important to avoid(3)
bevereges(4) containing caffeine. All correct (5)
Q.3. If you are feeling sleepy(1) during the day then(2) there is most like(3) an issue with the quality of sleep that
you’ve had during(4) the night. All correct (5)
Q.4. In the future, mobile broadband(1) will enable fully automated(2) homes, smart parking(3) among other
consumer and industrial applications.(4) All correct (5)
Q.5. Banks are going slow in purchasing(1)/ bonds ahead of the Budget as there is no certainity(2)/ about the bor-
rowing program that(3)/ may come in February.(4) All correct (5)
Q.6. The Parliament were divided in the opinions about the issue. All correct.
(1) (2) (3) (4) (5)
Q.7. The officials maintained that the rules were within the framwork of the Indian laws. All correct.
(1) (2) (3) (4) (5)
Q.8. Domestic airlines are incurring substential losses. All correct.
(1) (2) (3) (4) (5)
Q.9. Won third of the water in the pool is still polluted. All correct.
(1) (2) (3) (4) (5)
Q.10. More than halve the budget has been spent on modernising the factory. All correct.
(1) (2) (3) (4) (5)
EXERCISE- 2
Q.1. The Reserve Bank of India has clarified (1)/ that it won’t be interested (2) in giving some (3) relaxations to
banks on these losses.(4) All correct (5)
Q.2. While there are unlikely(1) to be any major changes in indirect tax(2)/ as most of them are beneath the pur-
view of(3)/ the Goods and Services Tax Council.(4) All correct (5)
Q.3. A schematic(1) to make good the difference(2) between the minimum support price announced(3) by the gov-
ernment and the actual market price would be in order(4). All Correct(5)
Q.4. PSBs need systemic(1) reform to overhaul their current decision-making structure and culture. The basic reform
they need is to change their holding structure(2) to give them autonomy(3), as well as acountability(4), not a
promise of no government interference. All Correct(5)
Q.5. Regional allies(1) active in the campaign(2) against Islamic extremists(3) in the east African country have
conducted(4) many missions too.All Correct(5)
Q.6. Farmers must get remunerative(1) prices for there(2) produce. Rural power supply, rural roads and food pro-
cessing(3) plants would be welcome(4).All Correct(5)
Q.7. The remuniration(1) structure of senior bankers at PSBs must move from public sector scales to board-deter-
mined(2), performance-linked, market-comparable(3) ones that reflect the time horizon of the risks assumed(4).
All Correct(5)
Q.8. To deal effectively with a crisis quick decisions are requited. All correct.
(1) (2) (3) (4) (5)

134 ENGLISH LANGUAGE


stportal.mahendras.org
Q.9. More than half the loan has been spent on rebuilding the factory. All correct.
(1) (2) (3) (4) (5)
Q.10. Rising prices of foodgrains will have an adverse impac on developing countries. All correct.
(1) (2) (3) (4) (5)
EXERCISE- 3
Q.1. Some household names are being persued(1) for six-figure income tax and national insurance(2) bills by HM
Revenue & Customs on the grounds(3) that they were in effect BBC employees while claiming to be freelanc-
ers(4) All correct (5)
Q.2. The government(1) has plans to expand the pipeline network in the country, especial(2) in the eastern part, in
a move to increase the consumption(3) of the environment-friendly(4) fuel. All correct (5)
Q.3. The idea that a large group(1) could being driven(2) westwards by newcomers(3) doesn’t make sense.(4) All
correct (5)
Q.4. Authorities ordered supplementary investigations into the case leaving the traders' families with an
(1) (2) (3)
uncertain wait. All correct
(4) (5)
Q.5. The company has announced its entry into the breakfast foods market with the launched of Horlicks oats.
(1) (2) (3) (4)
All correct
(5)
Q.6. A government panel wants the centre to initiate legal measures to spilt the other backward classes into
(1) (2) (3)
subgroups. All correct.
(4) (5)
Q.7. Airport operators, oil companies and vendars of all imaginable items have massive dues running with the
(1) (2) (3) (4)
company. All correct
(5)
Q.8. When your uncle einquired about your marks you lied to him. All correct.
(1) (2) (3) (4) (5)
Q.9. The posts were advertised by the planning comission in all the local dailies. All correct.
(1) (2) (3) (4) (5)
Q.10. He is the most intelligent boy in the class. All correct.
(1) (2) (3) (4) (5)
EXERCISE- 4
Q.1. By analyzing(1) the genomes of over 300 individuals(2) – many of which had been buried(3) with distictive(4)
objects tying them to the Beaker Culture. All Correct (5)
Q.2. There is a chance the government(1) may introduce a long term capital gains taxes(2) on equity shares,(3) or
may remove the dividend(4) distribution tax. All correct(5)
Q.3. Namrata was brought (1) up in Bareilly, where she began(2) writing primarily in English then she was married
to(3) Mr. Das at the age of 15 and thus(4) became Namrata Das. All correct (5) LANGUAGE
Q.4. High crude prices amid the oil crises in Iran pose inflationary risks in India. All correct
(1) (2) (3) (4) (5)

ENGLISH LANGUAGE 135


stportal.mahendras.org
Q.5. Eastern India is liberaly endowed with natural resources-deep fertile top soil, copious water and plentiful
(1) (2) (3) (4)
sunlight. All correct
(5)
Q.6. There is no doubt that Civil society plays a critical role in India’s diverce democracy, and its voice deserves to
(1) (2) (3) (4)
be heard. All correct
(5)
Q.7. Banking and insurance sector companies were the worst effected as clerical staff in all
(1) (2) (3)
major public sector banks and insurance companies abstained from work. All correct
(4) (5)
Q.8. Every year, equity strategists and analysis come out with their industry-specific expectations from the
(1) (2) (3) (4)
Union Budget. All correct
(5)
Q.9. The money policy may add fuel to the rally in case cut off of 25-30 basis points comes through in the cash reserve
(1) (2) 3) (4)
ratio. All correct
(5)
Q.10. Exports of several agricultural commodities have been facing hurdles. All correct
(1) (2) (3) (4) (5)

136 ENGLISH LANGUAGE


stportal.mahendras.org

EXPLANATION
Q.5.(4) ‘launching’ should be used.
EXERCISE 1
Q.6.(3) ‘split’ is to be used to convey the meaning ‘divide’.
Q.1.(2) The correct word is ‘need’.
Q.7.(2) ‘vendors’ is the correct spelling.
Q.2.(4) The correct word is ‘beverages’.
Q.8.(2) Inquired is the appropriate spelling
Q.3.(3) ‘likely’ is the correct word.
Q.9.(3) Commission is the appropriate spelling
Q.4.(5) All correct
Q.10.(5) All correct.
Q.5.(3) spelling error the correct word is ‘certainty’.
EXERCISE 4
Q.6.(5) All correct
Q.1.(4) Distinctive is the correct option.
Q.7. (4) Framework is the correct spelling.
Q.2.(2) Here we need singular noun not plural, the correct
Q.8.(4) Substantial is the correct spelling.
term will be as ‘a long term capital gains tax’ be-
Q.9.(1) One third should be used. cause this is the compound noun having indefinite
Q.10.(1) Half is the correct spelling. article ‘a’.

EXERCISE 2 Q.3.(3) In passive voice the verb ‘married’ followed by


‘with’ when subject is female as in the sentence
Q.1.(3) use ‘any’ in place of ‘some’ because in negative ‘she was married to Mr. Das….’ is wrong it
sentence ‘any’ is used whereas in positive ‘some’ should be ‘Namrata was married with Mr. Das….’
used. Note: no preposition is used after marry in active
Q.2.(3) use ‘under/within’ in place of ‘beneath’. voice.
The correct phrase is ‘under/within the purview’ Q.4.(3) The correct spelling of the word is crisis.
means within the scope of the influence or con- Q.5.(1) The correct spelling of the word is liberally.
cerns of something.
Q.6.(3) The correct spelling of the word is diverse.
Q.3.(1) The correct word is ‘scheme’.
Q.7.(3) Affected would the appropriate word to be used
Q.4.(5) The correct word is ‘accountability’. in the sentence given above.
Q.5.(5) All Correct Q.8.(2) Analysts should be used in place of analysis.
Q.6.(2) The correct word is ‘their’. Analysts- are the people who analyse.
Q.7.(5) The correct word is ‘remuneration’. Analysis- is the process of assessing a certain thing,
Q.8.(4) Required work or activity.
Q.9.(5) All correct Q.9.(1) Monetary would be the appropriate word [as re-
quired in the sentence].
Q.10.(2) Impact
Monetary-is used as an adjective and means con-
EXERCISE 3 nected with money, especially all the money in the
Q.1.(1) Pursued is the correct word. country. Money- is what you earn by working or
Q.2.(2) especially is an appropriate word here. selling things, and use to buy things.

Q.3.(2) be is the correct word. Q.10.(5)

Q.4.(3) ‘traders’ would be used to imply of the traders.

ENGLISH LANGUAGE 137


stportal.mahendras.org

CHAPTER SPOTTING
19 SIMILAR AND
DISSIMILAR WORDS
Scan the QR code to get video of this chapter.
The Synonyms and Antonyms form an integral part of the Synonyms can be any part of speech (Eg. nouns, verbs,
English Language. Acquaintance with vocabulary of the adjectives, adverbs or preposition), as long as both members of
English language is a necessity for effective expression either the pair are the same part of speech. More examples of English.
in the written or in the oral form. Noun : "Convenience" and
Synonym is nothing but the similar meaning of a particular word "comfort"
or its semantic relation. So, it is a word or a phrase that means Verb : "buy" and "purchase"
the same as another word or a phrase in the same language.
Adjective : "sick" and "ill"
Antonyms are the negative connotation of a particular word.
Adverb: "quickly" and "speedily”
An Antonym is a word or phrase that is opposite in meaning
to a particular word or a phrase in the same language. Preposition: "on" and "upon"
Synonyms are different words with almost identical or similar An antonym is a word that means the opposite of another.
meanings. Words that are synonyms are said to be synonymous, Example 1 : 'fat' is an antonym of 'thin'
and the state of being a synonym is called synonyms. Example 2 : The words open and closed are antonyms.
Word Synonym

Word Synonym Synonym Word Synonym Synonym


Abandon discard Vacate Bluff boast Feign
Accord agree Grant Bold daring fearless
Adversity difficulty misfortune Bonus award Gift
Affluent plentiful Rich Bother annoy Irritate
Aggravate annoy infuriate Brief concise Short
Alleviate lighten mitigate Brilliant clever intelligent
Amenable agreeable favorable Brisk fast Swift
Anguish distress Sorrow Budget allot Plan
Apathetic dispirited lifeless Candid honest truthful
Arrogant disdainful imperious Caricature cartoon imitation
Astonish confound overwhelm Casual informal natural
Atrocious cruel brutal Category classification division
Augment add enlarge Cease desist Stop
Avoid ignore Shun Chaotic disordered Messy
Awkward graceless Inept Cherish esteem Love
Baffle confuse perplexing Circumvent avoid go around
Banal common Plain Commemorate celebrate Honor
Barren desolate Sterile Compensate balance recompense
Berate criticize disapprove Competent able capable
Betray deceive cheat Conceive design Plan
Bias inclination predisposition Confirmation acknowledgement Proof
Bitter acrid Sour Contradict deny oppose
Blend combine Mix Contribution donation Grant
Bliss happiness Joy Courteous polite well-
mannered
138 ENGLISH LANGUAGE
stportal.mahendras.org

Craving desire longing Emanate arise Radiate


Credulous gullible naive Embezzle purloin Steal
Damp moist Wet Eminent distinguished Prominent
Dare challenge Defy Encourage foster Induce
Decay decline Rot Endure last Persist
Decent honorable Pure Essential cultured Learned
Dense filled Packed Essential basic Necessary
Designate name Select Estimate guess Predict
Detain hold Keep Evaluate appraise Judge
Disclose announce Reveal Exhaust deplete Empty
Dogma belief View Exhilarated cheerful Zestful
Durable constant Lasting Explicit definite Specific
Dwindle abate Diminish Fastidious exacting Particular
Eager earnest Keen Federation alliance Band
Eccentric abnormal Idiosyncratic Feeble helpless Infirm
Elaborate embellish Enhance Fervor intensity Passion

Words Antonyms List Words Antonyms List


happy sad, miserable hard soft
harmful harmless hasten dawdle
hate love healthy unhealthy, ill, diseased
here there heavy light
height depth brave coward
hill valley horizontal vertical
hinder aid, help honest dishonest
humble proud hunger thirst
imitation genuine immense tiny, minute
imprison free include exclude
increase decrease inhabited uninhabited
inferior superior inside outside
intelligent unintelligent, stupid inhale exhale
interior exterior, outside interesting uninteresting, dull
internal external intentional accidental
join separate junior senior
justice injustice king Queen
knowledge ignorance laugh cry
lawful unlawful lazy industrious, energetic
earth sky landloar tenant
large little, small last first
lawyer client lecturer student
lender borrower lengthen shorten
left right less more
light dark, heavy like dislike, unlike
likely unlikely leader follower
little large, much, big lofty lowly
long short loud soft/quite
ENGLISH LANGUAGE 139
stportal.mahendras.org

loss find, gain low high


loyal disloyal mad sane
magnetize demagnetize master servant
mature immature maximum minimum
me you merry mirthless, sad
minority majority miser spendthrift
misunderstand understand narrow wide
near far, distant neat untidy
new old night day
noisy quiet north south
obedient disobedient odd even
offer refuse open shut
optimist pessimist out in
parent child past present
patient impatient peace war
permanent temporary please displease
plentiful scarce poetry prose
possible impossible poverty wealth
powerful feeble, weak polite impolite, rude
private public prudent imprudent
pretty unsightly, ugly pure impure
qualified unqualified rapid slow
regularly irregularlry rich poor
right wrong, left rigid soft/flexible
rough smooth satisfactory unsatisfactory
security insecurity scatter collect
serious trivial second-hand new
sense nonsense shopkeeper customer
singular plural simple complicated
thin thick solid liquid
sober drunk speaker listener
sour sweet sorrow joy
sow reap stand lie
straight crooked strong weak
success failure sunny cloudy
take give tall short
tame wild teacher pupil
thick thin tight loose
top bottom transparent opaque
truth untruth up down
vacant occupied valuable valueless
victory defeat virtue vice
visible invisible voluntary compulsory
vowel consonant wax wane
wisdom folly within outside

140 ENGLISH LANGUAGE


stportal.mahendras.org

exercise
(3) Elementary (4) Daunting
EXERCISE- 1
(5) None of the above
Identify the synonyms of each word.
Q.10. SOMBRE
Q.1. EXIGUOUS
(1) Causing sleep (2) Squalid
(1) Tall (2) Large
(3) Gloomy (4) Complacent
(3) Wide (4) Scanty
(5) Malicious
(5) Broad
Q.2. RECREANCY EXERCISE- 2
(1) Recreation (2) Recuperation Identify the synonyms of each word.
(3) Bravery (4) Cowardice Q.1. CONFRONTATION
(5) Obstinate (1) Disapprove (2) Battle
Q.3. PROSCRIBE (3) Deny (4) Disobey
(1) To nominate (5) Critisize
(2) To be supportive of Q.2. UNTIE
(3) To give early warning signals (1) Attach (2) Unchain
(4) Outlaw (3) Clutter (4) Bind
(5) None of the above (5) Fasten
Q.4. INGEST Q.3. ALERT
(1) Enrage (2) Invigorate (1) Energetic (2) Observant
(3) To absorb (4) Burn up completely (3) Intelligent
(5) To stir up (4) Watchful (5) Exclaim
Q.5. MONTICULE Q.4. MOVING
(1) A small river (2) A small hut (1) Taking (2) Toying
(3) A lane (4) A small hill (3) Shifting (4) Turning
(5) A small plane (5) Running
Q.6. COMPENDIOUS Q.5. RECKLESS
(1) Comprehensive (2) Illustrative (1) Courageous (2) Rash
(3) Unbearable (4) Elaborate (3) Bold (4) Daring
(5) None of the above (5) Cautious
Q.7. NADIR Q.6. LAUNCH
(1) Asylum (2) Heaven (1) Review (2) Begin
(3) Depth (4) Nebulous (3) Propel (4) Push
(5) None of the above (5) Force
Q.8. SOMNAMBULISTIC Q.7. RELIED
(1) Sleep walking (1) Emphasised (2) Depended
(2) Ghost dancing (3) Convinced (4) Followed
(3) Women’s group activity (5) Referred
(4)Colourful scenario Q.8. Obtained
(5) Over-eating (1) Combined (2) Procured
Q.9. PRIMORDIAL (3) Acquired (4) Conquered
(1) Feeling of elation (2) Latest (5) Attained

ENGLISH LANGUAGE 141


stportal.mahendras.org
Q.9. Commemorate Q.9. Rigid
(1) Boast (2) Harmonise (1) soft (2) logical
(3) Manipulate (4) Remember (3) sympathetic (4) flexible
(5) Illegal (5) stable
Q.10. Connoisseur Q.10. Adversity
(1) Lover of art (2) Interpreter (1) Wealth (2) Prosperity
(3) Delinquent (4) Ignorant (3) Luxury (4) Money
(5) Gallant (5) growth
EXERCISE- 3 EXERCISE- 4
Identify the antonym of each word . Identify the antonym of each word .
Q.1. Repulsive Q.1. CONFESS
(1) lovely (2) mild (1) Refuse (2) Deny
(3) admirable (4) attractive (3) Contest (4) Contend
(5) joyous (5) Narrate
Q.2. Amateur Q.2. ABSOLUTE
(1) average (2) experienced (1) Deficient (2) Faulty
(3) professional (4) skilled (3) Limited (4) Scarce
(5) appraised (5) Final
Q.3. Affirmed Q.3. VALUABLE
(1) assert (2) attest (1) Invaluable (2) Worthless
(3) confirm (4) denied (3) Inferior (4) Lowly
(5) swear (5) Precious
Q.4. Vague Q.4. HINDRANCE
(1) clear (2) transparent (1) Aid (2) Persuasion
(3) plain (4) apparent (3) Cooperation (4) Agreement
(5) exact (5) Hurdle
Q.5. Casual Q.5. ALIEN
(1) careful (2) sincere (1) Native (2) Domiciled
(3) precise (4) flawless (3) Natural (4) Resident

(5) specific (5) International

Q.6. Pamper Q.6. GARBLE

(1) scold (2) scorn (1) Enjoy (2) Rinse

(3) discourage (4) neglect (3) Clarify (4) Accept

(5) ignore (5) Stutter

Q.7. Prevent Q.7. FORTITUDE

(1) facilitate (2) accelerate (1) Timidity (2) Laxity

(3) allow (4) aggravate (3) Placidity (4) Ambition

(5) conduct (5) firm

Q.8. Austere Q.8. PERNICIOUS

(1) luxurious (2) boisterous (1) Precious (2) Healing

(3) exciting (4) calm (3) Swerving (4) Conservative

(5) happy (5) Relaxed


142 ENGLISH LANGUAGE
stportal.mahendras.org
Q.9. ANATHEMA Q.5. INCOMMODE
(1) Appreciation (2) Blessing (1) Cause trouble (2) Comfortable
(3) Protection (4) Obstacle (3) Inconvenience (4) Small
(5) Frank (5) Fresh
Q.10. CONCUR Q.6. EXPLICIT
(1) Pertain (2) Reveal (1) Clear (2) Straightforward
(3) Oppose (4) Delay (3) Hidden (4) Closed
(5) Deduce (5) Blunt
EXERCISE- 5 Q.7. CHURLISH
Identify the antonym of each word. (1) Ill-mannered (2) Rude
Q.1. DENOUNCE (3) Grumpy (4) Decent
(1) Accept (2) Accuse (5) Sullen
(3) Condemn (4) Faith Q.8. ASTATIC
(5) ridicule (1) Dynamic (2) Unstable
Q.2. DIPSOMANIAC (3) Stable (4) Directionless
(1) Alcoholic (2) Teetotaller (5) Glad
(3) Sick (4) Lunatic Q.9. MOTLEY
(5) sincere (1) Homogeneous (2) Deadly
Q.3. PRECIPITOUS (3) Gloomy (4) Concise
(1) Rash (2) Steep (5) Dreary
(3) Thoughtful (4) Rain Q.10. BELITTLE
(5) poor (1) Allure (2) Disturb
Q.4. MAGNANIMOUS (3) Entangle (4) Ascend
(1) Generous (2) Giving (5) Magnify
(3) Stingy (4) Greedy
(5) Heavy

ENGLISH LANGUAGE 143


stportal.mahendras.org

EXPLANATION
For other options
EXERCISE 1
Squalid- means poor or run-down
Q.1.(4) Exiguous- small, slender.
which can also mean scanty Malicious means awful or venomous.

All other words are either antonyms or do not Complacement means pleasant.
relate to the word exiguous. EXERCISE 2
Q.2.(4) Recreancy- unfaithful, traitorous. Q.1.(2) Confrontation-- A bold challenge or battle Which
which can also mean cowardice. can also mean challenge.
For other options All other words are not related to word
confrontation.
Recreation-means the act of creating something new.
Bravery- means courage. Q.2.(2) Untie- to undo the string or cords of

Obstinate- means inflexibly persistence or an Which can also mean unchain.


unyielding. All other words are antonyms of the word untie.
Recuperation means to recover from financial loss Q.3.(4) Alert- fully aware and attentive
or sickness. Which can also mean watchful.
Q.3.(4) Proscribe-to put outside the protection of the law. For other options
which can also mean outlaw. Exclaim means to cry out or speak suddenly and
All other words are not related to word proscribe. vehemently, as in surprise or protest.
Q.4.(3) Ingest- to take, as food, into the body. Q.4.(3) Moving- causing or producing motion.
which can also mean to absorb. Which can also mean shifting.
For other options All other words are either antonyms or do not
Invigorate means to fill with life and energy, relate to word moving.
energize. Q.5.(2) Reckless- Utterly unconcerned about the
Enrage means to make extremely angry. consequences of some action.
Q.5.(4) Monticule-a small mountain, hill or mound. Which Which can also mean rash.
can also mean a small hill All other words are not related to words reckless.
All other words are either antonyms are not related
Q.6.(2) Launch-to start on a course, career, etc
to the word Monticule.
Which can also mean begin.
Q.6.(1) Compendious- concise
Q.7.(2) Relied-to depend confidently.
Which can also mean comprehensive.
Which can also mean depended.
All other words are either antonyms or do not
relate to the word compendious. Q.8.(5) Obtained-to attain or reach
Q.7.(3) Nadir- the lowest point. Which can also mean attained.
Which can also mean depth. For other options
For other options Procured- means to bring about, especially by
Nebulous means indistinct or confused. unscrupolous and indirect means.
Asylum means a shelter from danger or hardship. Conquered- means to acquire by force of arms.
Q.8.(1) Somnambulistic- sleep walking. Q.9.(4) Commemorate- to serve as a memorial or reminder
of which can also mean remember.
All other words are not related to the word
somnambulistic Q.10.(1) Connoisseur-a person who is especially competent to pass
Q.9.(3) Premordial- original critical judgements in an art.

Which can also mean elementary. Which can also mean lover of art.
All other words are either antonyms or do not For other options
relate to word premordial. Delinquent means failing in or neglectful of a duty
Q.10.(3) Sombre- shadowy. or obligation.
Which can also mean gloomy Gallant means brave or chivalrous.
144 ENGLISH LANGUAGE
stportal.mahendras.org
Q.6.(3) Garble means confuse which is opposite to clarify.
EXERCISE 3
Q.1.(4) Repulsive- means tending to drive away or keep All other words are not related to word Garble.
at a distance which is opposite to attractive. Q.7.(1) Fortitude means strength of mind or guts which
All other words are synonyms of word Repulsive. is opposite to timidity.
Q.2.(3) Amateur- means a person inexperienced or For other options
unskilled in a particular activity which is opposite Laxity means easy attitude.
to professional. Placidity means calm.
All other words are synonyms to word Amateur.
Q.8.(2) pernicious means bad or harmful which is opposite
Q.3.(4) Affirmed- means declare the truth of something to word healing.
which is opposite to denied.
For other options
All other words are synonyms to word affirmed.
Swerving- means turn aside or often to avoid
Q.4.(1) Vague- means not definite or clear which is collision.
opposite to clear.
Conservative- cautious or moderate.
All other words are synonyms to word vague.
Q.9.(2) Anathema means something hated which is
Q.5.(2) Casual- means without definite or serious intention
opposite to word blessing.
which is opposite to sincere.
All other words are either synonyms or do not
All other words are synonyms to word casual.
relate to word Anathema.
Q.6.(4) Pamper means Kindness which is opposite to neglect
Q.10.(3) Concur means agree approve which is opposite to
All other words are synonyms to word Pamper. word oppose.
Q.7.(3) Prevent- means to keep stop from doing occuring EXERCISE 5
which is opposite to allow.
Q.1.(1) Accept
All other words are synonyms to word prevent.
Q.2.(2) Dipsomaniac means a person who has a strong
Q.8.(4) Austere- means severe in manner or appearance
alcoholic drink therefore teetotaller means a
which is opposite to calm.
person who never drinks.
All other words are either synonyms or do not
relate to word Austere. Q.3.(3) Precipious- Done very quickly so thoughtful will
be correct synonyms/
Q.9.(4) Rigid means stiff or unyielding which is opposite
to flexible. For other options :
All other words are not related to word rigid. steep- rising or falling quickly.
Q.10.(2) Adversity means adverse fortune or fate which is Q.4.(3) Stingy- means not generous with money so this is
opposite to word prosperity. the opposite of magnanimous

EXERCISE 4 For other options :

Q.1.(2) Confess means to admit which is opposite to Deny generous- charitable.


All other words are either synonyms or do not Q.5.(2) Incommode means restless so comfortable is the
relate to word confess. opposite.
Q.2.(3) Absolute means without limit which is opposite Q.6.(3) Explicit means open and Hidden means closed, it
to limited. will be the opposite.
All other words are not related to word Absolute. Q.7.(4) Churlish means rude or bad tempered so ‘Decent’
Q.3.(2) Valuable means very important or priceless which will be its antonym.
is opposite to worthless. Q.8.(3) ‘Stable’ is the opposite word of ‘astatic’
All other words are either synonyms or do not Q.9.(1) Motley- Consisting of many different types of
relate to word valuable. people. ‘Homogeneous’ means same type of
Q.4.(1) Hindrance means obstruction or difficulty which people so is opposite word.
is opposite to Aid. For other options :
Q.5.(1) Alien means foreign which is opposite to native. Dreary - that makes you feel bad.
All other words are either synonyms or do not Q.10.(5) belittle-To make someone or something
relate to word Alien. unimportant so ‘magnify’ will be opposite.

ENGLISH LANGUAGE 145

You might also like